Ευκλειδης Β 81

84
« Νόμιζα ότι εα λίγες πιθανότητες, αλλά ταυτόχρονα πίστευα ότι προσπαθούσα λίγο παραπάνω, μπορεί και να τα κατάφερνα ... » Γ Βλάχος Ο Γιώ ρ γ ο ς Βλάχος (κέντρο) που κατέκτησε το χρυσό μετάλλιο. Αριστερά, ο Γιώργος Καλαντζής που κατέκτησε χάλκινο μετάλλιο και δεξιά ο Αλέξανδρος Μο υσάτοβ (χάλκινο μετάλλιο). Επίσης χάλκινο κατέκτησε ο Παναγιώτης Λώλας και ο Σπύρος Κακαρούμπας (εύφημη μνεία) αριστερά του Γ. Βλάχου, και ο Νίκος Αθανασίου, δεξιά, που διακρίνονται στην ομαδική φωτογραφία. nΛΗΡΩΜΕΝΟ ΤΕΛΟΣ Τσχ.Γφι ΚΕΜΠ.Αθ. ιθμόςιας 4156 argentina July 2012 ΕΝΠΟ ΚΛΕΙΠΟ ΑΡ. ΑΔΕΙΑΣ 1099/96 ΚΕΜΠ.ΑΘ.

description

Ευκλειδης Β 81

Transcript of Ευκλειδης Β 81

Page 1: Ευκλειδης Β 81

« ••• Νόμιζα ότι είχα λίγες πιθανότητες, αλλά ταυτόχρονα πίστευα ότι αν προσπαθούσα λίγο παραπάνω, μπορεί και να τα κατάφερνα ... »

Γ. Βλάχος

Ο Γιώ ρ γ ο ς Βλάχος (κέντρο) που κατέκτησε το χρυσό μετάλλιο. Αριστερά, ο Γιώργος Καλαντζής που κατέκτησε χάλκινο μετάλλιο και δεξιά ο Αλέξανδρος Μο υσάτοβ (χ ά λ κ ι ν ο μετάλλιο). Επίσης χάλκινο κατέκτησε ο Παναγιώτης Λώλας και ο Σπύρος Κακαρούμπας (εύφημη μνεία) αριστερά του Γ. Βλάχου, και ο Νίκος Αθανασίου, δεξιά, που διακρίνονται στην ομαδική φωτογραφία.

nΛΗΡΩΜΕ ΝΟ ΤΕΛΟΣ

Τσχ.Γραφιlο ΚΕΜΠ.Αθ.

Αριθμός"-δtιας 4156

argentina July 2012

ΕΝ"ΙΥΠΟ ΚΛΕΙΠΟ ΑΡ. ΑΔΕΙΑΣ 1099/96 ΚΕΜΠ.ΑΘ.

Page 2: Ευκλειδης Β 81

ΕΛΛΗΝΙΚΗ ΜΑΘΗΜΑΤΙΚΗ ETAIPEIA Τ ε ύ χ ο ς 81 - Ι ο ύ λ ι ο ς - Α ύ γ ο υ σ τ ο ς - Σ ε π τ έ μ β ρ ι ο ς 2011- Ε υ ρ ώ: 3,50

e-mail: [email protected] www.hms.gr

ΜΑ ΘΗΜΑΤΙΚΟ ΠΕΡΙΟΔΙΚΟ ΓΙΑ ΤΟ ΛΥΚ ΕΙΟ

ΠΕΡΙΕΧΟΜΕΝΑ ·

Γενικά θέματα I Τ α παιδία παίζει

Ι Χάοc;, yαία, iρως

Ι Μαθηματικiς Ολυμπιάδες

Ι Homo Mathematicus Α' Τάξη

Ι λλyεβρα: Πιθανότητες- Ταυτότητες

Ι r εωμετρία: Ισότητα τριyώνων

Ανισοτικiς Ιχiσεις

Β' Τάξη Ι λλyεβρα: Τριyωνομετρία

Πολυώνυμα

Ι Γεωμετρία: Μετρικiς σχiσεις

Ι Κατεύθυνση: Διανύσματα

Γ' Τάξη Ι r ενική Παιδεία: Διαφορικός Λοyισμός

Ι Κατεύθυνση: Μιyαδικοί

Δραστηριότητες Ι Ο Ευκλείδης προτείνει ...

I Τ α Μαθηματικά μας Διασκεδάζουν

Ι Στήλη του Μαθητή

7 13 36

40 46 49 54 59

61 66

76 78 81

[Π- ΟιΗΣΗ]

Από καιρό υπολόγιζα

Το γράμμα - αύμβολο πι

3 και 14 αε: κάθε: κύκλο

Από καιρό λογάριαζα

Τα πε:ριααότε:ρα αύγχρονα

ελληνικά ονόματα

αρχίζουν από πι

Παλαιότερα ο Πλάτων

ο Πραξιτέλης, ο Πυθαγόρας

ο Φε:ιδίας με: πι δααύ

ο Παύλος, ο Πέτρος, η Παναγία

Το παν ανήκει ατο πι

Το ίδιο και ο Παν, ο φιλόκροτος

Το παρελθόν και το παρόν

είναι κύκλος τέλειος

Το πι απαντά αε: κάθε: κύκλο

και αε: κάθε: γέφυρα

Το πι ατις καμάρες της ποίηαης

Είναι ένα πι καμαρωτό

Γιατί η ποίηαη υφίαταται πρώτα

ως οίηαη του πι

Δημήτρης Πανομήτρος Από τη αuλλογή "Πε:ς μου τ' όνομ6 α ου"

Μaβημaτικοr Διayωνισμοr: 7Ξ-Ξ��aλήςπ��- · 19 Νοεμβρίου 2011

Εu�_�crδης:

Αρχιμ.����: 21 Ιανουαρίου 2012

3 Μαρτίου 201 2

Γράμμα της Σύνταξης Αγαπητοί μαθητές και αuνάδε:λφοι,

Με: αφορμή την έμπνευαη του ποιητή, περιμένουμε αντί­

ατοιχες προαε:γγίαεις Μαθηματικών Θεμάτων μέαα από την

τέχνη, Μας ενδιαφέρει η αυμμετοχή, η διαφορετική άποψη,

η καινοτομία, η εφευρετικότητα, ... να είναι τα ατοιχεία εκ­

κίνηαης της νέας αχολικής χρονιάς. Σχετικά με το αχολικό βιβλίο, κατά τη γνώμη μας, το θέμα

δεν είναι η έλλειψη του, αλλά το ότι δεν επιτρέψαμε τόαα χρόνια τώρα ατους αυναδέλφους να ακολουθούν το Ανα­λυτικό πρόγραμμα ανεξαρτήτως αχολικών βιβλίων.

Να αναπτύααουν δηλαδή, ελεύθερα τις μεθόδους διδα­ακαλίας τους με τη χρήαη οποιουδήποτε βοηθήματος, με ατό χ ο πάντα τη βέλ τιατη προαέγγιαη της κατανόηαης των υπό διαπραγμάτε:υαη εννοιών. Ας ελπίαουμε ότι κάποτε θα φθάαουμε α' αυτό το επίπεδο. Μέχρι τότε ας καταβάλ­λουμε κάθε φιλότιμη προαπάθεια για να βοηθήαουμε τους μαθητές μας, αξιοποιώντας την πείρο ή το μεράκι μας. Θυμίζουμε ότι, βοηθάει οι εργααίες να είναι αε ηλεκτρονι­κή μορφή. "Εχουμε ήδη αρκετές, που θα δημοαιευθούν αε επόμενα τεύχη και περιμένουμε καινούργιες.

Καλή αχολική χρονιά αε όλους Ο πρόεδρος της Σuvτακτικής Εππροπής: Γιώργος Ταααόποuλος

Ο αντιπρόεδροι: Βαγγέλης Ευαταθίου, Γιάννης Κεραααρίδης

ΣύνΘεση εξωφύλλου: Φωτογραφίες από

την 52η ΔιεΘνή ΜαΘηματική Ολυμπιάδα

Θυμίζουμε ότι κάΘε Σάββατο γίνονται ΔΩΡΕΑΝ μαΘήματα, για τους διαγωνισμούς, στα γραφεί

της Ε.Μ.Ε. Έναρξη από Οκτώβριο 2011 ••••••••••••••••••••••••••••••••••••••••••••••••••••••••••••••••••• • ••••••••••

ΕΚΔΟΣΗ ΤΗΣ ΜΑΘΗΜΑτΙΚΗΣ ΗΑΙΡΕΙΑΣ

ΠΑΝΕΠΙΣΤΗΜΙΟΥ 34

106 79 ΑΘΗΝΑ

Trf..: 2103617784-3616532 Fax: 2103641025

Εκτελεστική Γραμματεία

Πρόεδρος: Ταcτcτόπουλος Γιώργος

Εκδότης: Αντιπρόεδροι: ΚcΜοyερόποuλος Γρηγόριος Ευσταθίου Βαγγέλης

Διευθυντής: Κεραcταρίδης Γιδννης

Κρητικός Εμμανουήλ Γραμματέας: Χριστόπουλος Παναγιώτης

Επιμέλεια Έκδοσης: Μέλη: Αργυρδκης Δ. Ζώτος Βαγγέλης Στεφανής π. Κωδικός ΕΛ.ΤΑ.: 2055 ISSN: 1105- 7998

Ταπεινός Ν.

Αθανασόπουλος Γεώργιος Αναστασίου Γιάννης Ανδρουλακάκης Νίκος Αντωνόπουλος Νίκος Αργυράκης Δημήτριος Βακαλόπουλος Κώστας Ευσταθίου ΒαyyεΛης Ζαχαρόπουλος Κων/νος Ζώτος Βαγγέλης Κακκαβάς Απόστολος Καλίκας Σταμάτης Καμπούκος Κυριάκος ΚανεΛλος Χρήστος Καραγκούνης Δημήτρης Καρκάνης Βασίλης Κατσούλης Γιώργος Κερασαρίδης Γιάννης Καρδαμίτσης Σπύρος Κηπουρός Χρήστος Κί\άδη Κατερίνα Κονόμης ί\ρτι

Συντακτική επιτροπή Κόντζιας Νίκος Κοτσιφάκης Γιώργος ΚουτρουμπεΛας Κώστας Κυριαζής Ιωάννης Κυριακόπουλος Αντώνης Κυριακοπούλου Κων/να Κυβερνήτου Χρυστ. Λαζαρίδης Χρήστος Λάππας Λευτέρης Λουρίδας Σωτήρης Μαλαφέκας Θανάσης Μανωλάκου Στοματική Μαυροyιαννάκης Λεωνίδας Μενδρινός Γιάννης Μεταξάς Νικόλαος Μπρίνος Παναγιώτης Μυλωνάς Δημήτρης Μώκος Χρήστος Πανδής Χρήστος Πανουσάκης Νίκος Πουλιάσης Παναγιώτης Ρέγκλης Δημήτρης

Σaί'rη Εύα Σταϊκος Κώστας Στάϊκος Παναγιώτης Στρατής Γιάννης Ταπεινός Νικόλαος Τασσόπουλος Γιώργος Τζιώτζιος Θανάσης Τριαντάφυλλος Γιώργος Τριάντος Γεώργιος Τσαγκάρης Ανδρέας Τσατούρας Ευάγγελος Τσικαλουδάκης Γιώργος Τσιούμας Θανάσης Τυρλής Ιωάννης ΦανεΛη Άννυ Χαρολαμπάκης Ευστάθιος Χαρολαμποπούλου Λίνα Χαρολάμπους Θάνος Χριστιάς Σπύρος Χριστόπουλος Θανάσης Χριστόπουλος Παναγιώτης Ψύχας ΒαyyεΛης

•••••••••••••••••••••••••••••••••••••••••••••••••••••••••••••••••••••••••••••• • Τ α διαφημιζόμενα βιβλία δε σημαίνει ότι προτείνοvrαι από την Ε.Μ.Ε. • Οι συνεργάτες, τα άρθρα, οι προτεινόμενες ασκήσεις, οι λύσεις ασκήσεων κτλ. πρέπει να σrε'λνοvr� σrmραφεία της Ε.Μ.Ε.

με την ένδειξη "Για τον Ευκλείδη 8'", Τα χειρόγραφα δεν εmσrρέφοvrαι. (j!!!jltfέ�_,;_f....:.. Ετήσια συνδρομή (12,00 + 2,00 Ταχυδρομικά = ευρώ 14,00). Ετήσια (nJνδρομή για Σχολεία ευρώ 12,00 Το αvrίτιμο για τα τεύχη που παραyyε'λνοvrαι σrε'λνεται: (1 ). Με απλή ταχυδρομική επιταγή σε διαταγή Ε.Μ.Ε. Τ αχ. Γραφείο Αθήνα

54 Τ.Θ. 30044 (2). Στην ισrοσελίδα της Ε.Μ.Ε., όπου υπάρχει δυνατότητα τραπεζικής συναλλαγής με την τράπεζα EUROBANK (3). Πληρώνεται σrα γραφεία της Ε.Μ.Ε. (4). Με ανnκαταβολή, σε εταιρεία ταχυμεταφορών σrο χώρο σας, κσrά την παραλαβή.

Εκτύπωση: ROTOPRINT IA. ΜΠΡΟΥΣΑΛΗ & Σ/Α EEJ. τηλ.: 210 6623778-358 Υnειίθuνοι; τunογpσφείοu: Δ. Παπαδόπουλος

Page 3: Ευκλειδης Β 81

Τα παιδία παίζει

Τάβλι και llυθαγόρας Παίζετε τάβλι; Εάν επιδίδεστε στο "άθλημα", δοκι­μάστε να παίξετε με έναν διαφορετικό τρόπο. Τοπο­θετήστε 5 πράσινα πούλια ώστε τα κέντρα τους να βρίσκονται σε ευθεία γραμμή yy και τα κέντρα κάθε δύο διαδοχικών να απέχουν την ίδια απόσταση α. Τα κέντρα των ακραίων πράσινων κομματιών α­πέχουν απόσταση 4α. Στη συνέχεια τοποθε­τείστε 3 ισομεγέθη κόκκινα πούλια ώστε τα κέντρα τους να βρί­σκονται πάνω στην ίδια ευθεία χ'χ και τα κέ­ντρα κάθε δύο διαδο­χικών να απέχουν την ίδια απόσταση α, όπως και προηγουμένως. Φροντίστε η ευθεία χ'χ να είναι κάθετη στην ευθεία yy και το

Μια άλλη χρήση για το τάΒλι

χ

αριστερό κόκκινο πούλι να απέχει από το κάτω πράσινο πούλι απόσταση α. Τα κέντρα του αριστε­ρού πράσινου κομματιού και του δεξιού κόκκινου κομματιού απέχουν απόσταση 3α. Προσπαθήστε τώρα να καλύψετε το διάστημα ανάμεσα στο πάνω πράσινο πούλι και στο δεξιό κόκκινο πούλι με άλλα γαλάζια ισομεγέθη πούλια των οποίων τα κέντρα κάθε δύο διαδοχικών να απέχουν απόσταση α. Θα διαπι-στώσετε ότι θα χρει­αστείτε ακριβώς 4 γαλάζια πούλια. Το σημαντικό είναι ότι η απόσταση ανάμεσα στα δύο ακραία πούλια (των κέντρων τούς) είναι 5α. Είναι βέβαιο ότι όλοι θα θυμηθείτε το Πυθαγό­ρειο θεώρημα που μελετή­σατε στο Γυμνάσιο . Βλέ­πετε λοιπόν ότι είναι εύ­κολο να παίξουμε Μαθηματικά με τα πούλια ενός ταβλιού. Το "παιχνίδι" αποτελεί μια εμπειρική α­πόδειξη του θεωρήματος αυτού σύμφωνα με το οποίο το άθροισμα των τετραγώνων των δύο κάθε-

:Μηv εvοχ λεiοτε με τις δυσκολiες σιχς οτιχ :Μαθηματικά. 'Μπορώ vιχ σιχς διιχj3rj3ιχιώσω

ότι οι δικές μου εivιχι ιχκόμιχ μεyιχλύτεpες.

Albert Einstein

Π. Παπαχρήστου

των πλευρών ισούται με το τετράγωνο της υποτεί­νουσας.

Προφανώς μπορείτε να ενεργήσετε ώστε τα πού­λια να εφάπτονται, αλλά τότε χρειάζεται μεγαλύτερη δεξιοτεχνία.

τέτοια παιχνίδια έπαιζαν και οι αρχαίοι Έλληνες, με τη διαφορά ότι αντί για πούλια χρησιμοποιούσαν βότσαλα. Με πολύ απλά μέσα πέτυχαν εξαιρετική πρόοδο στην αριθμητική και τη γεωμετρία. Τα παι­χνίδια με τα βότσαλα αποκάλυψαν και άλλες ιδιότη­τες των ακέραιων αριθμών.

Τριγωνικοί αριθμοί

Έτσι ανακάλυψαν τους λεγόμενους τριγωνικούς α­ριθμούς σχηματίζοντας με βότσαλα ισόπλευρα τρί­γωνα. Αν τοποθετήσουμε στις κορυφές ισόπλευρου τριγώνου τρία κόκκινα βότσαλα έχουμε τον τριγω­νικό αριθμό 3. Αν στο τρίγωνο αυτό προσθέσουμε μια σειρά από 3 γαλάζια βότσαλα παίρνουμε τον α­ριθμό 6. Αν στο νέο τρίγωνο προσθέσουμε μια ακό­μα σειρά από 4 πράσινα βότσαλα παίρνουμε τον α­ριθμό 1 0 . Αν στο νέο τρίγωνο προσθέσουμε μια σει­ρά από 5 πορτοκαλί βότσαλα παίρνουμε τον αριθμό 1 5 και το παιχνίδι συνεχίζεται. Να παρατηρήσετε ότι οι τριγωνικοί αριθμοί αποτελούν αθροίσματα όλων των προηγού-μενων ακεραίων αριθμών, δηλαδή

1 + 2=3, 1 + 2 + 3=6, 1 + 2 + 3 + 4=10, 1 + 2 + 3 + 4 + 5=15 .

Η ιστορία δεν τελειώνει εδώ. Τι θα συμβεί αν, αντί για τρίγωνα, χρησιμοποιήσουμε τετράγωνα και προ­σπαθήσουμε να τοποθετήσουμε βότσαλα; Θα κατα­σκευάσουμε εκείνους τους αριθμούς που ονομάζο­νται τετραγωνικοί αριθμοί. Δείτε την επόμενη εικό­να. Η πρώτη κατασκευή είναι ένα τετράγωνο με 4 κόκκινα βότσαλα στις 4 κορυφές του. Η δεύτερη κατασκευή αφορά ένα τετράγωνο που προήλθε από

ΕΥΚΛΕΙΔΗΣ Β 81 τ.1/1

Page 4: Ευκλειδης Β 81

----------------- Τα παιδία παίζει

το προηγούμενο μετά την πρόσθεση των 5 γαλάζι- Επιστήμη . Έχετε παρατηρήσει ότι πολλά επιστη­ων βότσαλων δεξιά και κάτω. Η τρίτη κατασκευή μονικά βιβλία, ιδιαίτερα αυτά που εξετάζουν την ι­είναι ένα τετράγωνο που προέκυψε από το προη- στορική εξέλιξη αυτού που σήμερα ονομάζουμε επι­γούμενο με την προσθήκη 7 πράσινων βότσαλων στήμη, ξεκινούν με αναφορές στους aρχαίους Έλ­δεξιά και κάτω από το προηγούμενο. Η τέταρτη κα- ληνες και στην προσφορά τους στη θεμελίωση της τασκευή είναι ένα νέο τετράγωνο που προέκυψε από

' θ ' 9 '.β ' το προηγουμενο με την προσ ηκη ;ι.ο;χ':''''·,i{'.ϊ ο-

τσαλων δεξιά και κάτω από το προηγούμενο .

Είναι φανερό ότι σε κάθε κατασκευή φροντίζουμε να διατηρείται η συμμετρία. Είναι εύκολο να διαπι­στώσετε ότι ο αριθμός των βότσαλων σε κάθε "διά­ταξη" αποτελεί το τετράγωνο ακέραιου αριθμού. Πράγματι : 4 = 2 2 , 9 = 3 2 , 16 = 4 2 , 25 = 5 2 Η κατασκευή αυτή κρύβει και ένα βασικό θεώρημα της αριθμητικής. Για να το ανακαλύψουμε έχουμε χρωματίσει κόκκινο βαθύ το πάνω αριστερό βότσα­λο του πρώτου τετραγώνου. Μπορούμε να υποθέ­σουμε ότι το βότσαλο αυτό αποτελεί το αρχικό "μη­δενικό" τετράγωνο . Με την υπόθεση αυτή το τε­τράγωνο με τα 4 κόκκινα βότσαλα μπορεί να θεω­ρηθεί ότι προήλθε από το οριακό τετράγωνο με το ένα βότσαλο στο οποίο προσθέσαμε 3 κόκκινα βό­τσαλα. Τώρα μπορούμε εύκολα να δούμε ότι

1 = 12 ' 1 + 3=4=22 ' 1 + 3 + 5=9=32 ' 1 + 3 + 5 + 7=16=42, 1 + 3 + 5 + 7 + 9 = 25 =52•

( 1 όρο) ( 2 όροι) ( 3 όροι) ( 4 όροι) ( 5 όροι)

Εκείνο που μας λένε οι παραπάνω ισότητες είναι ότι το άθροισμα διαδοχικών περιττών ακεραίων με πρώτο τον αριθμό 1 είναι τέλειο τετράγωνο του α­ριθμού που εκφράζει το πλήθος των περιττών στο άθροισμα. Σπουδαίο συμπέρασμα για να προκύψει από ένα παιχνίδι με βότσαλα. Πέρα από οτιδήποτε άλλο που μπορούμε να πούμε για τους παιχνιδιάρη­δες προγόνους μας, το σίγουρο είναι ότι διέθεταν αστείρευτη φαντασία. Και όπως είπε και ο Einstein,ένας από τους μεγάλους ονειροπόλους της φυσικής,

"η φο:vτcωiα εiτιcα σημο:vτικότερη rοτο τη yvωση ". Αυτά τα παραδείγματα δεν είναι βέβαια τα μο­

ναδικά της συμβολής των αρχαίων Ελλήνων στην αριθμητική και τη γεωμετρία. Αποτελούν ενδείξεις του ανήσυχου και δημιουργικού τους πνεύματος που έβαλε τα θεμέλια σε αυτό που σήμερα ονομάζουμε

επιστήμης; Αλλά η προσφορά των αρχαίων Ελλήνων στα μα­

θηματικά έχει και ένα άλλο κοινωνικό χαρακτηριστι­κό . Στους παλαιότερους πολιτισμούς η μαθηματική γνώση είχε έναν αποκρυφιστικό χαρακτήρα, αποτε­λούσε προνόμιο διάφορων "ιερέων" και ήταν προσι­τή μόνο στα υψηλότερα κλιμάκια του συστήματος εξουσίας. Επιπλέον ήταν δογματική και εξ'αποκαλύψεως. Το φαινόμενο αυτό του aποκρυ­φισμού εκδηλώθηκε ακόμα και στις τάξεις των Πυ­θαγόρειων. Παρά ταύτα οι αρχαίοι Έλληνες μαθημα­τικοί, στην μεγάλη πλειονότητα, ήταν αυτοί που εκ­δημοκράτισαν την γνώση με την εισαγωγή της από­δειξης. Τεκμήριο για την αλήθεια των ισχυρισμών δεν ήταν πλέον η επίκληση θεών και aυθεντιών, αλ­λά η απόδειξη με βάση κανόνες της λογικής. Η γνώ­ση έγινε προσιτή σε όσους επεδίωκαν να την απο­κτήσουν και η γνώση αυτή ήταν έγκυρη μόνον εάν μπορούσε να κατοχυρωθεί με την διαδικασία της α­πόδειξης. Τα λαμπρά αποτελέσματα των αρχαίων Ελλήνων μαθηματικών και φυσικών φιλοσόφων εί­ναι ευρύτερα γνωστά. Το γεγονός ότι σήμερα το μό­νο που μπορούμε ίσως να κάνουμε είναι να τα επι­σημαίνουμε με ένα σύνδρομο υστέρησης αποτελεί μια άλλη ιστορία. Έχει όμως κάποια σημασία να κα­ταγράψουμε μερικές από τις "ανακαλύψεις" τους για να διαπιστώσουμε πόσο προφητικοί υπήρξαν.

Τι συνέβη στην Ιωνία Οι ιστορικοί της επιστήμης θεωρούν ότι η περίοδος περί

το 500 π.Χ. σηματοδοτεί την αφε­τηρία μιας ανατροπής, μιας ανα­τροπής που μερικοί την χαρακτηρί­ζουν ως την ΕV.ηνική Επανάστα­

ση. Είναι η εποχή που οι αρχαίοι φιλόσοφοι "έριξαν το γάντι" στην αρχαία μυθολογία και τις εξηγήσεις της για την δημιουργία και τη λει-τουργία του κόσμου, απαιτώντας

Θαλής

μια λογική εξήγηση στο ερώτημα "από τι είναι κατασκευ­ασμένος ο κόσμος", απορρίπτοντας την άποψη ότι είναι αποτέλεσμα της καλής ή καλής διάθεσης θεών ή δαιμό­νων. Αυτή η επανάσταση στον τρόπο σκέψης οδήγησε στη γέννηση των φυσικών επιστημών. Πολλοί εκτιμούν ότι αυτή η εποχή αρχίζει με τον Θαλή (625-545 π.Χ.) από τη Μίλητο που έζησε περί το 600 π.Χ. Αναζητούσε την ερ­μηνεία των φυσικών φαινόμενων στην ίδια τη φύση προ­σπαθώντας να εξηγήσει την λειτουργία της στη βάση μιας λογικής αρχής. Υποστήριζε ότι πρέπει να υπάρχει μια "πρωτογενής ουσία" από την οποία να παράγονται όλα τα σώματα. Αξιολογώντας την τεράστια σημασία του νερού για την φύση έφτασε στο συμπέρασμα ότι τελικά όλα τα

ΕΥΚΛΕΙΔΗΣ Β 81 τ.l/2

Page 5: Ευκλειδης Β 81

------------------ Τα παιδία παίζει

σώματα αποτελούνται από νερό. Η αντίληψη αυτή προuπαρχε και σε αρχαίους μύθους. Η διαφορά είναι ότι ο Θαλής αναζητούσε λογικές εξηγήσεις πάνω στη γη και όχι στη βούληση των θεών.

Ο Αναξίμανδρος (6 1 1 -546 π.Χ.) είχε αντιρρήσεις. Μετά τον Θαλή πρόβαλλε την άποψη ότι ο κόσμος είναι κατασκευασμένος από μια α­

Αναξίμανδρος

φηρημένη ουσία την οποία ονόμασε άπειρο που σημαίνει αυτό που δεν μπορεί να προσδιοριστεί.

Ο Αναξιμένης (π.585-π.525 π.Χ.) που υπήρξε μαθητής του Αναξίμανδρου ισχυρίστηκε ότι ο κόσμος είναι κατασκευασμένος από αέρα.

Έναν αιώνα αργότερα ο Πυθαγό­

ρας (μεταξύ 580 και 572 π.Χ.­μεταξύ 500 και 490 π.Χ.) υποστήρι­ξε ότι τα πάντα αποτελούνται από αριθμούς. Αυτή την άποψη τη θεμε­λίωσε στα μουσικά διαστήματα που

Αναξιμένης

ανακάλυψε μελετώντας τους ήχους που παράγουν χορδές με διαφορετικά μήκη . Διαπίστωσε ότι οι χορδές παράγουν aρμονικούς ήχους, όταν τα μήκη τους βρίσκονται σε λόγο ακέραιων αριθμών. Κατά τον Πυθαγόρα και την σχολή του οι θεμελιώδεις οντότητες αυτού του κόσμου είναι οι αριθμοί. Η Σχολή του Πυθαγόρα είχε μυστικιστική νοο­τροπία και οργάνωση. Τα μέλη ορκίζονταν να διαφυλά­ξουν τα μυστικά τους και να μην τα ανακοινώνουν στο ευρύτερο κοινό. Κατά κάποια έν­νοια ο τρόπος λειτουργίας τους ερ­χόταν σε αντίφαση με τον δημοκρα­τικό χαρακτήρα των αρχαίων Ελλή­νων και την δημοκρατική φύση των μαθηματικών, από τη στιγμή που έγινε αποδεκτό ότι ο έσχατος κριτής για την αλήθεια των προτάσεων ή­ταν η απόδειξη και όχι ο ισχυρισμός Πυθαγόρας

κάποιας αυθεντίας ή οι διακηρύξεις κάποιου μυστικιστή δάσκαλου. Η διδασκαλία των Πυθαγόρειων είχε κάτι από τη μυθολογική χροιά των ερμηνειών του παρελθόντος. Ισχυρίζονταν ότι το Σύμπαν ξεκίνησε με το Ένα ή Πεπε­

ρασμένο που το θεωρούσαν σαν μια μονάδα αδιαμόρφω­τη και χωρίς έκταση . Η μονάδα περιβαλλόταν από το α­πέραντο που ήταν το βασικό συστατικό του χώρου . Το απέραντο προκάλεσε τον χωρισμό της μονάδας σε ξεχω­ριστούς αριθμούς, οι οποίοι στη συνέχεια αυτοοργανώ­θηκαν και διαμόρφωσαν απλά σχήματα που απετέλεσαν τα τέσσερα βασικά φυσικά στοιχεία : χώμα, αέρας, φω­

τιά, νερό. Θα μπορούσαμε να θεωρήσουμε ότι αυτές οι απόψεις συγκροτούν μια πρώιμη κοσμολογία. Αν αλλά­ξουμε μερικές εκφράσεις και επικαλεστούμε κάποιες α­ναλογίες θα μπορούσαμε ίσως να ισχυριστούμε ότι πε­ριέχει τα σπέρματα μερικών σύγχρονων θεωρητικών α­ντιλήψεων. Οι Πυθαγόρειοι ανακάλυψαν τον λόγο της "χρυσής τομής" μελετώντας το πεντάγωνο και το Πυθα­γόρειο Θεώρημα, το οποίο αποδίδεται, από πολλούς αρ­χαίους ιστορικούς, στον ίδιο τον Πυθαγόρα. Είναι γνωστό

πάντως ότι το χρησιμοποιούσαν και οι Αιγύπτιοι, χωρίς να είναι βέβαιο ότι μπορούσαν να το αποδείξουν. Από δω και πέρα τα πράγματα δυσκόλεψαν. Ίσως κάποιος Πυθαγόρει­ος σκέφτηκε να εφαρμόσει το πυθαγόρειο θεώρημα στο ορθογώνιο ισοσκελές τρίγωνο που αποτελεί το μισό ενός τετράγωνου με πλευρά μιας μονάδας. Το αποτέλεσμα προκάλεσε σοκ διότι οδηγούσε στο συμπέρασμα ότι το μήκος της διαγωνίου δεν μπορούσε να είναι ρητός αριθ­μός, δηλαδή κλάσμα ακεραίων. Η ανακάλυψη ήταν συ­νταρακτική και κλόνιζε την θεωρία τους. Έτσι ορκίστηκαν να την κρατήσουν μυστική . Σύμφωνα με τον μύθο, ένας Πυθαγόρειος, ο Ίππασος από το Μεταπόντιο πρόδωσε το μυστικό με αποτέλεσμα να τον πνίξουν οι υπόλοιποι για την προδοσία του . Το γεγονός είναι ότι οι Έλληνες μαθη­ματικοί βρέθηκαν μπροστά σε αριθμούς των οποίων δεν κατανοούσαν την σημασία και δεν μπορούσαν να τους χειριστούν. Είναι αυτοί ποι σήμερα ονομάζουμε άρρητους

αριθμούς ή ασσύμετρους. Ο αριθμός που συνάντησαν οι Πυθαγόρειοι όταν μελετούσαν το μοναδιαίο τετράγωνο εί-

ναι ο αριθμός που σήμερα θα τον συμβολίζαμε ως J2 .

Σύμφωνα με την μαθηματικοί τους αντίληψη αριθμοί ήταν μόνο οι θετικοί ακέραιοι και λόγοι αυτών και αυτούς μόνο γνώριζαν. Αυτό είχε ως αποτέλεσμα τον διαχωρισμό των μαθηματικών στη γεωμετρία που ήταν η μελέτη σημείων και γραμμών και οι σχετικές τους θέσεις και στην αριθ­

μητική που ήταν η μελέτη των αριθμών. Αυτό το "διαζύ­γιο" έμεινε ενεργό για περίπου 2000 χρόνια μέχρι που ο Ντεκάρτ και ο Φερμά επανένωσαν τα δύο πεδία σε αυτό που σήμερα ονομάζουμε αναλυτική γεωμετρία.

Ο Ηράκλειτος ήταν 40 χρόνια νεότερος του Πυθαγόρα. Τον αποκαλούσαν ο "σκοτεινός φιλόσοφος" διότι μιλούσε με γρί­φους και σκοπίμως παρουσίαζε τις ιδέες του με δυσνόητο τρόπο. Τον ενδιέφερε κυρίως η κίνηση και η αλλαγή και ισχυριζότανε ότι ο κό­σμος είναι μια συνεχής φωτιά. Εί­ναι ο πατέρας της διαλεκτικής λο­γικής που στηρίζεται στην ενότητα των αντιθέσεων. Αυτή ακριβώς η Ηράκλειτος διαλεκτική προσέγγιση των φυσι-κών φαινόμενων αποτέλεσε τη βάση νεότερων φιλοσο­φιών και στο πλαίσιο αυτής της διαλεκτικής βρήκε ερμη­νεία η επιστημονική προσπάθεια για την ενοποίηση των τεσσάρων φυσικών δυνάμεων (βαρυτική δύναμη, ισχυρή αλληλεπίδραση, ασθενής αλληλεπίδραση, ηλεκτρομαγνη­τική δύναμη). Ο Βέρνερ Χάιζενμπεργκ (Verner Heisenberg, 1 90 1 - 1 976), από τους πρωτοπόρους της κβα­ντικής θεωρίας έγραψε στο βιβλίο του Physics and Phi­losophy :

"αν αντικαταστήσουμε την λέξη φωτιά με την λέξη ενέργεια, μπορούμε σχεδόν να αντικαταστήσουμε τις προτάσεις του Ηράκλειτου λέξη προς λέξη .. . Η ενέρ­γεια είναι η "ουσία" από την οποία αποτελούνται όλα τα πράγματα και η ενέργεια είναι η αιτία της κί­νησης".

Ο Παρμενίδης (π. 520 π.Χ.-450 π.Χ.) ήταν σύγχρονος του Ηράκλειτου και εκείνος που δημιούργησε κρίση στην

ΕΥΚΛΕΙΔΗΣ Β 81 τ.l/3

Page 6: Ευκλειδης Β 81

------------------ Τα παιδία παίζει

Ελληνική φιλοσοφία. Τα επιχειρήματά του βασίζονταν στην έννοια της αλλαγής. Ισχυριζότανε ότι η αλλαγή εί­ναι λογικά αδύνατη, εφόσον προϋποθέτει το πέρασμα μέ­σα από μια κατάσταση ανυπαρξίας, μη ύπαρξης. Ισχυρι­ζότανε ότι η αλλαγή είναι μια αυταπάτη . Οι απόψεις του άνοιξαν έναν νέο δρόμο αφηρημένης σκέψης, χωρίς ανα­φορά στον εξωτερικό κόσμο. Έπρεπε να λυθεί το πρό­βλημα της αλλαγής. Οι απόψεις αυτές αποτέλεσαν την αφετηρία δημιουργίας πολλών παράδοξων, όπως τα παράδοξα του Ζήνωνα (π.χ. παράδοξο του δρομέα, παρά­δοξο του βέλους κ.α.)

Η πρώτη λύση ήρθε από τον Εμπεδοκλή (π. 490π.Χ.-430 π.Χ.) . Παρμενίδης Ισχυρίστηκε ότι ο κόσμος είναι κα-τασκευασμένος από 4 βασικές ουσίες τις οποίες αποκά­λεσε αρχικά "ριζικά" και στη συνέχεια "στοιχεία". Τα στοιχεία αυτά ήταν το χώμα, η φωτιά, ο αέρας και το νε­ρό. Αυτά τα στοιχεία συνδυάζονταν και παράγονταν νέες ουσίες. Αυτή η άποψη έδινε μια λύση στο πρόβλημα της αλλαγής διότι πλέον η αλλαγή θεωρείτο μια σύνθεση προς νέες καταστάσεις και όχι με-

ταβολή προϋπάρχουσας κατά­στασης. Ο Εμπεδοκλής ήταν ένας θρησκευόμενος μυστικιστής και θεωρούσε ότι τα στοιχεία ενώνο­νταν και διαχωρίζονταν εξαιτίας ηθικών δυνάμεων αγάπης και δι­απάλης.

Η επόμενη λύση προήλθε από Εμπεδοκλής

τον Αναξαγόρα (περ. 500 π.Χ.-428 π.Χ.) . Ισχυριζότανε ότι όλα τα πράγματα απο-τελούνται από αόρατους "σπό­ρους". Πίστευε σε μια ιδιόμορφη παγκοσμιό-τητα και ι­σχυριζότανε ότι υπάρχει κάτι από τα πάντα, στα πάντα. Τόσο ο Αναξαγόρας, όσο και ο Εμπεδοκλής πίστευαν ότι η ύλη είναι συνεχής και μπορεί να διαιρείται συνεχώς σε μικρότερα κομμάτια.

Ο Δημόκριτος (περ . 460 π.Χ.­περ. 370 π.Χ.) υποστήριζε ότι η ύλη είναι ασυνεχής. Πίστευε ότι μετά από έναν αριθμό διαιρέσεων θα φτάσουμε σε ένα όριο που δεν είναι δυνατόν να διαιρεθεί. Αυτά τα μικρά κομμα-τάκια ύλης που δεν διαιρούνται τα ονόμασε άτομα. Δημόκριτος Αποτελούν τα ελάχιστα δομικά στοιχεία της ύλης. Αυτά τα άτομα έχουν διαφορετική μορφή, σχήμα και βάρος και ενώνονται για να σχηματί­σουν νέες ουσίες καθώς κινούνται στο κενό. Η θεωρία του Δημόκριτου προέβλεπε την ύπαρξη κενού, μέσα στο οποίο κινούνται τα άτομα. Ισχυριζότανε ότι για να κό­ψουμε ένα μήλο με ένα μαχαίρι, πρέπει το μαχαίρι να πε­ράσει αναγκαστικά μέσα από περιοχή που δεν υπάρχει υλικό. Πρέπει να περάσει μέσα από κενό. Ο Αριστοτέλης τον θαύμαζε και εκφραζότανε με κολακευτικά λόγια για αυτόν. Ήταν γνωστός ως 'Ό γελαστός φιλόσοφος" και

έζησε 90 χρόνια (άλλοι ισχυρίζονται 1 1 0). Κληρονόμησε μεγάλη περιουσία την οποία σπατάλησε στα ταξίδια. Θε­ωρείται ο πατέρας της ατομικής θεωρίας της ύλης και εί­ναι εκπληκτικό το γεγονός ότι έφτασε στα συμπερά-σματα αυτά με την δύναμη της σκέψης, συμπεράσματα που επι­βεβαιώθηκαν επιστημονικά μετά από περισσότερο από 2000 χρόνια.

Οι περισσότεροι Έλληνες φιλόσοφοι απέρριπταν την ατομική θεωρία, διότι βασιζόταν στην έννοια του κενού. Προτιμούσαν την θεωρία των τεσσάρων στοιχείων την οποίαν υποστήριζαν ο Πλάτωνας (περ. 424-423 π.Χ.-περ.348-347 π.Χ.)και ο Αριστοτέλης (384π.Χ. -322 π.Χ.) οι γίγαντες της Ελληνικής φιλοσοφίας. Ο Αριστοτέλης εισήγα­γε επίσης και ένα πέμπτο στοιχείο, την πεμπτουσία (πέμπτη ουσία) από την οποία θεωρούσε ότι είναι κατα-σκευασμένα τα ουράνια σώματα. Η Ο Πλάτων ατομική θεωρία αναβίωσε για μικρό χρονικό διάστημα γύρω στον 5° αιώνα από τον Επίκουρο (34 1 π.Χ.-270 π.Χ.) που την ενσωμάτωσε σε μια υλιστική φιλοσοφία που σκόπευε να απομακρύνει τον φόβο των θεών και να προάγει την ευτυχία και την ηρεμία του πνεύ­ματος.

Η ατομική θεωρία είχε το "μειονέκτημα" να μην προϋ­ποθέτει την ύπαρξη Δημιουργού. Επανήλθε στο προσκή­νιο τον 17° αιώνα με την εμφάνιση της σύγχρονης επιστή­μης. Αναθεωρήθηκε από τον Γάλλο καθολικό ιερέα Πιερ Γκασσεντί (Pieπe Gassendi, 1 592-2655) που φρόντισε να εντάξει τον Θεό στις φυσικές διαδικασίες που προέβλεπε η θεωρία, μειώνοντας με αυτό τον τρόπο τον προφανή αθεϊ­στικό χαρακτήρα της.

Είναι ενδιαφέρον να παρατηρή- Ο Αριστοτέλης σουμε ότι πολλά από τα συμπερά-σματα στα οποία κατέληξαν οι φιλόσοφοι βρίσκονται πο­λύ κοντά σε πολλές από τις σύγχρονες επιστημονικές α­πόψεις για την κοσμολογία. Δεν αντέχουμε να μην παρα­τηρήσουμε ότι το Ένα και το Πεπερασμένο του Πυθαγό­ρα μας θυμίζουν την πρωτογενή πυ­ρόσφαιρα της θεωρίας της Μεγάλης Έκρηξης και αντίστοιχα το κενό που γέμισε απο τα στοιχεία του σύμπα­ντος. Ο Δημόκριτος με την ατομική θεωρία του και την aσυνέχεια της ύλης έβαλε τα θεμέλια της σύγχρονης επιστήμης που ο δογματισμός τα κράτησε στο σκοτάδι για 20 περίπου Επίκουρος αιώνες. Βέβαια η σύγχρονη έρευνα απέδειξε ότι τα άτομα δεν είναι άτμητα. Σε ένα βαθύτερο επίπεδο οργάνωσης τα άτομα αποτελούνται από τα πρω­

τόνια και τα νετρόνια και σε ένα ακόμα βαθύτερο επίπεδο τα τελευταία αποτελούνται από τα διάφορα κουάρκς (quarks).

ΕΥΚΛΕΙΔΗΣ Β 81 τ.l/4

Page 7: Ευκλειδης Β 81

----------------- Τα παιδία παίζει ----------------Οι φιλόσοφοι κοιτούν τον ουρανδ Η Ελληνική φιλοσοφία έστρεφε το βλέμμα της και προς του ουρανό. Οι Πυθαγόρειοι δίδα­σκαν ότι η Γη είναι σφαιρική . Φαίνεται ότι κατέληξαν στο συμπέ­ρα-

Οι αρχαίο Έλληνες φι­λόσοφοι

κατέληξαν στο σvμπέ-ρασμα για το

σφαιρικό σχήμα της Γης παρατηρώντας

σμα αυτό μελετώντας τις φάσεις της Σελήνης και την σκιά με μορφή κυκλικού τόξου που σχηματίζει η Γη πάνω στη Σελήνη . Ο Αναξα­γόρας παρατήρησε ότι οι εκλείψεις της Σελή­νης γίνονταν όταν υ­πήρχε πανσέληνος. Δεν χρειαζόταν ιδιαίτερη γεωμετρική διαίσθηση την σκια

Σ

' τηλ� πάνω στη

, ιλ , ε ηνη για να γινει αντ ηπτο ότι όταν υπήρχε πανσέληνος, ο Ήλιος και η σελήνη πρέπει να βρίσκονται στις αντίθετες πλευρές της Γης. Αυτό τον οδήγησε στο συμπέρασμα ότι σε αυ­τή την περίπτωση η Γη θα σχημάτιζε σκιά πάνω στη Σελήνη . Γύρω στο 300 π.Χ. οι περισσότεροι φιλό­σοφοι πίστευαν ότι η Γη είναι σφαιρική και ότι κι­νείται γύρω από τον άξονά της. Μερικοί, όπως ο Α­ρίσταρχος από τη Σάμο πίστευαν ότι το μέγεθος της Σελήνης είναι μικρότερο από το μέγεθος της Γης και ότι το μέγεθος του Ήλιου είναι μεγαλύτερο από το μέγεθος της Γης. Το εκπληκτικό με τον Αρίσταρχο είναι πίστευε πως η Γη κινείται γύρω από τον Ήλιο και η Σελήνη κινείται γύρω από τη Γη . Δυστυχώς δεν μπόρεσε να πείσει του συγχρόνους του, ούτε τους απογόνους τους μέχρι να εμφανιστούν ο Κο­πέρνικος, ο Κέπλερ, ο Γαλιλαίος και η άλλη παρέα.

, , , .,,"'" 80

,

Η έκλειψη Σελήνης παρατηρείται όταν ο Ήλιος, η Γη και η Σελήνη βρεθούν στην ίδια ευθεία με τη Γη να

βρίσκεται ανάμεσα. Για όσο διάστημα η Σελήνη βρίσκεται στον κώνο

σκιάς είναι αόρατη άπό τη Γη.

Ο Ερατοσθένης (276 π.Χ.- 1 94 π.Χ.) περί το 200 π.Χ. υπολ<?γισε με εκπληκτική ακρίβεια την περίμε­τρο της Γης. Ήταν εταίρος της μεγάλης βιβλιοθήκης της Αλεξάνδρειας που την εποχή εκείνη ήταν τόπος έρευνας, αλλά και θεματοφύλακας των γνώσεων ό­λου του κόσμου. Αξίζει να παρακολουθήσουμε τον τρόπο με τον οποίο πέτυχε να "μετρήσει τη Γη".

Ο μύθος λέει ότι ένας ταξιδιώτης που έφτασε στην Αλεξάνδρεια προερχόμενος από το σημερινό Ασουάν, που βρίσκεται νότια της Αλεξάνδρειας, διη­γήθηκε μια παράξενη ιστορία. Στην πόλη αυτή υ­πήρχε μια μέρα τον χρόνο που οι ακτίνες του Ήλιου έπεφταν κατακόρυφα στην επιφάνεια της γης. Αυτό φαινόταν από το γεγονός ότι αυτή την μέρα κατακό­ρυφοι στύλοι στερεωμένοι στο έδαφος δεν έριχναν σκιά. Επιπλέον το φως του Ήλιου έφτανε στους πυθμένες βαθιών πηγαδιών.

,

Σι -----80 --��������������----��

---------

, , , -----------------�-----------

----------------- � -----------

-----------------�-----------Σο

·-------------- """"'--- -- -------.--...------------

Ερατοσθένης

-----------------�----------------------------�-----------

-----------------�---------------------------- � -----------

Ο Ερατοσθένης με αυτήν την θαυμάσια σε απλότητα μέθοδο κατάφερε να υπολογίσει με σημαντική ακρίβεια την ακτίνα της Γης

ΕΥΚΛΕΙΔΗΣ Β 81 τ.l/5

Page 8: Ευκλειδης Β 81

----------------- Τα παιδία παίζει του χρόνου που είναι η σημερινή 22α Ιουνίου και δεν συνέβαινε ποτέ στην Αλεξάνδρεια. Η ιστορία κίνησε το ενδιαφέρον του Ερατοσθένη και αποφά­σισε να μελετήσει το φαινόμενο. Πίστευε ότι η Γη είναι σφαιρική και ο Ήλιος βρίσκεται σε μεγάλη α­πόσταση από τη Γη, πράγμα που σημαίνει ότι οι η­λιακές ακτίνες, όταν φτάνουν στην περιοχή της Γης είναι παράλληλες. Στην προηγούμενη εικόνα φαίνε­ται η εξήγηση που έδωσε ο Ερατοσθένης. Τη συ­γκεκριμένη ημερομηνία οι ακτίνες του Ήλιου πέ­φτουν κάθετα στην επιφάνεια της Γης στο Ασουάν, πράγμα που σημαίνει ότι ο κατακόρυφος στύλος Σο δεν σχηματίζει σκιά. Αυτό όμως δεν συμβαίνει και στην Αλεξάνδρεια εξαιτίας του σφαιρικού σχήματος της Γης. Αυτό σημαίνει ότι ένας κατακόρυφος στύ­λος Σι στην Αλεξάνδρεια πρέπει να ρίχνει σκιά. Εκείνο που έκανε ο Ε­ρατοσθένης ήταν πολύ απλό: μέ­τρησε τη γωνία που σχηματίζει η κατεύθυνση των ηλιακών ακτίνων με τον κατακόρυφο στύλο και την βρήκε ίση με 8°. Δείτε κάτι. Με α­πλή γεωμετρία φαίνεται ότι η γωνία που σχηματίζουν οι επιβατικές α­κτίνες από το κέντρο της Γης με τους τόπους Τι και Τ2 είναι και αυτή 80.

μπόρεσαν να υπολογίσουν την απόσταση ανάμεσα στην Αλεξάνδρεια και το Ασουάν. Οι υπολογισμοί οδήγησαν τον Ερατοσθένη να συμπεράνει ότι η πε­ρίμετρος της Γης είναι περίπου 45 χ 4.800 στάδια, τιμή που σε σημερινές μονάδες αντιστοιχεί περίπου σε 40.000 km , μια τιμή της οποίας η εκατοστιαία έκφραση ως προς την πραγματική τιμή είναι πολύ καλή.

Η παραπάνω ιστορία αναδεικνύει, χωρίς αμφιβο­λία, την περιέργεια, την οξυδέρκεια και την επινοη­τικότητα του Ερατοσθένη. Αναδεικνύει όμως και κάτι άλλο. Και αυτό είναι η σημασία του αναλογικού συλλογισμού για

Αν, λοιπόν γνώριζε και το μήκος του τόξου ΤιΤ2, θα μπορούσε με έ­ναν αναλογικό συλλογισμό να υπο­λογίσει το μήκος ολόκληρης της πε­

Οι φάσεις της Σελήνης. Οι εξασκημένοι Μοvσοv/.μάνοι των οποίων το ημερολόγιο στηρίζεται στον σεληνιακό μιjνα μπορούν να προσδιορίζουν

την ημερομηνία παρατηρcvντας το σχιjμα της Σελήνης

ριφέρειας, διότι γνώριζε ότι τα μήκη των τόξων εί­vαι ανάλογα με τις γωνίες που "βλέπουν" σε αυτά. Έπρεπε λοιπόν να μετρήσει την απόσταση Τι Τ2. Και αυτό ακριβώς έκανε. Πως όμως το έκανε; Σίγουρα δεν τέντωσε ένα νήμα από την Αλεξάνδρεια μέχρι το Ασουάν. Θα μπορούσε ίσως να κάνει το εξής: να πάρει μια άμαξα και να διασχίσει σε ευθεία γραμμή την απόσταση ανάμεσα στις δύο πόλεις. Το πιθανό­τερο είναι ότι μίσθωσε κάποιον άλλο για να κάνει αυτή τη μέτρηση.

Και πως το πέτυχε; Ας ακολουθήσουμε τον μύθο. Αυτοί που ανέλαβαν την μέτρηση δεν είχαν παρά να καταμετρήσουν τον αριθμό των ακέραιων περι­στροφών μια ρόδας της άμαξας.

Ήταν γνωστό ότι σε κάθε περιστροφή της ρόδας καλύπτεται πάνω στο έδαφος απόσταση που είναι ίση με το γινόμενο της διαμέτρου 2R της ρόδας με τον αριθμό π που η τιμή του είναι 3,14. Αν λοιπόν γνώριζαν την διάμετρο της ρόδας μπορούσαν να υ­πολογίσουν το μήκος που διανύει ο άξονας της άμα­ξας στην επιφάνεια της Γης σε κάθε περιστροφή. Με την καταμέτρηση του αριθμού των περιστροφών

την μελέτη της φύσης. Αυτός ο αναλογικός συλλογι­σμός μας δίνει την ευκαιρία να επεκτείνουμε τοπικές κλίμακες σε Συμπαντικές και να καταλήξουμε σε συμπεράσματα για ανάλογα φαινόμενα. Βέβαια οι αναλογικοί συλλογισμοί του Ερατοσθένη είχαν κα­θαρά ποσοτικό περιεχόμενο. Υπάρχουν όμως και περιπτώσεις που ο αναλογικός συλλογισμός μπορεί να αποκτήσει και ποιοτικό περιεχόμενο, aποκαλύ­πτοντας βαθύτερες αναλογίες. Για παράδειγμα, οι ε­πιστήμονες είχαν καταλήξει στο συμπέρασμα ότι το φως έχει δυικό χαρακτήρα. Αυτό σημαίνει ότι άλλοτε συμπεριφέρεται ως κύμα και άλλοτε ως σωμάτιο, ανάλογα με τις συνθήκες. Αυτή η συμπεριφορά έκα­νε τον Λουί ντε Μπρέιγ να αναρωτηθεί αν και τα σωμάτια θα μπορούσαν να συμπεριφερθούν σαν κύ­ματα. Η έρευνα απέδειξε ότι ο δυισμός του φωτός χαρακτηρίζει και τα υλικά σώματα. Επρόκειτο για μια επιτυχία ενός ποιοτικού αναλογικού συλλογι­σμού. Αλλά αυτό αποτελεί μία άλλη ιστορία.

ΕΥΚΛΕΙΔΗΣ Β 81 τ.Ι/6

Page 9: Ευκλειδης Β 81

ο . ""·' •

·,, ;!!

ΧΑΟΣ ΓΑΙΑ Ε: ΡΩΣ

« ... Το ταξίδι για την Ιθάκη

... έχει συνέχεια . . . »

Παναγιώτης Π. Χριστόπουλος Ησίοδος στη Θεογονία αναφέρει: Τα τρία aρχέγονα στοιχεία από τα οποία προέκυψε ο κόσμος είναι το Χάος, η Γαία και ο Έρως, μάλιστα ο Έρως ζωογονεί τα δύο άλλα στοιχεία και παρά­γονται έτσι τα πάντα. Οι θεοί της Ελληνικής Μυθολογίας γεννήθηκαν από αυτή την πρωταρχι­κή τριάδα που προέκυψε από το Κοσμικό Αυγό που άφησε η νύχτα στους κόλπους του Ερέ­βους. Οι Αρχαίοι Έλληνες ονόμασαν το Σύμπαν Κόσμο; Η ονομασία Κόσμος δηλαδή κόσμημα είναι εν­δεικτική της aρχαιοελληνικής αντίληψης για το Σύμπαν. Τον άνθρωπο ανέκαθεν απασχολεί τόσο η δική

του προέλευση όσο και του Σύμπαντος. Υπάρχει αρχή και τέλος των πραγμάτων; Αρχή, βασικό στοι­χείο του Σύμπαντος, ήταν το νερό κατά τον Θαλή τον Μιλήσιο ( φιλόσοφο και θεμελιωτή της Γεωμετρί­ας) . Η φωτιά για τον Ηράκλειτο τον Εφέσιο . Αέρας, γη , νερό, φωτιά τα τέσσερα στοιχεία για τον Εμπε­δοκλή, «στοιχεία αχάλαστα, αρχή και τέλος των πραγμάτων», κατά τον ποιητή. Τι είναι όμως το zcιo.:; Το χάος είναι η κύρια και δημιουργός αρχή κατά τον Ησίοδο. Λένε «η τάξη βγαίνει από την απελ­

πισία του χάους». Η λέξη χρησιμοποιείται με διαφορετικό τρόπο, σε διαφορετικές περιπτώσεις. Άλλη η έννοια του χάους στην θρησκεία, άλλη στην αρχαία ελληνική φιλοσοφία, άλλη η αναπαράστασή του στα διάφορα σύνολα τύπου Mandelbrot. Στην καθημερινή μας ομιλία είναι η τυχαία συμπεριφορά, η έλλειψη ελέγχου, η αταξία, η διάλυση , η σύγχυση , το μπάχαλο, κ.ά. Η έννοια του χάους στην επιστήμη είναι δια­φορετική . Επιστημονικά έχουμε χάος όταν η παραμικρή μεταβολή στην αρχική κατάσταση ενός συστή­ματος επιφέρει σημαντική μελλοντική μεταβολή σε αυτό. Ο όρος χρησιμοποιήθηκε για πρώτη φορά από τον μετεωρολόγο Edward Lorenz κατά τη δεκαετία του 1960. Τα τελευταία χρόνια οι επιστήμονες θέτουν πά­λι όλες τις απόψεις και τις γνώσεις που είχαμε για τον κόσμο στο μικροσκόπιο της θεωρίας του χάους.

Πρώτος οδηγήθηκε σε αυτό ο Poincarέ στη μελέτη του σχετικά με την ευστάθεια του ηλιακού συ­στήματος. Η μελέτη αυτή έγινε με αφορμή ένα διαγωνισμό που προκήρυξε το 1889 ο βασιλιάς της Σουη­δίας και της Νορβηγίας Όσκαρ ο Β ' για την καλύτερη εργασία που αφορούσε στη σταθερότητα του ηλι­ακού μας συστήματος και το πρόβλημα των τριών σωμάτων. Νικητής αναδείχθηκε ο Henri Poincare, κα­θηγητής στο Πανεπιστήμιο του Παρισιού, υποβάλλοντας μια εργασία γεμάτη πρωτοποριακές ιδέες. Στη μελέτη του αυτή ο Poincare, που πήρε και βραβείο Nobel, παρατήρησε ότι είχε οδηγηθεί σε χαοτικό απο­τέλεσμα το οποίο τον βασάνιζε σε όλη του τη ζωή αφού δεν μπόρεσε να συνεχίσει χωρίς Η/Υ(δεν υπήρ­χαν τότε) . «Το πρ6βλημα τω\' τριι;η, σωμάτων» είχε βασανίσει πολλές γενιές μαθηματικών μέχρι που ο Poincarέ διαπίστωσε ότι ήταν ακόμα πιο δύσκολο από ότι πίστευαν. Για να αντιμετωπίσει τη μεγάλη δυ­σκολία του προβλήματος υπέθεσε ότι υπάρχουν μόνο 3 σώματα που κινούνται σε ένα επίπεδο (και όχι στον τρισδιάστατο χώρο). Τα δύο έχουν μεγάλη μάζα και το τρίτο έχει aπειροελάχιστα μικρή μάζα, αμε­λητέα σε σχέση με τα άλλα δύο, όπως για παράδειγμα δύο αστέρια και ένας aστεροειδής. Ο Poincarέ υ­πέθεσε ακόμη ότι οι αρχικές συνθήκες, οι θέσεις δηλαδή και οι ταχύτητες των δύο άστρων ήταν τέτοιες ώστε αυτά να κινούνται με σταθερή ταχύτητα σε ελλείψεις γύρω από το κέντρο της μάζας τους. Επειδή έμοιαζε αδύνατη η επίλυσή του με Άλγεβρα το αντιμετώπισε με Γεωμετρία. Εισήγαγε ένα χώρο πολλών διαστάσεων και έτσι η κατάσταση του συστήματος κάθε στιγμή αναπαρίστατο με ένα σημείο . Σε α�\rο

ΕΥΚΛΕΙΔΗΣ Β 81 τ.l/7

Page 10: Ευκλειδης Β 81

---------------- Χάος - Γαία- Έρως

χώρο των φάσεων, όπως τον ονόμασε, μπορούσε να μετατρέπει αριθμούς σε εικόνες. «Πρέπει με ταπεινοφροσύνη να δεχθούμε ότι ενώ ο αριθμός είναι προϊόν του μυαλού μας ο χώρος είναι

μια πραγματικότητα έξω από το μυαλό μας έτσι ώστε να μη μπορούμε να προδιαγράψουμε με ακρίβεια τις ιδιότητές του». Gauss (1777-1855).

Ο Benoit Mandelbrot Στις 14 Οκτωβρίου 20 10 έφυγε από τη ζωή ο άνθρωπος που δημιούργησε τα fractals. Ο Mandelbrot

είχε παρατηρήσει ότι στις τηλεφωνικές συνδέσεις για την μεταφορά δεδομένων μεταξύ των υπολογιστών υπήρχε θόρυβος. Τότε κατέστρωσε ένα μαθηματικό μοντέλο για να μελετήσει τη δομή των εκρήξεων του θορύβου και μεγεθύνοντας τις εκρήξεις διαπίστωσε την αυτοομοιότητά τους. Αυτό του θύμισε το σύνολο που είχε ανακαλύψει ο Cantor ο οποίος από ένα ευθύγραμμο τμήμα είχε αφαιρέσει το μεσαίο ένα τρίτο τμήμα τού, κατόπιν από κάθε τμήμα που είχε απομείνει είχε αφαιρέσει πάλι το κεντρικό ένα τρίτο και συνέχισε για άπειρα βήματα. Το σύνολο των σημείων που απέμειναν διαπίστωσε ότι είχαν μηδενικό μή­κος (γνωστό ως σύνολο Cantor). [μηδενικό μήκος γιατί αν το αρχικό ευθύγραμμο τμήμα είχε μήκος έστω

1 'λ , , , , ι 2 4 8 ι [ ι J ι Ά 11 ο] α= , ο α τα μερη που αφαιρεσαμε εχουν μηκος

-+- +- +- + .. . =

- --2 = ρα - =

3 9 27 81 3 1--3

Αν αυτό το εφαρμόσουμε σε ένα επίπεδο θα απομείνουν σημεία με μηδενικό εμβαδό (τάπητας Sierpinskί). Εφαρμόζοντάς το σε κύβο θα έχουμε ένα αντικείμενο με μηδενικό όγκο (σπόγγος του Mengker).

--11 11 11 11 11 11 111111 111111

--11 11 1111 1111 11 11 111 11� 111

Σύνολο Cantor Τάπητας Sierpinski Σπόγγος Mandelbrot Ο Mandelbrot σύντομα διαπίστωσε ότι το φαινόμενο αυτό εμφανιζόταν και σε άλλα πεδία, έτσι το

1975 το αποκάλεσε (fractal= σπασμένο) . Εδώ πρέπει να σημειώσουμε ότι και ο Σουηδός Koch το 1904 είχε αφαιρέσει από ένα ισόπλευρο τρίγωνο το μεσαίο ένα τρίτο της κάθε πλευράς και το είχε αντικατα­στήσει με ένα όμοιο με το αρχικό αλλά μικρότερο, έκανε το ίδιο ξανά και ξανά μέχρι που η περίμετρος του να γίνει οριακά άπειρη . Αυτό το αντικείμενο πεπερασμένου εμβαδού με άπειρη περίμετρο ονομά­στηκε νιφάδα χιονιού του Koch.

Αυτό που εντυπωσιάζει στο σύνολο Mandelbrot, πέρα από την πολυπλοκότητά του -που αποκαλύ­πτεται σταδιακά καθώς κάνουμε διαδοχικές μεγεθύνσεις στα διάφορα τμήματά του- είναι ο εξαιρετικά

ΔΟ 00

απλός μαθηματικός τύπος που υπεισέρχεται στην κατασκευή του (είναι της μορφής f(z)=z2+c). Το σύνολο Mandelbrot και η χιονονιφάδα του Koch είναι προϊόντα ανάδρασης.

Γύρω από αυτά τα θέματα έγραψε και ένα βιβλίο με τίτλο «Η μορ­φοκλασματική Γεωμετρία της Φύσης». Δηλαδή κατάλαβε ότι η Ευ­κλείδεια Γεωμετρία δεν επαρκεί για να περιγράψει τις φυσικές δομές οι οποίες είναι πολύ πιο περίπλοκες. Εκ πρώτης όψεως η ακτογραμμή ή η γραμμή της νιφάδας του Koch μοιάζουν μονοδιάστατες αλλά είναι γραμμές «διαταραγμένες» και καταλαμβάνουν επιφάνεια αλλά με διά­σταση μεταξύ 1 και 2, δηλαδή κλασματική. Η κλασματική διάσταση είναι το νέο μαθηματικό εύρημα προκειμένου να χαρακτηρίσει σύνολα

σημείων τα οποία δε μπορούν να γεμίσουν μια επιφάνεια ενώ περισσεύουν όταν τα στοιβάξουμε σε μια ευθεία ή το αντίστοιχο μεταξύ επιφάνειας και τρισδιάστατου χώρου . Έτσι βρέθηκαν οι εξής διαστάσεις: η νιφάδα του Koch έχει διάσταση 1,26 18, ο Τάπητας Sίerpίnski έχει διάσταση 1,8928, ο σπόγγος Mengker έχει διάσταση 2, 727.

Βασικό χαρακτηριστικό της γεωμετρίας των fractals είναι η αυτοομοιότητα υπό κλίμακα, η ιδιότητα δηλαδή ενός σχήματος να περιέχει μέρη που είναι πανομοιότυπα με το σύνολο. Το αγαπημένο παράδειγ­μα αυτοομοιότητας του Mandelbrot ήταν το μπρόκολο, κάθε κομματάκι του οποίου, αν μεγεθυνθεί, έχει

ΕΥΚΛΕΙΔΗΣ Β 81 τ. l/8

Page 11: Ευκλειδης Β 81

---------------- Χάος - Γαία- Έρως

την ίδια μορφή . Η συνεισφορά του Mandelbrot στην επιστήμη δεν περιορίζεται στη μελέτη και γραφική απεικόνιση του ομώνυμου συνόλου μέσω ηλεκτρονικού υπολογιστή αλλά στρέφει το ενδιαφέρον του και στα φαινόμενα που δεν μπορούν να μελετηθούν με τις μέχρι τότε γνώσεις και τα παραδοσιακά μαθηματι­κά, όπως η διακύμανση των τιμών, η μορφή των φυτών, η μορφή των ακτογραμμών, οι μεταβολές του καιρού, η κατανομή των αστεριών ή των γαλαξιών στο σύμπαν κ.ά.

Με την ίδια διαδικασία μπορεί να δημιουργηθεί μια τεράστια ποικιλία εικόνων και τρισδιά­στατων σχημάτων. Μάλιστα, περί τα τέλη της δεκαετίας του 1 980, αποδείχθηκε το λεγόμενο «θεώρημα του κολάζ» που λέει: Κάθε εικόνα μπορεί να προσεγγιστεί σε οποιονδήποτε επιθυμητό βαθμό από ένα πεπερασμένο σύστημα «στοιχειωδών» εικόνων, που είναι προϊόντα ανάδρασης.

Σήμερα η ιδιαίτερα ακριβή και χρονοβόρα διαδικασία αποθήκευσης και μετάδοσης εικόνων έγινε φτηνή και πολύ πιο γρήγορη, με συμπίεση και κωδικοποίηση, που βασίζεται στα fractals .

Έτσι ο Poincare και ο Μ:<:1 μυθικά πρόσωπα για τις θετικές επιστήμες, έδωσαν την έναρξη για το ταξίδι στη νέα επιστήμη της πολυπλοκότητας.

Ο μετεωρολόγος Edward Lorenz το 1963, δημοσίευσε ένα άρθρο με τίτλο «Ντετερμινιστική, μη Πε­ριοδική Ροή», σε ένα μετεωρολογικό περιοδικό, που δε διαβαζόταν από μαθηματικούς ή φυσικούς και έτσι η έρευνα στην επιστήμη του χάους καθυστέρησε περισσότερο από μια δεκαετία, ώσπου να γίνουν

Η εκτύπωση που πήρε ο Lorenz το 196 1

κατανοητές πλήρως οι συνέπειες της ανακά­λυψης του. Ο Lorenz ανακάλυψε την αδυνα­μία για μακροπρόθεσμη πρόβλεψη στα ντε­τερμινιστικά συστήματα διαφορικών εξισώ­σεων που μοντελοποιούσαν την εξέλιξη του καιρού. Είχε δημιουργήσει ένα μοντέλο για να προσομοιώσει την εξέλιξη του καιρού, σε ένα πρωτόγονο ηλεκτρονικό υπολογιστή εκείνης της εποχής. Μετά από αρκετές δοκιμές κατέ­

ληξε σε ένα σύστημα 12 διαφορικών εξισώσεων που εξέφραζαν τις σχέσεις ανάμεσα στις μεταβλητές από τις οποίες εξαρτάται η εξέλιξη του καιρού όπως η θερμοκρασία, η πίεση, η ταχύτητα του ανέμου κ.α. Χρησιμοποιώντας την αριθμητική μέθοδο επίλυσης του συστήματος, κάθε ένα λεπτό εργασίας ο υπολο­γιστής τύπωνε μια σειρά αριθμούς, οι οποίοι αντιστοιχούσαν στην εξέλιξη του καιρού μιας ολόκληρης μέρας. Η εξέλιξη του καιρού, με τον τρόπο αυτό, ήταν ντετερμινιστικά προδιαγεγραμμένη από τις διαφορικές εξισώσεις και την αρχική συνθήκη . Κάποια μέρα ο Lorenz, θέλοντας να εκτυπώσει μια ακολουθία της εξέλιξης του καιρού που να έχει μεγαλύτερο μήκος αντί να αρχίσει από την αρχή , άρχισε από τη μέση των υπολογισμών της προηγούμενης μέρας. Πληκτρολόγησε τους αριθμούς κατευθείαν από την προηγούμενη τυπωμένη σελίδα και ώσπου να τελειώσει ο υπολογιστής τις εκτυπώσεις του πήγε για καφέ. Όταν γύρισε είδε κάτι απρόσμενο. Η καινούργια εκτέλεση, που κανονικά έπρεπε να είναι ίδια με την παλ�ά, διέφερε από κάποιο σημείο και μετά εντελώς, ώστε ο (ψηφιακός) καιρός έπειτα από λίγους μήνες είχε χάσει κάθε ομοιότητα με αυτόν που είχε υπολογίσει ο υπολογιστής την προηγούμενη μέρα.

Η απόκλιση των αποτελεσμάτων οφειλόταν στο γεγονός ότι τα νούμερα που ξανα-εισήγαγε ο Lorenz είχαν μικρότερη ακρίβεια (λιγότερα δεκαδικά ψηφία) από εκείνα που εσωτερικά αποθήκευε ο υπολογι­στής δηλαδή αντί να βάλει τον αριθμό 0.506 127 με έξι δεκαδικά ψηφία, τον οποίο είχε στη μνήμη ο υπο­λογιστής την προηγούμενη μέρα, αυτός πληκτρολόγησε τον αριθμό στρογγυλεμένο 0.506 υποθέτοντας ότι η διαφορά του ενός χιλιοστού δε θα είχε συνέπειες. Το συμπέρασμα ήταν ότι, στο συγκεκριμένο μο­ντέλο, και η ελάχιστη ακόμη έλλειψη ακρίβειας είναι καθοριστική, κάτι που αργότερα αποκαλύφθηκε ότι είναι γενικό χαρακτηριστικό μιας ολόκληρης κλάσης συστημάτων, των λεγόμενων χαοτικών.

Στη συνέχεια ο Lorenz aπλοποίησε το μοντέλο του, το κλα­σικό μοντέλο που συνήθως αποκαλείται σύστημα των εξισώσεων του Lorenz και έγινε το εξής:

dx/dt=IO(y-x), dy/dt=xz+28x-y, dz/dt=xy-(8/3)z Παρατηρούμε ότι αποτελείται από τρεις διαφορικές εξισώ­

σεις που περιέχουν δύο μη γραμμικούς όρους xz και xy και μο­ντελοποιούσαν ρεύματα μεταφοράς θερμότητας μέσα σε ένα ρευστό . Ένα μέρος της τροχιάς της λύσης είναι το σχήμα:

ΕΥΚΛΕΙΔΗΣ Β 81 τ.l/9

Ο Ελκυστής του Lorenz.

Page 12: Ευκλειδης Β 81

---------------- Χάος - Γαία - Έρως ---------------

Στο φαινόμενο αυτό δόθηκε η ονομασία φαινόμενο της πεταλούδας. Η παραπάνω εικόνα έγινε το σύμβολο της επιστήμης του χάους. Είναι η απεικόνιση μιας λύσης του συστήματος των διαφορικών εξι­σώσεων με τις τρείς μεταβλητές για μια δεδομένη αρχική συνθήκη . Κάθε στιγμή οι τρείς μεταβλητές προσδιορίζουν τη θέση ενός σημείου στον τρισδιάστατο χώρο. Καθώς το σύστημα μεταβάλλεται, η κίνη­ση του σημείου παριστάνει την εξέλιξη του συστήματος στο χρόνο. Η τροχιά του σημείου έχει σχεδιαστεί για ένα περιορισμένο χρονικό διάστημα και μοιάζει με πεταλούδα ή σαν ένα είδος διπλής έλικας. Όλες οι τροχιές, μετά από αρκετό χρόνο, συγκεντρώνονται τελικά σε μια περιοχή που αποτελεί τον ελκυστή του συστήματος. Το γεωμετρικό αυτό σύνολο δεν έχει καμιά από τις συνήθεις ιδιότητες, ονομάστηκε παράξενος ελκυστής και είναι μορφοκλασματικό σύνολο ή αλλιώς σύνολο fractals με διάσταση 2,06.

Ένας τρόπος να παρουσιάσουμε οπτικά την χαοτική κίνηση ή οποιαδήποτε άλλη κίνηση , είναι η κα­τασκευή ενός διαγράμματος φάσης της κίνησης. Σε ένα τέτοιο διάγραμμα υπεισέρχεται σιωπηρά ο χρό­νος και σε κάθε άξονα αναπαρίσταται μια μεταβλητή της κατάστασης. Για παράδειγμα, θα μπορούσε κά­ποιος να αναπαραστήσει την θέση ενός εκκρεμούς σε σχέση με την ταχύτητά του . Ένα εκκρεμές σε ακι­νησία θα σχεδιαστεί ως ένα σημείο ενώ σε περιοδική κίνηση θα σχεδιαστεί ως απλή κλειστή καμπύλη που λέγεται τροχιά. Το εκκρεμές μπορεί να παρουσιάσει άπειρες τέτοιες τροχιές. Συχνά τα διαγράμματα φάσης αποκαλύπτουν ότι η πλειοψηφία των τροχιών καταλήγουν να πλησιάζουν ένα κοινό όριο τον r.λ­κυστή του συστήματος. Αντίθετα με το ιδανικό εκκρεμές, τα χαοτικά συστήματα έλκονται προς παράξε­να και πολύπλοκα σχήματα.

Τι ρόλο έπαιξε το χάος στο σχηματισμό του Ηλιακού συστήματος; Μήπως το σύμπαν είναι μορφο­κλασματικό; Ο Charl ier, απέδειξε ότι ο ουρανός τη νύχτα σωστά δεν είναι φωτεινός (παράδοξο του 0\­bers) αφού το συμπάν αποτελείται από σμήνη, σμήνη σμηνών, κλπ και δεν είναι άπειρο. Ο Enon στη με­λέτη για τις τροχιές των αστέρων στα σφαιρωτά σμήνη βρήκε ελκυστή . Επίσης ο Kolmogoroν, ο Amold και Mozer ασχολήθηκαν με τον συντονισμό μεταξύ τροχιακών περιόδων. Δηλαδή αν δυο φεγγάρια είναι σε τροχιά γύρο από έναν πλανήτη και το ένα ολοκληρώνει τρείς φορές την περιφορά του όταν το άλλο κάνει μόνο μια, τότε έχουμε συντονισμό 3: Ι . Πολλές εκδοχές για χάος έχουμε σήμερα από συντονισμούς 2 : 1, 3 : 1, 3 :2 . Όμως οι μικρές διαταραχές που έχουμε σε όλο το Ηλιακό σύστημα απέδειξαν ότι είναι απί­θανο να επιφέρουν χάος στο άμεσο μέλλον. Ένα από τα 8 φεγγάρια του Κρόνου ο Υπερίων διαπιστώθηκε ότι έχει χαοτική τροχιά. Η Γη μάλλον σταθεροποιήθηκε με την δημιουργία της Σελήνης. Με την εμφάνι­ση των Υπολογιστών οι επιστήμονες μελέτησαν αμέσως (πρώτη έρευνα το 1965) την ευστάθεια του Ηλι­ακού συστήματος για χιλιάδες και στη συνέχεια για εκατομμύρια χρόνια. Έγιναν πολλές μελέτες και με τα διαστημικι'i προγράμματα στους πλανήτες, τους aστεροειδείς, τους κομήτες. Το Ηλιακό σύστημα ίσως γίνει χαοτικό σε δισεκατομμύρια χρόνια, το χάος όμως ενδέχεται να έπαιξε σημαντικό ρόλο στο παρελ­θόν κατά την δημιουργία του .

Θα μπορούσαμε να φτάσουμε μέχρι την αρχή του σύμπαντος υπερβαίνοντας το σημείο όπου η γενική σχετικότητα παύει να ισχύει, αν ήμασταν σε θέση να λύσουμε την διαφορική εξίσωση Eίiler και De Vit η επίλυση της οποίας απαιτεί αρχικές συνθήκες, αυτές που θέλουμε να ανακαλύψουμε.

Οι επιστήμονες υποθέτουν ότι σχεδόν αμέσως μετά τη Μεγάλη Έκρηξη που δημιούργησε το σύμπαν θα πρέπει να υπήρξε χάος, μια ελάχιστη αστάθεια στη σχέση ύλης και aντιύλης, αφού διαφορετικά δε θα υπήρχε η ύλη . Η ασυμμετρία αυτή μεταξύ ύλης και aντιύλης θα μελετηθεί σε έναν από τους ανιχνευτές­επιταχυντές του CERN τον LHC, σε μια προσπάθεια να κατανοηθεί γιατί το σύμπαν στο οποίο ζούμε α­ποτελείται από ύλη . Ο επιταχυντής του CERN λειτουργώντας ως χρονομηχανή θα μας πάει πίσω στη στιγμή του Big Bang και θα προσπαθήσει να ρίξει φως στο μυστήριο της "εξαφάνισης" , ερευνώντας εάν και πώς παραβιάστηκε αυτή η ισορροπία ύλης και aντιύλης.

Οι επιστήμονες πιστεύουν ότι μετά τη Μεγάλη Έκρηξη, η οποία συνέβη πριν από 13,7 δισεκατομ­μύρια χρόνια παρήχθησαν σωματίδια ύλης και aντιύλης σε ίσες περίπου ποσότητες. Τούτο σημαίνει πως κάθε σωματίδιο π.χ. πρωτόνιο είχε το αντίστοιχο πανομοιότυπό του σωματίδιο αντιπρωτόνιο, μεταξύ τους διαφέρουν μόνο ως προς το φορτίο που φέρουν (θετικό /αρνητικό). Μόλις 1 Ο μικροδευτερόλεπτα αργότερα, το σύμπαν ψύχθηκε τόσο πολύ, που τα γκλουόνια δέσμευσαν τα κουάρκ, σχηματίζοντας πρω­τόνια και νετρόνια, δηλαδή τους δομικούς λίθους των ατομικών πυρήνων. Από τότε, κανένα κουάρκ δεν παρέμεινε ελεύθερο στο ορατό σύμπαν.

Στόχος της σύγχρονης φυσικής είναι να περιγράψει τις 4 δυνάμεις-αλληλεπιδράσεις με έναν ενιαίο τρόπο, σε μια ενοποiημένη θεωρία που θα τις εξηγεί ως 4 διαφορετικές εκδηλώσεις μίας και μοναδικής

ΕΥΚΛΕΙΔΗΣ Β 81 τ. 1/10

Page 13: Ευκλειδης Β 81

---------------- Χάος - Γαία - Έρως

δύναμης ή που θα το διαψεύδει. Ο "γρίφος" αυτής της θεωρίας βρίσκεται στο μποζόνιο Higgs ή σωματί­διο του Θεού. Η θεωρία λέει, όλα τα σωματίδια της ύλης που γνωρίζουμε αποκτούν μάζα μέσω της αλ­ληλεπίδρασης τους με αυτό . Δηλαδή, όσο περισσότερο αλληλεπιδρά ένα σωματίδιο με το πεδίο Higgs, . τόσο μεγαλύτερη αντίσταση συναντά στην κίνηση του και κατά συνέπεια τόσο βαρύτερο είναι.

Η περίφημη θεωρία του Albert Einstein περιγράφει τη βαρυτική δύναμη και έχει επαληθευθεί πολλές φορές. Το πρόβλημα είναι οι ισχυρές αλληλεπιδράσεις μεταξύ των στοιχειωδών σωματιδίων. Στην περί­πτωση μάλιστα των σωματιδίων - φορέων της βαρυτικής αλληλεπίδρασης οδηγούμαστε μαθηματικά σε μη λογικά αποτελέσματα. Μια θεωρητική λύση στο πρόβλημα είναι να δεχτούμε ότι οι μικρότεροι δομι­κοί λίθοι που συγκροτούν την ύλη δεν είναι σημειακά σωματίδια, αλλά μικρές μονοδιάστατες παλλόμε­νες χορδές. Τότε, αν είναι έτσι, οι βαρυτικές αλληλεπιδράσεις δε λαμβάνουν χώρα σε ένα σημείο, αλλά σε ένα μικρό χώρο μη μηδενικού μήκους, γεγονός που οδηγεί σε μια πιο «λογική» κβαντική συμπεριφο­ρά. Τα στοιχειώδη σωματίδια σε αυτή την περίπτωση δεν είναι τίποτε άλλο από τις «μουσικές νότες» που παράγουν οι παλλόμενες χορδές, ακριβώς όπως οι ήχοι στην κιθάρα παράγονται από την ταλάντωση των χορδών της. Η χορδή μιας διαστάσεως μοιάζει με ένα μικροσκοπικό ανοικτό ή κλειστό (βρόχος) κομμάτι κλωστής. Οι χορδές Ι και 2 διαστάσεων λέγονται μεμβράνες ή βράiιες, (0-βράνες, 3 -βράνες) .

Στο CERN θα ερευνηθεί ένα ακόμη μυστήριο, η φύση της σκοτεινής ενέργειας ή της ενέργειας του κενού η οποία αποτελεί το 73% της συνολικής μάζας του σύμπαντος. Η ενέργεια αυτή είναι η αιτία της επιταχυνόμενης διαστολής του σύμπαντος. Ο Albert Einstein την ονόμασε "κοσμολογική σταθερά" λ.

1-1 πρ{ιβλεψ ι ι κ α ι η μετ6βαση στο χ6.ος Για τους προϊστορικούς ανθρώπους τα φυσικά φαινόμενα και ο κόσμος όλος παρουσιάζονταν σαν χάος.

Με την πάροδο του χρόνου και την ανάπτυξη των επιστημών άρχισε να μπαίνει μια τάξη στο χάος αυτό, κα­θώς όλο και περισσότερα φαινόμενα εξηγούνταν μιας και διατυπώνονταν νόμοι που τα ερμήνευαν. Στα μέσα του 19°" αιώνα πολλοί δεν πίστευαν ότι με τα μαθηματικά μπορούσε κάποιος να προβλέψει τη θέση ενός πλανήτη. Με τα μαθηματικά(διαφορικό λογισμό) που εισήγαγε ο Νεύτωνας στη φυσική και την αστρονομία, δεν άργησε να δημιουργηθεί η εικόνα ενός σύμπαντος που λειτουργεί με απόλυτη τάξη, όπως ο μηχανισμός ενός ρολογιού ακριβείας. Μπορούσαν να μελετηθούν οι κινήσεις των πλανητών αλλά και να προβλεφθεί η θέση τους κάθε χρονική στιγμή και για πολλά χρόνια. Ο Laplace μάλιστα είχε φανταστεί την ύπαρξη μιας διάνοιας(δαίμονας) που να διαθέτει τη γνώση του συνόλου των δυνάμεων που κυβερνούν τη φύση και που με έναν μαθηματικό τύπο δεν θα της ήταν τίποτα αβέβαιο. Το πρώτο πλήγμα σε αυτό ήταν η αρχή της α­προσδwριστίας του Wemer Heisenberg. Μέχρι τότε όμως δεν μπορούσαμε να προβλέψουμε τις κινήσεις των μορίων του αέρα, την τροχιά σε ένα φύλλο χαρτί που πέφτει από ψηλά, την τροχιά ενός φύλλου δένδρου στη ροή του ποταμού, τον καιρό του επόμενου μήνα, κ.ά.

Αφού ο Poincare άνοιξε νέους ορίζοντες, ο Eϋler το 18° αιώνα στην Αγία Πετρούπολη άρχισε να μελετά τον Ήλιο και το Ηλιακό σύστημα. Το έργο του συνέχισε ο Lagrange και ο Laplace μέχρι που στο σύστημα Γη-Σελήνη-Ήλιος βρέθηκε η λύση με τη θεωρία των διαταραχών, καθώς και η ευστάθεια του Ηλιακού συ­στήματος αφού εκφράζεται με τη μορφή μιας συγκλίνουσας σειράς (καλλιτέχνης στις σειρές ήταν ο Ramou­jan). Οι Boltsman και Maxwell βασιζόμενοι στις εργασίες του Bernoulli έδωσαν την κινητική θεωρία των αερίων, την θεωρία των ατόμων και απέδειξαν ότι η εντροπία(=ο λόγος της θερμότητας που περιέχεται σε ένα σύστημα προς τη θερμοκρασία του) αποτελούσε μέτρο της αταξίας του συστήματος.

Στην προσπάθεια να μελετηθεί η κίνηση των μορίων των αερίων επινοήθηκε η στατιστική μηχανι­κή, όπως και στην προσπάθεια να μελετηθεί η τροχιά ενός ηλεκτρονίου επινοήθηκε η κβαντική μηχανι­κή . Η θεωρία του χάους(διερευνά την έννοια της προβλεπτικότητας), η κβαντική θεωρία(διερευνά την αρχή της αιτιότητας) και η θεωρία της σχετικότητας( βρήκε τη σχέση του χώρου και του χρόνου) είναι οι σημαντικότερες ανακαλύψεις του 20°" αιώνα, για τον 2 1 ο ακόμη είναι η θεωρία του χάους.

Με τη θεωρία του χάους και τη κβαντική θεωρία του χάους, οι μαθηματικοί ερεύνησαν και τη συ­μπεριφορά ορισμένων μη γραμμικών δυναμικών συστημάτων της καθημερινής ζωής όπως: οι μεταβολές του καιρού, η διακύμανση σε πληθυσμούς αγρίων ζώων, η εξέλιξη των τιμών στο χρηματιστήριο, η βρύση που στάζει, η τυρβώδης ροή του νερού στο ποτάμι( Ladaou, Kolmogoroν, Ruelle, Takens), το με­λάνι που διαχέεται μέσα σε ένα ποτήρι νερού, οι σεισμοί, η πολυμερής δομή DNA-RNA στα κύτταρα, θέματα αστρονομίας και πολλά άλλα. Πολλοί επιστήμονες τα τελευταία χρόνια αναπαριστούν τα ανεξέ­λεγκτα αυτά φαινόμενα με μη-γραμμικές εξισώσεις σε Η/Υ για να ανακαλύψουν παράξενους ελκυστές και την κρυφή τάξη που τα ορίζει. Ποιο είναι όμως το μέλλον της θεωρίας αυτής; Πώς θα εξελιχθεί τα επόμενα χρόνια; Ίσως, ακόμη, αναρωτηθούμε τι βρίσκεται πέρα από το χάος. Κάποιοι απαντούν αντι­χάος ή πολυπλοκότητα. Δηλαδή το χάος μάς δείχνει ότι, ορισμένες φορές, απλά συστήματα παρουσιά­ζουν εξαιρετικά αποδιοργανωμένη συμΠεριφορά ενώ η πολυπλοκότητα μάς δείχνει ότι η πολύπλοκη συ-

ΕΥΚΛΕΙΔΗΣ Β 81 τ.1/1 1

Page 14: Ευκλειδης Β 81

---------------- Χάος - Γαία - Έρως

μπεριφορά οδηγεί κάποιες φορές σε οργανωμένη συμπεριφορά. Η επιστήμη του Χάους και της Πολυ­πλοκότητας αναπτύσσεται ραγδαία τις τελευταίες δεκαετίες τόσο σε θεωρητικό όσο και σε πρακτικό επί­πεδο. Μεγάλη συμβολή για την ανάπτυξη της νέας αυτής επιστήμης προήλθε από τους ηλεκτρονικούς υπολογιστές αφού οι μη γραμμικές εξισώσεις μόνο με ισχυρούς υπολογιστές μπορούν να λυθούν.

Στον 2 Ι ο αιώνα, από τις θεωρίες της βαρύτητας, του ηλεκτρομαγνητισμού, της σχετικότητας, της κβαντομηχανικής, των κυμάτων, των ατομικών και υποατομικών σωματίων, φτάσαμε να μιλάμε για την Περί Παντός Θεωρία ή Μ-Θεωρία των Edward Witten και Paui Townsend. Σύμφωνα με τη Μ­Θεωρία οι 6 θεωρίες των χορδών-υπερχορδών είναι μέρος της ίδιας θεωρίας που αντιστοιχούν σε διαφορετικές απόψεις. Η ουσία της θεωρίας αυτής είναι ότι οι οικοδομικοί λίθοι του σύμπαντος δεν εί­ναι τα άτομα, τα μόρια ή τα στοιχειώδη σωμάτια, αλλά μονοδιάστατες απείρως μικρές, παλλόμενες χορδές (χορδία) μήκους J(J33cm (το μήκος Planck) όπου το σύμπαν είναι Ι Ι διαστάσεων 1 0 για το χώρο και Ι για το χρόνο . Οι δονήσεις των χορδών είναι που διακρίνουν τη μια από την άλλη . Κάθε δια­κεκριμένη δόνηση καθορίζει το κατά πόσο πρόκειται για ύλη ( φερμιόνια) ή ενέργεια (μποζόνια), που εί­ναι τα συστατικά του σύμπαντος. Η μαθηματική ομορφιά των θεωριών αυτών της υπερσυμμετρίας έ­γκειται στο ότι δεν εμφανίζονται μαθηματικές aσυνέπειες. Η υπερσυμμετρία μετασχηματίζει τις συ­ντεταγμένες του χώρου και του χρόνου έτσι ώστε οι νόμοι της φυσικής να είναι ίδιοι για όλους τους παρατηρητές. Επιπλέον, οι θεωρίες υπερσυμμετρίας κατάφεραν να συμβιβάσουν τις προσπάθειες ενοποίησης της θεωρίας του Κβαντικού Πεδίου και της θεωρίας της Σχετικότητας.

Σύμφωνα με την Μ-θεωρία, τη' στιγμή του Big Bang οι Ι Ι διαστάσεις ήταν ίσες και ενσωματωμένες σε μια απειροστού μεγέθους φυσαλίδα ή σημείο . Μετά το Big Bang, το Σύμπαν διασπάστηκε σε μια 4-διάστατη φυσαλίδα, που άρχισε να διαστέλλεται, ενώ το υπολειπόμενο μέρος του «συμπτύχθηκε» σε 7 διαστάσεις. Δηλαδή το σύμπαν στο οποίο ζούμε και η ύλη που περικλείει είναι «παγιδευμένα» σε μια ε­πιφάνεια τριών διαστάσεων χώρου, γνωστή ως βράνη, και μόνο τα γκραβιτόνια, τα σωματίδια-φορείς της βαρυτικής αλληλεπίδρασης, μπορούν να απομακρύνονται από αυτήν και να κινούνται στις υπόλοιπες δι­αστάσεις. Πολλά πράγματα μοιάζουν να βρίσκονται πολύ πιο πέρα από τα μαθηματικά μοντέλα μας, άλ­λωστε στα μαθηματικά για τρεις περίπου αιώνες δεν είχε προκύψει η ανάγκη αντιμετώπισης τέτοιων θε­μάτων. Πάντως τα νέα μαθηματικά μας επιτρέπουν να μιλάμε για πολυδιάστατα σύμπαντα. Η «Μ­Θεωρία» προβλέπει ότι πάρα πολλά σύμπαντα δημιουργήθηκαν εκ του μηδενός και υπάρχουν παράλλη­λα με το δικό μας. Ο S. Hawking στο τελευταίο του βιβλίο υποστηρίζει ότι δεν υπάρχει ενιαία «θεωρία του παντός». Η προτίμηση του Hawking είναι η λεγόμενη «Μ-θεωρία» των χορδών, η οποία πιστεύει ότι αρκεί για να εξηγήσει τα κεντρικά μυστήρια της φύσης.

Στο μοντέλο που λέγεται τοπολογία Picard, το σύμπαν καμπυλώνεται με έναν παράξενο τρόπο. Ο Frank Steiner υποστηρίζει ότι το σύμπαν διαστέλλεται προς ένα σχήμα που μοιάζει με μια μακριά χοάνη, με έναν στενό λαιμό στο ένα άκρο και πιο ανοικτό το άλλο άκρο. Αυτό το σχήμα δείχνει ότι το διάστημα είναι πεπερασμένο και εξελίσσεται με έναν ασυνήθιστο τρόπο. Αυτά τα μοντέλα του σύμπαντος - σαν χοάνη - προτάθηκαν στη δεκαετία του '90 για να ταιριάζουν με μια παρόμοια ανωμαλία που φάνηκε από το δορυφόρο CQBE. Ό όγκος του σύμπαντος με τα μοντέλα αυτά εκτιμάται σε Ι 032 κυβικά έτη φωτός.

Σήμερα που έχει ξεσπάσει ο πόλεμος της οικονομικής κρίσης το ερώτημα είναι: Υπάρχει ελκυστής στην οικονομία; Αν υπάρχει και βρεθεί, χαράς ευαγγέλια, γιατί θα εφαρμοστεί από τους οικονομολόγους και θα αποφύγαμε τον πόλεμο από αυτούς που τη χρονική αυτή στιγμή κερδίζουν και μας λοιδορούν.

Ο John Nash το Ι 95 Ι (βραβείο Nobel το Ι 994) απέδειξε ότι υπάρχει σημείο ισορροπίας (ελκυστής) στην οικονομία. Όμως πώς και που το βρίσκουμε αυτό το σημείο ισορροπίας; Ο Κωνσταντίνος Δασκα­λάκης, απαντώντας στον γρίφο του Nash πριν δύο χρόνια, δυστυχώς για μας, απέδειξε ότι δεν υπάρχει τρόπος να προσδιοριστεί το σημείο ισορροπίας. Άρα για μια ακόμη φορά οι μαθηματικοί βρίσκονται αντιμέτωποι με την πρόκληση . Το ότι είναι αδύνατος υπολογιστικά ο προσδιορισμός του σημείου ισορ­ροπίας δεν σημαίνει ότι θα είναι για πάντα αδύνατος. Σημαίνει πως τα υπολογιστικά μαθηματικά εργα­λεία που διαθέτουμε σήμερα δεν επαρκούν για την επίλυσή του . Σκεφτείτε ότι κάποτε ήταν αδύνατα ή άλυτα και άλλα προβλήματα όπως ο τετραγωνισμός του κύκλου ή ο διπλασιασμός του κύβου, κ.ά.

Το Μαντείο των Δελφών, έχει να δώσει το χρησμό : «Το ταξίδι για την Ιθάκη συνεχίζεταυ>. Σίγου­ρα εκεί κοντά στην Ιθάκη, στο φουρτουνιασμένο Ιόνιο, θα βρούμε σημεία γαλήνης και ισορροπίας.

[ 1 ] Χάος και Αστρονομία Baπy Parker [3] Χάος και αρμονία Trinb Huan Thuan

[2] Τύχη και Χάος Daνid Ruelle [4] Οι νόμοι του Χάους Ilya Prigogine

ΕΥΚΛΕΙΔΗΣ Β 81 τ. 1/12

Page 15: Ευκλειδης Β 81

Υ . ! l i / . , ./ /

' .� ( ,-' . )

Επιτροπή Διαγωνισμών της Ε.Μ.Ε.

00 Ολλανδία, Άμστερνταμ, 12-24 Ιουλίου 2011

Η 52η Διεθνής Μαθηματική Ολυμπιάδα διεξήχθη στο Άμστερνταμ της Ολλανδίας, από 12 έως και 24 Ιουλίου 201 1 . Συμμετείχαν 564 μαθητές από 101 χώρες. Την Ελληνική ομάδα αποτελού­σαν οι μαθητές :

Γεώργιος Βλάχος Χρυσό μετάλλιο Γεώργιος Καλαντζής Χάλκινο μετάλλιο Αλέξανδρος Μουσάτοβ Χάλκινο μετάλλιο Παναγιώτης Λώλας Χάλκινο μετάλλιο Σπυρίδων Κακαρούμπας Εύφημη μνεία Αθανασίου Νικόλαος Συμμετοχή

Σημειώνουμε ότι το , που κατέκτησε ο μαθητής Γεώργιος Βλάχος, είναι το στην ιστορία των 33 μέχρι τώρα συμμετοχών της Ελληνικής ομάδας στις Διεθνείς Μα­

θηματικές Ολυμπιάδες. Στην ανεπίσημη βαθμολογία μεταξύ κρατών στις πρώτες θέσεις ήταν οι ομάδες της Κίνας (βαθμοί 189), των Ηνωμένων Πολιτειών Αμερικής (βαθμοί 184), της Σιγκα­πούρης (βαθμοί 1 79) . Η Ελληνική ομάδα κατέλαβε την 40η θέση με 99 βαθμούς.

Αρχηγός της Ελληνικής ομάδας ήταν ο Αναπληρωτής Καθηγητής του Ε. Μ. Πολυτεχνείου Ανάργυρος Φελλούρης και υπαρχηγός ο Μαθηματικός Ευάγγελος Ζώτος.

Για κάθε δεδομένο σύνολο Α = {a 1 , a2 , a3 ' a4 } με στοιχεία τέσσερις διαφορετικούς θετι­

κούς ακέραιους, συμβολίζουμε με s Α το άθροισμα a1 + a2 + a3 + a4 • Έστω η Α είναι ο αριθ­

μός των ζευγαριών ( i , j) με 1 � ί < j � 4 για τα οποία ο αριθμός a; + aj διαιρεί τον s A . Βρεί­τε όλα τα σύνολα Α με στοιχεία τέσσερις διαφορετικούς θετικούς ακέραιους, για τα οποία επιτυγχάνεται η μεγαλύτερη δυνατή τιμή για το η Α •

Κατ' αρχή θα αποδείξουμε ότι η μέγιστη τιμή του n Α είναι το πολύ 4. Χωρίς βλάβη της γενι­κότητας μπορούμε να υποθέσουμε ότι α1 < α2 < α3 < α4 • Παρατηρούμε ότι για κάθε ζευγάρι δει­κτών ( i, j ) το άθροισμα αί + α1 διαιρεί τον sA , αν, και μόνον αν, ο αί + α1 διαιρεί τον

s, - (α, + α 1 ) = α, + α1 , όπου k και Ι είναι οι άλλοι δύο δείκτες. Επειδή υπάρχουν (;) = 6 δια-

ΕΥΚΛΕΙΔΗΣ Β ' 81 τ.Ι/13

Page 16: Ευκλειδης Β 81

--------- Μαθηματικοί Διαγωνισμοί - Μαθηματικές Ολυμπιάδες -------­

φορετικά ζευγάρια δεικτών, αρκεί να αποδείξουμε ότι δύο τουλάχιστον από αυτά δεν ικανοποι­ούν την προηγούμενη συνθήκη .

Πράγματι, δύο τέτοια ζευγάρια είναι τα ( α2 , α4 ) και ( α3 , α4 ) , αφού ισχύει ότι α2 + α4 > a1 + α3

και α3 + α4 > α1 + α2 , οπότε οι α2 + a4 , α3 + α4 δεν διαιρούν το s Α • Άρα έχουμε n Α � 4 . Υ ποθέ­τουμε ότι n Α = 4 . Σύμφωνα με τα προηγούμενα έχουμε :

αι + α4 iα2 + α3 και α2 + α3 iαι + α4 '

α1 + α2 iα3 + α4 και α3 + a4 j α1 + α4 ,

α1 + α3 iα2 + α4 και α2 + a4 j α1 + α3 .

Επομένως, υπάρχουν θετικοί ακέραιοι u, ν με ιι > ν 2:: 2 τέτοιοι ώστε: α1 + α4 = α2 + α3 (1), u (a1 + a4 ) = a3 + a4 (2) , ν (α1 + α3 ) = α2 + α4 • (3 )

Με πρόσθεση κατά μέλη των εξισώσεων ( 1 ) και (3) λαμβάνουμε: ν (α1 + a3 ) = 2α2 + α3 - a1 • (4)

Αν ν 2:: 3, τότε ν ( α1 + a3 ) > 3α3 > 2α2 + α3 > 2α2 + α3 - α1 , που είναι άτοπο. Άρα είναι ν = 2 , οπότε από την (4) λαμβάνουμε την ισότητα: 3a1 + a3 = 2a2 • (5) Με πρόσθεση κατά μέλη των ( 1 ) και (2) λαμβάνουμε: ( u + 1 ) a1 + ( u - 1 ) a2 = 2α3 • (6) Από την (5) έχουμε a3 = 3a1 - 2a2 , το οποίο με αντικατάσταση στην (6) δίνει

(ιι + 7 ) a1 = ( 5 - ιι ) a2 • (7) . Επειδή είναι ( u + 7 ) a1 > Ο και α2 > Ο , έπεται ότι 5 - u 2:: 1 , δηλαδή u � 4 . Επειδή είναι u > ν 2:: 2 , έχουμε ότι u = 3 ή u = 4 .

Για ( ιι , ν) = ( 3, 2 ) , από το σύστημα των ( 1 )-(3) λαμβάνουμε τα σύνολα {d, 5d, 7d, 1 1d} , όπου d wχαίος θετικός ακέραιος. Για (u, ν) = (4, 2 ) , από το σύστημα των ( 1 )-(3) λαμβάνουμε τα σύνολα {d, 1 1d, 1 9d, 29d} , όπου d τυχαίος θετικός ακέραιος.

Έστω S ένα πεπερασμένο σύνολο σημείων του επιπέδου με δύο τουλάχιστον σημεία. Υ­ποθέτουμε ότι δεν υπάρχουν τρία σημεία του S που να είναι συνευθειακά. Ένας ανεμόμυλος είναι μία διαδικασία η οποία αρχίζει με μία ευθεία [ που περνάει από ένα μόνο σημείο Ρ ε S . Η ευθεία περιστρέφεται κατά τη φορά των δεικτών του ωρολογίου με κέντρο το σημείο Ρ μέχρι που να συναντήσει για πρώτη φορά κάποιο άλλο σημείο που ανήκει στο σύ­νολο S. Αυτό το σημείο, έστω Q , γίνεται τώρα νέο κέντρο περιστροφής και η ευθεία περι­στρέφεται κατά τη φορά των δεικτών του ωρολογίου γύρω από το Q , μέχρι που να συνα­ντήσει ένα σημείο του S. Η διαδικασία αυτή συνεχίζεται απεριόριστα με κέντρο περιστρο­φής πάντοτε ένα σημείο του S. Να αποδείξετε ότι μπορούμε να επιλέξουμε ένα σημείο Ρ του συνόλου S και μία ευθεία { που περνάει από το Ρ, έτσι ώστε ο παραγόμενος ανεμόμυλος

να χρησιμοποιεί κάθε σημείο του S ως κέντρο περιστροφής άπειρες φορές.

Καθορίζουμε τα δύο ημιεπίπεδα που ορίζει η περιστρεφόμενη ευθεία ως πορτοκαλί ημιεπίπεδο και μπλε ημιεπίπεδο. Σημειώνουμε ότι οποτεδήποτε το κέντρο περιστροφής αλλάζει από κάποιο σημείο τ σε κάποιο άλλο σημείο υ ' μετά την αλλαγή το σημείο τ βρίσκεται στο ημιεπίπεδο που ήταν προηγουμένως το σημείο υ . Επομένως, ο αριθμός των σημείων του S στο πορτοκαλί ημιεπίπεδο και ο αριθμός των στοιχείων του S στο μπλε ημιεπίπεδο παραμένει ο ίδιος σε όλη τη διάρκεια της διαδικασίας, εκτός από τις στιγμές που η ευθεία περιέχει δύο σημεία.

Θεωρούμε πρώτα την περίπτωση με IS I = 2n + 1 . Ισχυριζόμαστε ότι από κάθε σημείο Τ του

ΕΥΚΛΕΙΔΗΣ Β' 81 τ. 1 / 14

Page 17: Ευκλειδης Β 81

--------- Μαθηματικοί Διαγωνισμοί - Μαθηματικές Ολυμπιάδες -------­

S περνάει ευθεία η οποία σε καθένα από τα δύο ημιεπίπεδα που ορίζει περιέχει ακριβώς n ση­μεία του S . Πράγματι, αν θεωρήσουμε μία ευθεία που περνάει μόνο από το σημείο Τ του συ­νόλου S και υποθέσουμε ότι περιέχει n + r σημεία του συνόλου S στο πορτοκαλί ημιεπίπεδο της, με r > Ο , τότε, όταν η ευθεία περιστρέφεται γύρω από το σημείο Τ , ο αριθμός των σημείων του συνόλου S που βρίσκονται στο πορτοκαλί ημιεπίπεδο της μεταβάλλεται κατά 1 κάθε φορά που η ευθεία μόλις έχει περάσει από ένα σημείο του S .

υ Ί •'

τ • τ

Σχήμα 1 \ Επομένως μετά από περιστροφή 1 80° , ο αριθμός των σημείων του S που βρίσκονται στο πορ­

τοκαλί ημιεπίπεδο θα είναι n - r . Επομένως, θα υπάρχει μία ενδιάμεση θέση στην οποία το πορ­τοκαλί ημιεπίπεδο, άρα και το μπλε, θα περιέχει n σημεία του συνόλου S .

Στη συνέχεια επιλέγουμε τυχαίο σημείο Ρ του συνόλου S και ευθεία ε που περιέχει n ση­μεία του συνόλου S σε κάθε ημιεπίπεδο της, ως αρχική κατάσταση του ανεμόμυλου . Θα αποδεί­ξουμε ότι κατά τη διάρκεια περιστροφής κατά 1 80° , η ευθεία ε θα χρησιμοποιήσει κάθε σημείο του συνόλου S ως κέντρο περιστροφής. Για την απόδειξη αυτού, επιλέγουμε οποιοδήποτε ση­μείο Τ του συνόλου S και ευθεία f που περνάει από το Τ και διαχωρίζει το σύνολο S σε δύο υποσύνολα με n σημεία. Οποιαδήποτε ευθεία παράλληλη προς την ευθεία f θα έχει στα δύο ημιεπίπεδά της άνισο αριθμό σημείων του συνόλου S . Επομένως, όταν η ευθεία ε γίνει παράλ­ληλη προς την ευθεία f , τότε θα πρέπει ε = f και Τ Ε ε .

Θεωρούμε τώρα την περίπτωση με IS I = 2n . Ανάλογα με την προηγούμενη περίπτωση, για κάθε σημείο Τ Ε S υπάρχει ευθεία που περνάει από το Τ που περιέχει n - 1 σημεία του συνό­λου S στο πορτοκαλί ημιεπίπεδο αυτής και n σημεία του συνόλου S στο μπλε ημιεπίπεδο. Ε­πιλέγουμε μία τέτοια προσανατολισμένη ευθεία ε που περνάει από τυχόν σημείο Ρ του συνό­λου S , ως αρχική κατάσταση του ανεμόμυλου. Θα αποδείξουμε ότι κατά τη διάρκεια περιστρο­φής κατά 360° , η ευθεία του ανεμόμυλου χρησιμοποιεί κάθε σημείο του συνόλου S ως κέντρο περιστροφής. Για την απόδειξη αυτού, επιλέγουμε οποιοδήποτε σημείο Τ του συνόλου S και ευθεία f που περνάει από το Τ και διαχωρίζει το σύνολο S σε δύο υποσύνολα με n - 1 σημεία στο πορτοκαλί ημιεπίπεδο και n σημεία στο μπλε ημιεπίπεδο. Οποιαδήποτε ευθεία παράλληλη προς την ευθεία f θα έχει στα δύο ημιεπίπεδά της διαφορετικό αριθμό σημείων του συνόλου S . Επομένως, όταν η ευθεία ε γίνει παράλληλη προς την ευθεία f και με τον ίδιο προσανατολι­σμό, τότε θα πρέπει ε =: f και Τ Ε ε .

Για την τυχούσα ευθεία f που περιέχει το σημείο L Ε S ορίζουμε τη μία από τις δύο ημιευθεί­ες με αρχή το σημείο L ως θετική και την άλλη ως αρνητική . Όταν η ευθεία αλλάζει κέντρο πε­ριστροφής, τότε θετική ημιευθεία είναι αυτή που περιέχεται ή περιέχει την προηγούμενη θετική ημιευθεία.

Σχήμα 2 Δ ι. Α +

ΕΥΚΛΕΙΔΗΣ Β ' 81 τ. l/15

Page 18: Ευκλειδης Β 81

--------- Μαθηματικοί Διαγωνισμοί - Μαθηματικές Ολυμπιάδες -------­

Επίσης ορίζουμε τα δύο ημιεπίπεδα που ορίζει η ευθεία f ως αριστερό και δεξιό και συμβολί­ζουμε με D ( f ) τη διαφορά Δ - Α , όπου Δ (αντίστοιχα, Α ) είναι ο αριθμός των σημείων του συνόλου S που περιέχονται στο ανοικτό ημιεπίπεδο Δ (αντίστοιχα, Α).

Σχήμα 3

Δ Υποθέτουμε ότι η ευθεία f που περιέχει το σημείο Ακ Ε S περιστρέφεται κατά την αρνητική

φορά (ίδια με αυτή της κίνησης των δεικτών του ωρολογίου) και έστω ότι πρώτη η θετική η μι­ευθεία συναντά κάποιο σημείο Ακ+ ι του S . Όταν η ευθεία f συνεχίσει την περιστροφή της, τότε το σημείο Ακ δεν ανήκει πλέον στην ευθεία f , αλλά στο ημιεπίπεδο που ανήκε το σημείο Ακ+ι πριν το συναντήσει η ευθεία f . Έτσι άμεσα προκύπτει ότι η διαφορά D ( f ) = Δ - Α παραμένει σταθερή κατά την περιστροφή της ευθείας f . Θα συνεχίσουμε με δύο βοηθητικές προτάσεις;

Λ�� μμα Ι ., Δεν υπάρχουν δύο σημεία Α1 , Α2 του S από τα οποία περνούν δύο παράλληλες

και ομόρροπες ευθείες ε1 και ε2 , αντίστοιχα, έτσι ώστε D( ε1 ) = D( ε2 ) . + +

Α Δ Α n σημι:ία Δ

ι.· σημεία m σημι:ίο.

Α1 Az

Σχήμα 4

Απόδειξη . Διαφορετικά, αν υποθέσουμε ότι στο αριστερό ημιεπίπεδο της ε1 περιέχονται n σημεία, στο δεξιό ημιεπίπεδο της ε2 περιέχονται m σημεία, ενώ στη ζώνη των δύο παραλλήλων ευθειών περιέχονται κ σημεία, τότε θα έχουμε:

D( ε1 ) = D( ε2 ) => ( κ+ m + 1 ) - n = m - ( κ + n + 1 ) => κ = -1 , άτοπο. Λt1μμα 2. Για κάθε σημείο Ακ Ε S υπάρχει ευθεία εκ που το περιέχει και είναι τέτοια

ώστε ID (εκ )I να γίνεται ελάχιστο.

Λπόδηξη ,. Περιστρέφουμε την ευθεία εκ με κέντρο το σημείο Ακ . Τότε η τιμή της διαφοράς D ( εκ ) μεταβάλλεται κατά δύο κάθε φορά η ευθεία εκ στην περιστροφή της αφήνει στο αριστε­ρό της ημιεπίπεδο ένα σημείο του S , το οποίο προηγουμένως βρισκόταν στο δεξιό ημιεπίπεδό της. Επειδή η διαφορά Δ - Α παίρνει ακέραιες τιμές και η τιμή της μεταβάλλεται κάθε φορά κα­τά δύο, οι ελάχιστες θετικές τιμές της είναι Ο και 1 . Όταν η ευθεία εκ περιστραφεί κατά 1 80° , τότε τα δύο ημιεπίπεδα αλλάζουν αμοιβαία θέση, οπότε θα είναι Δ - Α = Α- Δ => Δ = Α . Αυτό συμβαίνει όταν το πλήθος των σημείων του S είναι περιττό$ αριθμός, ενώ όταν ο αριθμός αυτός είναι άρτιος, τότε η ελάχιστη διαφορά είναι 1 .

Θεωρούμε τώρα το σημείο Χ Ε S και την ευθεία f του ανεμόμυλου που περιέχει το σημείο · Χ και είναι τέτοια ώστε η διαφορά D ( f ) να έχει ελάχιστη απόλυτη τιμή και επιπλέον να μην είναι παράλληλη σε κάποια ευθεία που ορίζεται από δύο σημεία του S . Το τελευταίο επιτυγχά-

ΕΥΚΛΕΙΔΗΣ Β ' 81 τ. l/16

Page 19: Ευκλειδης Β 81

-------- Μαθηματικοί Διαγωνισμοί - Μαθηματικές Ολυμπιάδες ------­

νεται με περιστροφή της ευθείας .e περί το Χ κατά κατάλληλη μικρή γωνία. Για το τυχόν άλλο σημείο του Χ1 ε S θεωρούμε ευθεία .e 1 που το περιέχει και έχει αντίστοιχες ιδιότητες με την ευ-θεία € . Επειδή η ευθεία € περιστρέφεται συνεχώς κάποια στιγμή θα γίνει παράλληλη και ο­μόρροπη με την ευθεία .€ 1 • Τότε όμως, σύμφωνα με το [λήμμα 1 ] , θα έχουμε ότι .e = .€ 1 • Επειδή η διαδικασία αυτή μπορεί να επαναλαμβάνεται άπειρες φορές, ο ανεμόμυλος μπορεί να χρησι­μοποιεί κάθε σημείο του συνόλου S ως κέντρο περιστροφής άπειρες φορές.

Έστω f : R � R μία πραγματική συνάρτηση που ορίζεται στο σύνολο R των πραγματι­κών αριθμών και ικανοποιεί τη σχέση f (χ + y} � yf (χ} + f ( f (χ)) (1 ), για όλους τους

πραγματικούς αριθμούς χ και y. Να αποδείξετε ότι f (χ} = Ο , για κάθε χ � Ο. ι\ ι\ ;σ !� Αν θέσουμε y = t - χ , τότε η ( 1 ) γράφεται στη μορφή f ( t) � if (x) - xf ( χ ) + f (f (x) ) . (2) Τώρα, για α , b ε R , θέτουμε στην (2) διαδοχικά t = f ( α ) , χ = b και t = f ( b ) , χ = α , οπότε

λαμβάνουμε : f ( f (α )) - f ( f ( b )) � f (α) f ( b ) - bf ( b ) } , , , , με προσθεση κατα μελη λαμβανουμε: f ( f ( b )) - f ( f (α) ) � f (α) f ( b ) - αf (α)

2 I (α ) .f ( b) � αl (α) + bl ( b) q ( 2 I (α) - b ) I ( b) � αf (α) . Από την τελευταία θέτοντας b = 2 f (α ) λαμβάνουμε af (α ) � Ο , από την οποία προκύπτει ότι f (α ) � Ο, για κάθε α < Ο . (3 ) Ας υποθέσουμε τώρα ότι υπάρχει χ0 ε IR τέτοιο ώστε f (χ0 ) > Ο . Τότε, λόγω της (2), ισχύει ότι

f (ι) < Ο, όταν if (x) - if (x) + J(f( χ)) < 0 ή όταν ι < .ςJ(.χ;,) (/�/(.χ;,)) , το οποίο αντίκειται στη I Χο

σχέση (3) . Άρα f (χ ) � Ο, για κάθε χ ε R. (4) Από (3 ) και (4) έχουμε: f(x) = 0, για κάθε χ < Ο . Επομένως μένει να αποδείξουμε ότι f (O) = Ο . Πράγματι, από την (4) έχουμε ότι f (O) � Ο ,

ενώ από τη (2), για t = χ < Ο , προκύπτει ότι f( χ) � f (f ( χ) ) ή Ο � f( 0) , η οποία σε συνδυασμό με τη σχέση (4) δίνει το ζητούμενο.

Π αματ{φψ. •η. Μετά την απόδειξη της σχέσης (3), η λύση θα μπορούσε να τελειώσει ως εξής: Γρά-φουμε Α = .f ( Ο) και Β= 1(1( Ο)) = I( Α) . Θέτοντας y = Ο στην ( 1 ) πpοκύπτει ότι Ι( χ) �Ι(Ι(χ)) , για

κάθε χ ε IR , από την οποία, για χ = Ο και χ = Α , λαμβάνουμε Α � Β � I ( Β) .

Όμως, ΑΒ = Af (Α ) � Ο , οπότε θα είναι Α � Ο � Β . Ομοίως, από τη σχέση B.f (Β ) � Ο λαμβάνουμε

ότι Β � Ο � I (Β ) . Άρα έχουμε Β = Ο και A = I (O) = I (B) � O = B, οπότε θα είναι και Α = Ο. Τώ­

ρα, από τη σχέση ( 1 ), για χ = Ο , προκύπτει ότι f (y ) � Ο, για κάθε y ε ffi. , από την οποία προκύπτει και το ζητούμενο.

Θα χρησιμοποιήσουμε τις ( 1 ), (2) από την προηγούμενη λύση και θα προχωρήσουμε με τέσσερα βήματα. Β ;)μα ] . Θα αποδείξουμε ότι η f παίρνει μόνο μη θετικές τιμές.

Ας υποθέσουμε ότι υπάρχει z ε ffi. τέτοιο ώστε f ( z ) > Ο. Θέτουμε στη (2) χ = z και Α := .f ( z ) , Β := -zf ( z ) - .f (.f ( z ) ) , οπότε λαμβάνουμε: l ( t ) � At + Β, για κάθε t ε IR . Άρα, αν για οποιο­δήποτε θετικό πραγματικό αριθμό t θέσουμε στη σχέση ( 1) χ = -t και y = t θα λάβουμε: '

ΕΥΚΛΕΙΔΗΣ Β ' 81 τ. 1/17

Page 20: Ευκλειδης Β 81

-------- Μαθηματικοί Διαγωνισμοί - Μαθηματικές Ολυμπιάδες --------f(O) �tf( -t) + f(f( -t)) � t( -At+B) +Jif( -t) +B�-t(At-B) +A( -At+B) +B=-At2 -( Α2 -B) t+( Α+1)Β. Αυτό όμως δεν είναι δυνατόν να αληθεύει για t αρκετά μεγάλο . Επομένως, έχουμε f (χ ) � Ο, για κάθε χ ε IR.. Από αυτό το συμπέρασμα, η ( 1 ) δίνει ότι: f(x+y) �J.f(x) , για κάθε x,y ε iR . (5)

Θα αποδείξουμε ότι η συνάρτηση f έχει ένα τουλάχιστον σημείο μηδενισμού.

Α ν είναι f ( Ο) = Ο, τότε ισχύει το ζητούμενο. Α ν f ( Ο) :;t: Ο, τότε f (Ο) < Ο . Από την ( 5), για χ = Ο προκύπτει ότι f (y) � yf ( Ο) , για κάθε y ε IR . Επομένως, μπορούμε να βρούμε έναν θετι-κό πραγματικό αριθμό αρκετά μεγάλο, ώστε να ισχύει f (α )2 > -f (Ο) . Θέτοντας b = f (α ) και αντικαθιστώντας χ = b και y = -b στην ( 5), λαμβάνουμε -b2 < f (Ο) � -bf ( b ) , δηλαδή b < f ( b) . Τελικά, εφαρμόζοντας τη σχέση (2) με χ = b και t = f ( b ) λαμβάνουμε

f (f (b ) ) � (f (b ) - b ) f (b ) + f (f (b ) ) => ο � (f (b ) - b ) f (b ) . Επομένως, f ( b) ;;:: Ο . Έτσι, σύμφωνα με το πρώτο βήμα, f ( b) = Ο .

Θα αποδείξουμε ότι, αν f (α ) = Ο και b < α , τότε f ( b) = Ο . Για την απόδειξη αυτού, aντικαθιστούμε στη σχέση (5) χ = b και y = α - b , οπότε λαμβάνουμε f ( b) ;;:: Ο , οπότε από το βήμα [ 1 ] προκύπτει η ισότητα f ( b) = Ο .

Θα αποδείξουμε ότι f (Ο) = Ο , που, σύμφωνα με το βήμα [3}, ολοκληρώνει την απόδειξη

του προβλήματος. Θεωρούμε ένα σημείο μηδενισμού r της f και θέτουμε στη σχέση ( 1 ) χ = r και y = Ο, οπότε λαμβάνουμε f(O) ;;:: O. Έτσι, από το βήμα [ 1 ] προκύπτει η ισότητα f (O) = Ο .

Έστω η ακέραιος, με η > Ο . Έχουμε μία ζυγαριά και η βάρη με τιμές 2° , 2\ . . . , 2"-1 •

Τοποθετούμε καθένα από τα βάρη στη ζυγαριά, το ένα μετά το άλλο, με μία ακολουθία n κινήσεων. Κατά τη πρώτη κίνηση επιλέγουμε ένα β�ρος και το τοποθετούμε πάνω στον α­ριστερό δίσκο της ζυγαριάς. Σε κάθε μία από τις επόμενες κινήσεις επιλέγουμε ένα από τα υπόλοιπα βάρη και το τοποθετούμε, είτε στον αριστερό δίσκο είτε στον δεξιό δίσκο της ζυ­γαριάς. Να προσδιορίσετε τον αριθμό των τρόπων με τους οποίους μπορούμε να εκτελέ­σουμε αυτές τις η κινήσεις κατά τέτοιο τρόπο, ώστε ο δεξιός δίσκος να μην είναι ποτέ βα­ρύτερος από τον αριστερό δίσκο.

Υποθέτουμε ότι n ;;:: 2 . Θα αποδείξουμε ότι: f (n ) = (2n - 1 ) f ( n - Ι ) . ( 1 ) Κατ' αρχή παρατηρούμε ότι κάθε σωστή τοποθέτηση των n βαρών στους δύο δίσκους της

ζυγαριάς ξεκινάέι με την τοποθέτηση κάποιου βάρους στον αριστερό δίσκο, οπότε μετά το πρώ­το βήμα ο αριστερός δίσκος θα είναι πάντοτε βαρύτερος κατά 1 τουλάχιστον από τον δεξιό δί­σκο. Επομένως, οποιαδήποτε σωστή τοποθέτηση των n βαρών στη ζυγαριά, χωρίς τη θεώρηση του βάρους 2° , οδηγεί σε μία σωστή τοποθέτηση των βαρών 2 1 , 2 2 , • • • , 2n- ι .

Α ν διαιρέσουμε τις τιμές όλων των δεδομένων βαρών με το 2, η απάντηση δεν αλλάζει, οπότε αυτά τα n - Ι βάρη μπορούν να τοποθετηθούν στη ζυγαριά με f ( n - 1 ) τρόπους.

Εξετάζουμε τώρα το βάρος 2° . Α ν αυτό τοποθετηθεί στο πρώτο βήμα, τότε πρέπει να τοποθε­τηθεί μόνο στον αριστερό δίσκο, ενώ διαφορετικά μπορεί να τοποθετηθεί σε οποιοδήποτε από τους δύο δίσκους, αφού μετά το πρώτο βήμα η διαφορά των βαρών του αριστερού και δεξιού δίσκου είναι τουλάχιστον 2 . Επομένως υπάρχουν ακριβώς 2n - 1 δυνατές τοποθετήσεις του βά­ρους 1 σε καθεμία από τις f ( n - Ι ) σωστές τοποθετήσεις των n - 1 βαρών με σκοπό να πάρου-

ΕΥΚΛΕΙΔΗΣ Β' 81 τ. 1/18

Page 21: Ευκλειδης Β 81

--------- Μαθηματικοί Διαγωνισμοί - Μαθηματικές Ολυμπιάδες -------­

με μία σωστή τοποθέτηση των n βαρών. Από αυτό προκύπτει η σχέση ( 1 ) . Επειδή είναι f ( Ι ) = Ι , με επαγωγή λαμβάνουμε ότι για κάθε θετικό ακέραιο n ισχύει:

Ι ( n) = 1 · 3 · 5 · · · ( 2n - Ι ) := ( 2n - Ι ) ! ! . Π rφπτ(j ρ ψτη ι . Είναι χρήσιμο να παρατηρήσει κανείς ότι η απάντηση είναι η ίδια για οποιο­

δήποτε σύνολο βαρών, στο οποίο τα βάρη είναι τέτοια ώστε έκαστο βάρος να είναι βαρύτερο από το άθροισμα όλων των μικρότερων βαρών από αυτό. Πράγματι, σε τέτοιες περιπτώσεις η δεδομένη συνθήκη του προβλήματος είναι ισοδύναμη με τη συνθήκη, κατά τη διάρκεια της διαδικασίας το μεγαλύτερο βάρος που έχει τοποθετηθεί στη ζυγαριά να είναι στον αριστερό δίσκο.

Π 2 . . Αντί να θεωρήσουμε το ελαφρύτερο βάρος, μπορούμε να θεωρήσουμε το βά-

ρος που τοποθετούμε στο τελευταίο βήμα. Αν αυτό είναι το 2"- 1 , τότε αυτό πρέπει να τοποθετηθεί μόνο στον αριστερό δίσκο, ενώ οποιοδήποτε άλλο βάρος μπορεί να τοποθετηθεί σε οποιοδήποτε από τους δύο δίσκους, αφού η συνθήκη του προβλήματος ικανοποιείται γιατί μέχρι τότε το βαρύτε­ρο από τα βάρη έχει ήδη τοποθετηθεί στον αριστερό δίσκο. Έτσι σε καθεμία από αυτές τις 2n - 1 περιπτώσεις ο αριθμός των σωστών τρόπων τοποθέτησης των προηγούμενων βαρών είναι

f ( n - Ι ) , οπότε πάλι προκύπτει η σχέση ( Ι ) . ηΗ�:ιως

Θα βρούμε πάλι τη σχέση ( 1 ) με διαφορετικό τρόπο. Θέτουμε f (Ο) = Ι . Θα βρούμε πρώτα μία αναδρομική σχέση για το f ( n ) . Υποθέτουμε ότι n � Ι και ότι το βάρος 2"-ι τοποθετείται στη ζυγαριά στο ί - βήμα, με Ι � ί � n . Το βάρος αυτό τοποθετείται στον αριστερό δίσκο. Στα προηγούμενα βήματα έχουμε (� - 1) επιλογές τοποθέτησης των βαρών και από την Παρατήρη-

ι - Ι ση 1 υπάρχουν f ( ί - Ι ) σωστές τοποθετήσεις αυτών. Για τα βήματα που ακολουθούν τα εναπο­μένοντα βάρη μπορούν να τοποθετηθούν σε οποιονδήποτε από τους δύο δίσκους, οπότε υπάρ­χουν ( n - ί) ! · 2"-1 δυνατοί τρόποι τοποθέτησης. Επομένως έχουμε

f (n ) = Σ(�� 1)f ( i - 1 ) (n - ί ) ! · 2"-i = I (n - 1 ) !(�(i �1) 2n-i (2) i�ι ι 1 i�ι ( ι 1 ) .

�n-2) '/(ί-Ι)zι-Η Αντικαθιστώντας στην σχέση (2) το n με το n - Ι λαμβάνουμε /( n-1) = �� ( ί -Ι) ! , οπότε από τη (2) προκύπτει f ( n ) = 2 (n - l ) � ( n - 2)!?�ί

ι}/) 2"_ , _, + f (n - I ) = (2n - 2) f ( n - I) + f ( n - I ) = (2n - I ) f ( n - I ) .

3 ' ' " τμΛ:rι ο;� { ι\ iJ(Hj i {: ι!ψγωυ Βλιί χο υ ) Ονομάζουμε L ( ί ) και R ( ί ) τα αθροίσματα των βαρών που έχουν τοποθετηθεί μέχρι και την

ί - κίνηση , ί = 1, 2 , . . . , n , στον αριστερό και στο δεξιό δίσκο της ζυγαριάς, αντίστοιχα. Λέμε ότι μία τοποθέτηση των βαρών στη ζυγαριά είναι καλή, αν ικανοποιεί τη συνθήκη του προβλήματος, δηλαδή, αν ισχύει L ( ί ) > R ( ί ) , για κάθε ί = 1, 2 , . . . , n .

Για την εύρεση του ζητούμενου αριθμού των καλών τοποθετήσεων θα χρησιμοποιήσουμε τα επόμενα δύο λήμματα:

', η L ( ί) > R ( ί ) <::::> μέχρι και την ί - κίνηση , το μέγιστο από τα βάρη που έχουν τοποθετηθεί στον αριστερό δίσκο είναι μεγαλύτερο από το μέγιστο από τα βάρη που έχουν τοποθετηθεί στο δεξιό δίσκο.

Λ,π/ti''i� . ιξη . Έστω ότι το μέγιστο βάρος που έχει τοποθετηθεί στον αριστερό (αντίστοιχα, στο δεξιό) δίσκο μέχρι την ί - κίνηση είναι το 2' (αντίστοιχα, το 2Υ ) . Γ ράφουμε:

ΕΥΚΛΕΙΔΗΣ Β ' 8 1 τ. 1/19

Page 22: Ευκλειδης Β 81

--------- Μαθηματικοί Διαγωνισμοί - Μαθηματικές Ολυμπιάδες --------

χ-1 v ι. Αν χ > y , τότε L ( i) � Υ > Σ 2i � Σ 2 i � R ( i) .

i=O i=O J•-1 χ

e Αντίστροφα, αν L(i) > R(i) και χ < y , τότε R (i) � 2Υ > Σ2ί � Σ2ί � L( i) , που είναι άτοπο. i=O i=O

Έστω f ( n ) το πλήθος των καλών τοποθετήσεων για τα n βάρη , n ε Ν* . Λήμμα Ζ . f (n + 1} = (2n + l} f (n) , n � l . Απ6δ�ιξη . Θ ' + 1 β ' 20 21 2n-l 2" δ ' ' εωρουμε τα n αρη , , . . . , , και ιακρινουμε τις περιπτωσεις: n ερίπτωση I . Στο τελευταίο βήμα τοποθετούμε στη ζυγαριά το βάρος γ . Τότε, από το

Λήμμα 1 , προκύπτει ότι το βάρος αυτό πρέπει να τοποθετηθεί μόνο στον αριστερό δίσκο. Το σύνολο των υπόλοιπων βημάτων αντιστοιχεί σε μία ακριβώς καλή τοποθέτηση των n βαρών. Αντίστροφα, κάθε καλή τοποθέτηση των n βαρών οδηγεί σε μία καλή τοποθέτηση των n + 1 βαρών με τελευταίο τοποθετούμενο βάρος το 2" στον αριστερό δίσκο. Επομένως ορίζεται μία αμφιμονοσήμαντη αντιστοιχία μεταξύ του συνόλου Κ " των καλών τοποθετήσεων των n βαρών και του συνόλου Κ n+ Ι ,2" των καλών τοποθετήσεων των n + 1 βαρών στις οποίες το βάρος 2" το-ποθετείται στο τελευταίο βήμα. 'Ετσι έχουμε Ικn+ Ι ,2" I = f ( n ) .

Περ ίπτωση 2 . Στο τελευταίο βήμα τοποθετούμε το βάρος 2a , Ο � α < n. Από το Λήμμα 1 προκύπτει ότι, αν στα n πρώτα βήματα η τοποθέτηση είναι καλή, τότε και

με την τοποθέτηση του βάρους 2a στο τελευταίο βήμα η τοποθέτηση παραμένει καλή, αφού έ­χει ήδη τοποθετηθεί το μέγιστο βάρος 2" στον αριστερό δίσκο.

Θεωρούμε τώρα μία καλή τοποθέτηση των n βαρών 2° , 2 1 , • • • , 2"- 1 • Θα την aντιστοιχίσουμε σε δύο καλές τοποθετήσεις των n + 1 βαρών. Στη πρώτη από αυτές το βάρος 2a τοποθετείται στον αριστερό δίσκο στο τελευταίο βήμα και στη δεύτερη το βάρος 2a τοποθετείται στον δεξιό δίσκο στο τελευταίο βήμα. Η aντιστοίχιση γίνεται ως εξής:

� Α ν στο βήμα ί τοποθετείται σε κάποιο δίσκο το βάρος 2k , k < α , τότε στη νέα τοποθέ­τηση των n + 1 βαρών στο βήμα ί το βάρος 2k παραμένει στον ίδιο δίσκο.

• Α ν στο βήμα ί τοποθετείται σε κάποιο δίσκο το βάρος 2k , k � α , τότε στη νέα τοποθέ­τηση των n + 1 βαρών τοποθετούμε στο βήμα ί το βάρος 2k+ Ι στον ίδιο δίσκο.

$ Στο τέλος της διαδικασίας τοποθετούμε το βάρος γ στον αριστερό ή στο δεξιό δίσκο της ζυγαριάς.

Σύμφωνα με το λήμμα [ 1 ] , εύκολα προκύπτει ότι σε καθεμία από τις δύο περιπτώσεις λαμβά­νουμε μία καλή τοποθέτηση των n + 1 βαρών.

Αντίστροφα, έστω μία καλή τοποθέτηση των n + I βαρών στην οποία στο τελευταίο βήμα τοποθετούμε το βάρος γ , Ο � α < n στον αριστερό ή στο δεξιό δίσκο της ζυγαριάς. Σε καθεμία από τις παραπάνω περιπτώσεις μπορούμε να aντιστοιχίσουμε μία καλή τοποθέτηση των n βα­ρών ως εξής:

"' Αν στο βήμα ί τοποθετείται σε κάποιο δίσκο το βάρος 2k , k < α , τότε στη νέα τοποθέ­τηση των n βαρών στο βήμα ί το βάρος 2k παραμένει στον ίδιο δίσκο.

• Αν στο βήμα ί τοποθετείται σε κάποιο δίσκο το βάρος 2k , k � α , τότε στη νέα τοποθέ-

ΕΥΚΛΕΙΔΗΣ Β ' 81 τ. l /20

Page 23: Ευκλειδης Β 81

-------- Μαθηματικοί Διαγωνισμοί - Μαθηματικές Ολυμπιάδες -------­

τη ση των n βαρών τοποθετούμε στο βήμα ί το βάρος 2k-ι στον ίδιο δίσκο .

• Κατάργηση του τελευταίου βήματος. Έτσι προκύπτει μία καλή τοποθέτηση των n βαρών. Πράγματι, αν υποθέσουμε ότι η τοποθέ-

τηση που προκύπτει δεν είναι καλή, τότε σε κάποιο βήμα θα ισχύει ότι max R (i) > max L (i ) . Αν είναι max R ( ί ) = 2-' , max L ( ί ) = 2Υ , τότε :

• Αν χ � α > Υ , τότε χ + 1 > Υ � γ+ι > 2Υ. > ' 1 1 2x+l 2y+l • Αν χ > Υ - α , τοτε χ + > Υ + � > .

• Αν α � χ > Υ , τότε χ > Υ � 2χ+' > 2Υ. Σε όλες τις περιπτώσεις το συμπέρασμα είναι άτοπο, γιατί η αρχική τοποθέτηση είναι καλή . Επομένως ορίζεται μία αμφιμονοσήμαντη αντιστοιχία μεταξύ του συνόλου Κ n των καλών το-

ποθετήσεων των n βαρών και του συνόλου Kn+ι .2a (Α ) , (αντίστοιχα, Kn+ι ,2a (Δ)), Ο � α < 1 , των καλών τοποθετήσεων των n + 1 βαρών στις οποίες το βάρος 2α , Ο � α < 1 τοποθετείται στο τελευταίο βήμα στον αριστερό (αντίστοιχα, δεξιό) δίσκο.

Άρα έχουμε f (n ) = IKn l = Jκn+ι ,2a (Α ) j = Jκn+ι ,2α (Δ ) j . Επέιδή τα σύνολα Kn+ι,2a (A) , Κn+ι,2a (Δ)), Ο� α< 1 , είναι μη κενά, ανά δύο ξένα μεταξύ τους και

n-1 η ένωσή τους ισούται με το σύνολο Κη+ι , έπεται ότι: f(n+1) = f(n) +2ΣJ( n) =(2n+ 1)f(n) .

i=O Επειδή είναι f ( 1 ) = 1 , τελικά με μία εύκολη επαγωγή λαμβάνουμε:

f ( n + 1 ) = ( 2n + 1 ) ( 2n - 1 ) · · · 3 · 1 = (2n + 1 ) ! ! ,

από την οποία προκύπτει το ζητούμενο αποτέλεσμα f ( n) = ( 2n - 1 ) ( 2n - 3) · · · 3 · 1 = ( 2n - 1) ! ! .

Π ρόβλημα 5. Έστω f μία συνάρτηση με πεδίο ορισμού το σύνολο των ακεραίων και τιμές στο σύνολο

των θετικών ακεραίων. Υποθέτουμε ότι, για οποιουσδήποτε δύο ακέραιους m και n , η δι­αφορά f ( m) - f (η ) διαιρείται με τον f ( m - η ) . Ν α αποδείξετε ότι, για όλους τους ακέραι-

ους m και η με f { m) � r ( η ) , ο αριθμός r ( η ) διαιρείται με τον f { m ) .

Λύση Για όλες τις λύσεις που ακολουθούν, σύμφωνα με την υπόθεση, έχουμε ότι: f (m - n ) if (m ) - f (n) , για κάθε m, n ε 7l. . ( 1 ) Ί ος τρόπος Αν f(x) = f(y) , τότε f(x) if(y) . Θεωρούμε x, y ε 7l. με f(x) <f(y) . Θα δείξουμε ότι: f(x) if (y) . Από την ( 1 ) για m = χ και n = Υ λαμβάνουμε f (χ - Υ ) ι ι ι (χ) - f ( y )I = f ( y) - f (χ) > ο '

οπότε f (χ - y) � f (y ) - f (χ ) < f (y) . Επομένως για τη διαφορά d = f (χ ) - f (χ - y) ισχύει ότι -f (y) < -f (χ - Υ) < d = f (χ) - f (χ - Υ) < f (χ) < f (y) · (η

Από την ( 1 ), για m = χ και n = χ -y , προκύπτει ότι f(x-(x-y)) = f(y) if(x) -f(x-y) =d . (3) Από τις (2) και (3) προκύπτει ότι d = O, δηλαδή f (x) = f (x - y) , οπότε από την ( 1 ) , για

m = χ και n = Υ , λαμβάνουμε : f(x) = f(x-y) jf(x) -f(y) � f(x) if(x) -(f(x) -f(y)) = f(y) . 2°" τρόπος Υποθέτουμε ότι δεν ισχύει το ζητούμενο. Ονομάζουμε ένα ζευγάρι ακεραίων ( m , n) κακό, αν

ΕΥΚΛΕΙΔΗΣ Β ' 81 τ. l/21

Page 24: Ευκλειδης Β 81

-------- Μαθηματικοί Διαγωνισμοί - Μαθηματικές Ολυμπιάδες --------f ( m ) ::; f ( n ) και ο f ( m ) δεν διαιρεί τον f ( n ) . Α ν άμεσα σε όλα τα ζεύγη ακεραίων που είναι κακά επιλέγουμε ένα ζεύγος ( m0 , n0 ) τέτοιο ώστε f ( m0 ) < f ( n0 ) και το f ( n0 ) να είναι ελάχι-στο. Επειδή έχουμε: f( m0 - ( m0 - n0 )) = f( n0 ) if( m0 ) -f( m0 - n0 ) < f( m0 ) < f( n0 ) , έπεται ότι f ( m0 ) - f ( m0 - n0 ) ::; Ο, δηλαδή είναι: f ( m0 - n0 ) ;?: f ( m0 ) • ( 4)

Επιπλέον, f(m0 - n0 ) if (m0 ) -f ( n0 ) < Ο , οπότε : f ( m0 - n0 ) ::; - (f ( m0 ) - f ( n0 ) ) < f (n0 ) . Επειδή το f ( n0 ) είναι ελάχιστο μεταξύ των κακών ζευγαριών, έπεται ότι το ζευγάρι

(m0 , m0 - n0 ) δεν είναι κακό, οπότε από την (4) λαμβάνουμε ότι f (m0 ) if (m0 - n0 ) . Όμως και f (m0 - n0 ) if (m0 ) - f ( n0 ) , οπότε και f(m0 ) if ( m0 ) - f (n0 ) , με συνέπεια f ( m0 ) if ( n0 ) . Άρα το ζευγάρι (m0 , n0 ) δεν είναι κακό, που είναι άτοπο.

Έστω ABC ένα οξυγώνιο τρίγωνο και Γ ο περιγεγραμμένος κύκλος του. Έστω f μία εφαπτομένη ευθεία του Γ , και έστω f a , f h και f, οι συμμετρικές ευθείες της f ως προς άξονα συμμετρίας τις ευθείες BC, CA και ΑΒ , αντίστοιχα. Να αποδείξετε ότι ο περιγε­γραμμένος κύκλος του τριγώνου που ορίζεται από τις ευθείες f. " , f. h και f. c εφάπτεται του κύκλου Γ .

Συμβολίζουμε με Τ το σημείο επαφής της ευθείας f με τον κύκλο Γ . Έστω Α' = f. b n f. c ,

Β' = f.a n ( , C' = f. a n f.h . Θεωρούμε το σημείο Α" πάνω στον κύκλο Γ , έτm ώστε ΤΑ = ΑΑ" (Α" * Τ , γιατί διαφορετικά η ΤΑ θα ήταν διάμετρος). Με ανάλσyο τρόπο ορίζουμε και τα σημεία Β" και C" . Επειδή τα σημεία C και Β είναι τα μέσα των τόξων TC" και ΤΒ" , αντίστοιχα, έχουμε: L( f.,RC) =L( f.,Π:)+L(Π:,RC) =2L( f.,'IC)+2L(1C,RC) =2( L( f.,'IC) +L(Π:,RC)) =2L( f.,Ir} =L( f.,f.0) .

Σχήμα 5 Επομένως οι ευθείες f. a και B"C" είναι παράλληλες. Ομοίως προκύπτει ότι f.6 1 1 A"C" και

f. c 1 1 Α" Β" , οπότε τα τρίγωνα Α 'Β' C ' και Α" Β" C" είναι ομοιόθετα ή το ένα είναι παράλληλη με­ταφορά του άλλου.

Στη συνέχεια θα αποδείξουμε ότι είναι πράγματι ομοιόθετα και ότι το κέντρο της ομοιοθεσίας, έστω Κ , ανήκει στον κύκλο Γ . Τότε και οι περιγεγραμμένοι κύκλοι των δύο τριγώνων θα είναι ομοιόθετοι ως προς την ίδια ομοιοθεσία κέντρου Κ , οπότε αυτοί θα εφάπτονται στο Κ .

Για την απόδειξη των παραπάνω χρειαζόμαστε τους επόμενους δύο ισχυρισμούς: 1 . Το σημείο τομής Χ των B"C και BC" ανήκει στην (, . 2 . Το σημείο τομής Ι των ευθειών ΒΒ' και CC' βρίσκεται πάνω στον κύκλο Γ .

ΕΥΚΛΕΙΔΗΣ Β' 81 τ. 1 /22

Page 25: Ευκλειδης Β 81

--------- Μαθηματικοί Διαγωνισμοί - Μαθηματικές Ολυμπιάδες --------

(1) Πράγματι, τα σημεία Χ και Τ είναι συμμετρικά ως προς την ευθεία BC , επειδή οι ευ­θείες CT και CB" είναι συμμετρικές ως προς την ευθεία αυτή, όπως είναι και οι ευθείες Β Τ και BC" .

(2) Θεωρούμε την περίπτωση που η ευθεία f δεν είναι παράλληλη προς κάποια πλευρά του τριγώνου ABC . Οι υπόλοιπες περιπτώσεις μπορούν να θεωρηθούν ως οριακές. Έστω D = R n BC , E = R n AC και F = f n AB .

Λόγω συμμετρίας, η ευθεία DB είναι μία από τις διχοτόμους των γωνιών των ευθειών B'F και DF. Ομοίως η ευθεία FB είναι μία από τις διχοτόμους των γωνιών των ευθειών B'F και DF . Επομένως το σημείο Β είναι το έκκεντρο ή ένα από παράκεντρα του τριγώνου B'DF . Σε κάθε περίπτωση έχουμε L( BD, DF) + L ( DF, FB) + L ( B'B, B'D) = 90° ,

οπότε L (B'B, B'C' ) = L (B'B, B'D) == 90° - L(BC, DF) - L (DF, BA) = 90° - L (BC, AB) .

Ομοίως, λαμβάνουμε και την ισότητα L ( C'C, B'C') = 90° - L ( BC, AC) , οπότε έχουμε L(BI, CI) = L (B'B, B'C' ) + L (B'C', C'C) = L (BC, AC) - L (BC, AB) = L ( AB, AC) ,

από την οποία προκύπτει ότι τα σημεία Α , Β , Ι , C είναι ομοκυκλικά. Τώρα είμαστε σε θέση να ολοκληρώσουμε τη λύση του προβλήματος. Έστω Κ είναι το δεύ­

τερο σημείο τομής της ευθείας Β 'Β " με τον κύκλο Γ . Με εφαρμογή του θεωρήματος του Pascal

στο εξάγωνο ΚΒ" CIBC" λαμβάνουμε ότι τα σημεία Β ' = ΚΒ " n !Β και Χ = Β" C n BC" είναι συνευθειακά με το σημείο τομής S των ευθειών C/ και C"K . Επομένως, S = CI n B'X = C' και τα σημεία C' , C " και Κ είναι συνευθειακά. Άρα το Κ είναι το σημείο τομής των ευθειών Β 'Β " και C'C" , οπότε το Κ είναι το κέντρο της ομοιοθεσίας που απεικονίζει το τρίγωνο Α' Β' C' στο τρίγωνο Α" Β" C" και αυτό ανήκει στον κύκλο Γ .

Ορίζουμε τα σημεία Τ, Α ' , Β ' και C' , όπως στην προηγούμενη λύση . Έστω Χ , Υ και Ζ , τα συμμετρικά του σημείου Τ ως προς τις ευθείες B C , CA και ΑΒ , αντίστοιχα.

Γ ι' ' b. - . _ Ki Α , , . . ..... . . '

· - · - · - · -. - .f.α_.,.

Σχήμα 6

,.

ι' / /

I

Σημειώνουμε ότι οι ορθές προβολές του σημείου Τ πάνω στις πλευρές του τριγώνου ABC ορίζουν την ευθεία Simson του Τ ως προς το τρίγωνο ABC , οπότε και τα συμμετρικά Χ , Υ και Ζ των προβολών θα είναι συνευθειακά. Επιπλέον, έχουμε ότι Χ Ε B'C' , Y Ε C 'A ' και z Ε Α 'Β ' .

ΕΥΚΛΕΙΔΗΣ Β ' 81 τ. l/23

Page 26: Ευκλειδης Β 81

-------- Μαθηματικοί Διαγωνισμοί - Μαθηματικές Ολυμπιάδες --------

Έστω a = L(f, TC) = L(BT,BC) . Χρησιμοποιώντας τη συμμετρία ως προς τις ευθείες AC και BC , λαμβάνουμε τις ισότητες L(BC,BX) =L(Bτ,BC) =a κm L(XC,XC) =L(f,Π') =L(YC,YC) =α . Επειδή είναι L (XC, XC') = L (YC, YC' ) , τα σημεία X , Y, C, C ' ανήκουν στον ίδιο κύκλο, έστω Γ c • Ομοίως ορίζουμε και τους κύκλους Γ α και Γ 6 • Έστω Γ ' ο περιγεγραμμένος κύκλος του τριγώνου Α 'Β' C'.

Από το θεώρημα του Miquel για τις τέσσερις ευθείες A 'B' , A ' C ' , B 'C' και ΧΥ προκύπτει ότι οι κύκλοι Γ', Γα , Γ6 και Γc τέμνονται σε κάποιο σημείο, έστω Κ . Θα αποδείξουμε ότι το ση­μείο Κ ανήκει στον κύκλο Γ και ότι οι εφαπτόμενες των κύκλων Γ και Γ' στο σημείο αυτό συμπίπτουν, το οποίο και ολοκληρώνει τη λύση του προβλήματος.

Λόγω συμμετρίας, έχουμε ότι ΧΒ = ΤΒ = ΖΒ , οπότε το σημείο Β είναι το μέσον ενός από τα δύο τόξα ΧΖ του κύκλου Γ6 • Άρα έχουμε L (ΚΒ, ΚΧ) = L(XZ, XB) και ομοίως προκύπτει η ισότητα L (ΚX, KC) = L (XC, XY) . Με πρόσθεση κατά μέλη των δύο τελευταίων ισοτήτων λαμβάνουμε L (KB, KC) = L (XZ, XB) + L (XC, XZ) = L (XC, XB) = L (TB, TC) , λόγω και της συμμετρίας των Τ και Χ ως προς την ευθεία BC . Επομένως Κ Ε Γ.

Στη συνέχεια υποθέτουμε ότι k είναι η εφαπτομένη του κύκλου Γ στο σημείο Κ . Τότε έ­χουμε L (k, KC' ) = L (k, KC) + L (KC, KC') = L (KB, BC) + L (XC, XC')

= ( L (KB, BX) - L (BC, BX)) + a = L (KB', B'X) - a + a = L (KB', B'C' ) , από την οποία προκύπτει ότι και ο κύκλος Γ ' έχει την ευθεία k εφαπτομένη στο σημείο Κ .

3"ς ·φόπος (Λ�Ίση τ ο υ Γεωργίου Καλαντζ11) Έστω S το σημείο επαφής της f με τον κύκλο Γ . Υποθέτουμε χωρίς βλάβη της γενικότητας

ότι το σημείο S βρίσκεται πάνω στο μικρό τόξο BC . Έστω ότι οι ευθείες fα , f 6 ,( τέμνονται ανά δύο στα σημεία P, Q, R. Ονομάζουμε Ω τον περιγεγραμμένο κύκλο του τριγώνου PQR . Για την επίλυση του προβλήματος θα αποδείξουμε ότι οι κύκλοι Ω και Γ έχουν ένα τουλάχι­στον σημείο κοινό στο οποίο έχουν κοινή εφαπτομένη . Λόγω συμμετρίας οι ευθείες f, fa και BC συντρέχουν, έστω στο σημείο Α' . Αντίστοιχα οι ευθεί-

ες f, f 6 και CA συντρέχουν στο σημείο Β' και οι ευθείες f, f c και ΑΒ συντρέχουν στο σημείο C' . Έστω S1 , S2 και S3 οι ορθές προβολές του σημείου S πάνω στις πλευρές BC, CA και ΑΒ ,

αντίστοιχα. Επειδή το σημείο S ανήκει στον περιγεγραμμένο κύκλο του τριγώνου ABC τα ση­μεία S1 , S2 και S3 είναι συνευθειακά (ορίζουν την ευθεία Simson του S ) . Θεωρούμε και τα σημεία 1Ί , Τ2 και Τ3 , συμμετρικά του σημείου S ως προς τα σημεία S1 , S2 και S3 , αντίστοιχα. Τότε, λόγω ομοιοθεσίας ως προς κέντρο το σημείο S , και τα σημεία 1Ί , Τ2 και Τ3 , είναι συνευθειακά. Επιπλέον, λόγω συμμετρίας 1Ί Ε fc , T2 Ε f6 και τ; Ε fa .

Έχουμε ότι LRPQ = π - LPB'C' - LPC'B' = π - ( π - 2LSCΆ) - ( π - 2LSBΆ) = 2 ( LSCΆ + LSBΆ) - π = 2 (π - LA ) - π = π - 2LA. Επίσης έχουμε και τις ισότητες

LRPQ = LRA'B' + RB'A' = LRA'B' + (π - LRB'S) = π - 2 ( LCB'S - LCA' S) = π - 2LC. LRQP = π - (π - 2LΑ) - (π - 2LC) = π - 2LB .

Από το πλήρες τετράπλευρο PRIΊI;I;Q προκύπτει ότι ο περιγεγραμμένος κύκλος Ω του τριγώνου ΕΥΚΛΕΙΔΗΣ Β ' 81 τ. l/24

Page 27: Ευκλειδης Β 81

-------- Μαθηματικοί Διαγωνισμοί - Μαθηματικές Ολυμπιάδες -------­

PQR συντρέχει με τους περιγεγραμμένους κύκλους των τριγώνων ΡΊ;Ι;,RΙ;ΙΊ και Qτ.,Τ3 στο ση­μείο Miquel, έστω Μ , του τετραπλεύρου. Άρα έχουμε LRMQ =π-LRPQ = 2LA. ( 1 )

\ ·,

·, ·,

'' · .... . .... . .... .

Σχήμα 7

\ I \ I

' / · , / .

' · , , / ·

· , . ..... . ... · " · ' , Ρ � · - · - · - · � ι '

Από το εγγράψιμο τετράπλευρο Rτ.,Μτ2 προκύπτουν και οι ισότητες Lτ.,Μτ2 = Lτ.,RP = π - 2LB και L7ΊRI; = 2LC (2) Επειδή, λόγω συμμετρίας είναι CS = C7Ί και CS = CT2 , το σημείο C είναι το περίκεντρο του

τριγώνου 7ΊSΤ2 • Άρα έχουμε L7ΊCT2 = 2LJΊST2 (εγγεγραμμένη - επίκεντρη γωνία) = 2LSβS2 = 2LS1CS2 (από εγγράψιμοSβCS2 ) = 2LC, οπότε, λόγω της (2), τα σημεία C,τ;,Μ,Ι; και R είναι ομοκυκλικά, και άρα LCMR = LCΓβ = LCJΊA' = LCSA' ( συμμεφία ως προ; BC) = LCMR = = LSA C (γωνία χορδής-εφαπτομένης) (3)

Ομοίως αποδεικνύουμε ότι το Β είναι το περίκεντρο του τριγώνου S7ΊΙ; , ότι είναι ομοκυ­κλικό με τα σημεία τ., , Τ3 , Q, Μ και τελικά προκύπτει η ισότητα LBMQ = L.BAS . (4)

Από τις σχέσεις (3) και (4) λαμβάνουμε ότι L.BMC = L.BMS + LSMC = L.BAS + LCAS = L.A , δηλαδή, το σημείο Μ ανήκει στον κύκλο Γ . Συνεπώς οι κύκλοι Ω και Γ έχουν ένα τουλάχι­στον κοινό σημείο, οπότε μένει να αποδείξουμε ότι εφάπτονται στο σημείο αυτό.

Έστω χ η εφαπτομένη του Ω στο σημείο Μ. Τότε LRMx = LRQM(από το εγγράψιμοRΡQΜ)= LJΊQΜ= LJΊΒΜ(από τοεγγεγραμμένο JΊBI;QM). (5)

Για να εφάπτεται η ευθεία χ στον κύκλο Γ , αρκεί: LC!vfx = LCBM ( χορδής-εφαπτομένης) <::::> LCMR + L.RMx = LCBJΊ + L.7ΊΒΜ � LCMR = LCBJΊ (από την (5 ) ) � LCJΊR = LCBS ( χορδής-εφαπτομένης) � LC7ΊA' = LCSA', που ισχύει, (συμμετρία ως προς την BC) .

ΕΥΚΛΕΙΔΗΣ Β ' 81 τ.l /25

Page 28: Ευκλειδης Β 81

-------- Μαθηματικοί Διαγωνισμοί - Μαθηματικές Ολυμπιάδες --------

Ρουμανία, Ιάσιο, 3-8 Μιiίου 2011

Η 27η Βαλκανική Μαθηματική Ολυμπιάδα έγινε στο Ιάσιο της Ρουμανίας από 3 έως 8 Μα'fου 20 1 1 . Επισήμως συμμετείχαν εννέα χώρες: Αλβανία, Βουλγαρία, Ελλάδα, Κύπρος, Μαυροβούνιο, Δημοκρατία της Μολδαβίας, Π.Γ.Δ. της Μακεδονίας, Ρουμανία, Σερβία και Τουρκία. Ανεπισήμως συμμετείχαν και οι χώρες: Αζερμπαϊτζάν, Γαλλία, Ηνωμένο Βασίλειο, Ιταλία, Ινδονησία, Καζακστάν, Σαουδική Αραβία, Τατζικιστάν, Τουρκμενιστάν και η δεύτερη ομάδα της Ρουμανίας. Στη βαθμολογία μεταξύ των χωρών μελών πρώτευσε η Ρουμανία με 1 70 βαθμούς, ενώ η Ελληνική ομάδα κατέλαβε την πέμπτη θέση με 84 βαθμούς. Την Ελληνική ομάδα αποτελούσαν οι μαθητές:

Γεώργιος Βλάχος Αργυρό μετάλλιο Γεώργιος Καλαντζής Χάλκινο μετάλλιο Σπυρίδων Κακαρούμπας Χάλκινο μετάλλιο Παναγιώτης Λώλας Χάλκινο μετάλλιο Αλέξανδρος Μουσάτοβ Χάλκινο μετάλλιο Ζαχαρίας Τσαμπασίδης Συμμετοχή.

Αρχηγός της Ελληνικής ομάδας ήταν ο ομότιμος Καθηγητής του Πανεπιστημίου Ιωαννίνων Θεόδωρος Μπόλης και υπαρχηγός ο διδάκτωρ Μαθηματικός Δημήτριος Κοντοκώστας.

Έστω ABCD ένα εγγράψιμο τετράπλευρο που δεν είναι τραπέζιο και έστω Ε το σημείο τομής των διαγωνίων του. Τα μέσα των πλευρών ΑΒ και CD είναι τα F και G αντίστοι­χα, και η ευθεία R. περνάει από το G και είναι παράλληλη προς την ευθεία ΑΒ . Τα ίχνη των κάθετων από το Ε προς τις ευθείες R. και CD είναι τα Η και Κ , αντίστοιχα. Να αποδείξετε ότι οι ευθείες EF και ΗΚ είναι κάθετες. [Ηνωμένο Βασίλειο]

Α

Τα σημεία E,K,H, G ανήκουν στον ίδιο κύκλο διαμέτρου GE , οπότε L.EHK = L.EGK . ( 1 )

Επίσης, από LDCA = L.DBA

τις και

ισότητες CE ΒΕ

- = -

CD ΒΑ , , CE 2CE 2ΒΕ ΒΕ επεται οτι - = -- = -- = -

CG CD ΒΑ BF ' οπότε τα τρίγωνα CGE και BFE είναι όμοια. Άρα θα έχουν και LEGC = LBFE , οπότε από τη σχέση ( 1 ) έπεται ότι LEHK = LBFE .

Όμως είναι ΗΕ j_ FB και επειδή οι Σχήμα 1 ευθείες ΗΚ και FE λαμβάνονται με περιστροφή αυτών των ευθειών κατά

την ίδια (προσανατολισμένη) γωνία, έπεται ότι και FE j_ ΗΚ .

ΕΥΚΛΕΙΔΗΣ Β ' 81 τ.l/26

Page 29: Ευκλειδης Β 81

-------- Μαθηματικοί Διαγωνισμοί - Μαθηματικές Ολυμπιάδες --------

Α ν χ, y, z πραγματικοί αριθμοί με χ + y + z = Ο , να αποδείξετε ότι

χ(χ + 2) + y(y + 2) + z(z + 2) > 0 2χ2 + ι 2y2 + ι 2z2 + ι -

Πότε ισχύει η ισότητα ; [Ελλάδα, προτάθηκε από τον Σ. Μπραζιτίκο]

χ( χ + 2) + y(y + 2) + z(z + 2) > Ο _.._.... 2χ(χ + 2) + 2y(y + 2) + 2z(z + 2) > 0 Έχουμε ότι -........-2χ2 + 1 2y2 + Ι 2z2 + Ι - 2χ2 + 1 2y2 + 1 2z2 + 1 -

θ ' 1 ' θ λ ' ' δ 'ξ ' (2χ+1)2 (2y+1)2 (2z+1)2 3 ( 1 ) και προσ ετοντας σε κα ε κ ασμα αρκει να ει ουμε οτι 7 + 7 + 2 � •

2χ- + 1 2y +1 2z + 1 Τώρα έχουμε ότι ανάμεσα στους 3 πραγματικούς αριθμούς x, y, z , με άθροισμα Ο, δύο θα

είναι ομόσημοι. Λόγω συμμετρίας μπορούμε να υποθέσουμε ότι αυτοί είναι οι y, z, δηλαδή yz � Ο . Από την ανισότητα Cauchy-Schwartz έχουμε

(2y + 1)2 (2z + 1)2 (2y + 2z + 2)2 (2(-χ) + 2)2 (2(-χ) + 2)2 (2) 2y2 + 1 + 2z2 + 1 � 2(/ + z2 + 1) = 2((y + z)2 - 2yz + l) � 2((-χ)2 + 1) ' όπου η τελευταία ανισότητα ισχύει γιατί yz � Ο . Η σχέση (2) γράφεται και στη μορφή

(2y + Ι)2 (2z + 1)2 2(χ - Ι)2 ...:..._::.-:-� + > (3 ) Επομένως, αρκεί να αποδείξουμε ότι 2 y2 + 1 2z2 + Ι - χ2 + Ι . (2χ + Ι)2 2(χ - Ι)2 3 (2x + l ) 2 Ι 2(χ - 1) 2 2 0 2χ(χ + 2) 4χ > Ο -'----'-- + > <=> - + - > <=> - --

2χ2 + Ι χ2 + Ι - 2χ2 + 1 χ2 + Ι - 2χ2 + Ι χ2 + Ι -

<=> 2x ( (x + 2)(x2 + 1 ) -;(2x2 + I) J � O <=> 2χ2 (χ - 1)2 � Ο, (2χ2 + 1)(χ- + 1 ) (2χ2 + 1)(χ2 + Ι)

η οποία προφανώς ισχύει, οπότε έχουμε το ζητούμενο . Η ισότητα στην τελευταία ισχύει αν χ = Ο ή χ = 1 , και η ισότητα στην ( Ι ) ισχύει αν y = z ή z = Ο , οπότε η ισότητα ισχύει αν

(x,y, z) = (0,0,0) , (x, y, z) = (1,-..!.. ,_..!..) και όλες σε όλες τις κυκλικές μεταθέσεις αυτών. 2 2

Σ η μ ;:; , Η ανισότητα (3) μπορεί να λυθεί και με απευθείας πράξεις, όπου και πάλι μετά τις

aπαλοιφές θα καταλήξουμε στην ανισότητα χ2 (χ - Ι )2 � Ο που αποδεικνύει την ανισότητα

Έστω S ένα πεπερασμένο σύνολο θετικών ακεραίων το οποίο έχει την ιδιότητα: αν χ εί­ναι ένα στοιχείο του S , τότε και όλοι οι θετικοί διαιρέτες του χ είναι στοιχεία του S . Ένα

μη κενό υποσύνολο τ του s είναι καλό, αν οποτεδήποτε χ, Υ Ε τ και χ < Υ ' τότε ο λόγος r χ είναι δύναμη ενός πρώτου ακέραιου. S . Ένα μη κενό υποσύνολο Τ του S είναι κακό, αν

οποτεδήποτε χ, y Ε Τ και χ < y , τότε ο λόγος Υ

δεν είναι δύναμη ενός πρώτου ακέραιου. χ

Δεχόμαστε ότι κάθε υποσύνολο του S με ένα στοιχείο είναι ταυτόχρονα και καλό και κακό. Έστω k ο μεγαλύτερος δυνατός αριθμός στοιχείων ενός καλού υποσυνόλου του S . Να α­ποδείξετε ότι ο k είναι επίσης ο ελάχιστος αριθμός κακών υποσυνόλων του S που είναι ανά δύο ξένα μεταξύ τους και η ένωσή τους ισούται με το σύνολο S . [Βουλγαρία]

Σημειώνουμε ότι ένα κακό υποσύνολο του S περιέχει το πολύ ένα μόνο στοιχείο από ένα καλό υποσύνολο του S , οπότε μία διαμέριση του S σε κακά υποσύνολα έχει τουλάχιστον τόσα στοιχεία όσα έχει ένα μέγιστο καλό υποσύνολο . Επομένως, αρκεί να κατασκευάσουμε k κακά

ΕΥΚΛΕΙΔΗΣ Β ' 81 τ. l/27

Page 30: Ευκλειδης Β 81

--------- Μαθηματικοί Διαγωνισμοί - Μαθηματικές Ολυμπιάδες --------­

υποσύνολα του S που είναι ανά δl)ο ξένα μεταξύ τους και έχουν ένωση το σύνολο S . Σημειώνουμε επίσης ότι τα στοιχεία ενός καλού υποσυνόλου του S πρέπει να είναι ανάμεσα

στους όρους μιας γεωμετρικής προόδου με λόγο ένα πρώτο ακέραιο, αφού, αν χ < y < z είναι στοιχεία ενός καλού υποσυνόλου του S , τότε y = xpa και z = yqP = xpa qP , για κάποιους πρώτους p, q και για κάποιους θετικούς ακέραιους α και β . Επομένως, για να είναι ο .:_

χ δύναμη πρώτου ακέραιου πρέπει p = q . · Έστω τώρα m = max { exp Ρ χ : χ ε S και p πρώτος} , όπου exp I' χ είναι ο εκθέτης του πρώτου p στη κανονική ανάλυση του χ

Σημειώνουμε ακόμη ότι ένα μέγιστο καλό υποσύνολο του S πρέπει να είναι της μορφής {α, ap, . . . , αpι" } , για κάποιο πρώτο θετικό ακέραιο p και για κάποιο θετικό ακέραιο α , ο οποίος δεν διαιρείται με το p . Επειδή ο μεγαλύτερος δυνατός αριθμός στοιχείων ενός καλού

υποσυνόλου του S δίνεται ότι είναι k , έπεται ότι k = m + 1 , οπότε μία διαμέριση του S σε κακά υποσύνολα έχει τουλάχιστον m + 1 στοιχεία.

Τελικά, επειδή το ιn είναι μέγιστο μεταξύ των εκθετών exp Ρ χ , τα σύνολα

si = {χ : χ ε S και Σ expr x = i (mod k )} , ί = 0, 1 , . . . , k - 1, με Ρ

σχηματίζουν μία διαμέριση του S σε k κακά υποσύνολα, οπότε ο k = m + 1 είναι ο ελάχιστος αριθμός υποσυνόλων ξένων ανά δύο με ένωση το S , όπως ζητείται.

Π αρατήρηση Μία διαφορετική παρουσίαση της ίδιας λύσης μετά την πρώτη παράγραφο που αφορά την

κατασκευή των k κακών υποσυνόλων του S μπορεί να δοθεί ως εξής:

Έστω p1 , p2 , • • . , Ριι όλοι οι πρώτοι ακέραιοι του συνόλου S . Επειδή το σύνολο S περιέχει όλους τους διαιρέτες των στοιχείων του, κάθε στοιχείο του S πρέπει να είναι της μορφής

,. ι· ι· ' < k J ' θ . 1 2 ' θ ' Χ . Ο χ = p1 ' p2' • • • Ρι;' , οπου t; _ · -για κα ε ι = , , . . . , n , αφου οι αρι μοι -. , 1 = , . . . , t; p;'

σχηματίζουν ένα καλό υποσύνολο του S με t; + 1 στοιχεία. Για κάθε τέτοιο στοιχείο χ ε S ορίζουμε σ (χ) = 'ί + r2 + . . . + r,, και πaρατηρούμε ότι, για χ, y ε S με χ < y που ανήκουν σε ένα καλό υποσύνολο, ισχύει: 1 � σ (y) - σ( χ) � k - 1 . (*)

Θεωρούμε τώρα τα σύνολα S; = {χ ε S : σ ( χ) = ί ( mod k )} , ί = 1, 2, . . . , k , τα οποία, σύμφωνα με τους προηγούμενους ορισμούς και της σχέσης (*), είναι κακά υποσύνολα του S , ανά δύο ξένα μεταξύ τους και έχουν ένωση το S .

Π ρόβλημα 4 Έστω ABCDEF ένα κυρτό εξάγωνο εμβαδού 1 , του οποίου οι απέναντι πλευρές είναι

παράλληλες. Οι ευθείες AB, CD και EF τέμνονται ανά δύο και τα σημεία τομής τους ορί-

ζουν τρίγωνο. Ομοίως, οι ευθείες BC, DE και F Α τέμνονται ανά δύο και τα σημεία τομής

τους ορίζουν ένα άλλο τρίγωνο. Να αποδείξετε ότι το εμβαδόν ενός τουλάχιστον από τα δύο

' ' ' λ ' ' 3

[Β λ ' ] αυτα τριγωνα ει ναι του αχιστον ει ναι - . ου γαρια 2 Λύση Σημειώνουμε ότι στη λύση που ακολουθεί οι δείκτες παίρνουν τιμές από Ο μέχρι 5 και γίνεται

αναγωγή modulo 6 . Συμβολίζουμε τις κορυφές του εξαγώνου κυκλικά Αο , Α1 , • • • , As και έστω ότι

οι ευθείες των απέναντι πλευρών Α;Α;+ ι και Ai+2Ai+3 τέμνονται στα σημεία Β; .

ΕΥΚΛΕΙΔΗΣ Β ' 81 τ.l/28

Page 31: Ευκλειδης Β 81

--------- Μαθηματικοί Διαγωνισμοί - Μαθηματικές Ολυμπιάδες

Για να αποδείξουμε ότι το εμβαδόν ενός τουλάχιστον από τα τρίγωνα Β0Β2Β4 , Ββ3Β5 είναι

μεγαλύτερο ή ίσο του � , αρκεί να αποδείξουμε ότι το συνολικό εμβαδόν των έξι τριγώνων 2

5

Αί+βίΑί+l είναι τουλάχιστον 1 , δηλαδή Σ area ( Ai+βiAi+l ) :::::: 1 . i=O

83 Α4 �- - - - - - - - - - - - - - - - - - - - - - - - - - - - - - -ι.------------------------------� ' ' ' ' ' '

\, ,\ Α' \ ' 5

\, Α' � Α'3 Α2 \ 1 '---- - - - - - - - - - - - - - - - - - - - - - - - - -' ' ' ' ' ' ' ' ' ' ' ' ' ' ' ' ' ' ' ' ' ' ' ' ' ' ' ' ' ' ' ' ' ' ' '

Α \ ,ι \ 5

' • • • • • • • ·fl

'· · · • • • • • • • • • • • • • • • • • · �· I 1 1 Β' 8 3 ' ' 4 ,ι' ,ι'

·--- - - - - -•'-------------84 Αο Α1 Σχήμα 2

Κατ' αρχή θεωρούμε το συμμετρικό s; κάθε σημείου Bi ως προς το μέσο των πλευρών Αί+ΙΑί+l . Θα αποδείξουμε ότι τα έξι τρίγωνα Ai+β;Ai+l καλύπτουν το εξάγωνο. Γι αυτό θεωρούμε και το συμμετρικό �ί+ι του σημείου �ί+Ι ως προς το μέσο του ευθύγραμμου τμήματος �i�i+l , ί = Ο, 1, 2 . Έτσι το εξάγωνο χωρίζεται σε τρία παραλληλόγραμμα A2;�i+Ι�i+l�i+i ' ί = 0, 1, 2 και σε ένα τρίγωνο Α;Α�Α;, , πιθανώς εκφυλισμένο . Παρατηρούμε κατ ' αρχή ότι κάθε �ί�ί+Ι �ί+l�ί+Ι , παραλληλόγραμμο καλύπτεται από το ζευγάρι των τριγώνων

( A2 ;B� i+5A2i+ 1 , A2i+β�iA2 i+2 ) , ί = Ο, 1 , 2 . Η απόδειξη ολοκληρώνεται με την απόδειξη του ότι ένα τουλάχιστον από τα παραπάνω

ζευγάρια τριγώνων περιέχει ένα τρίγωνο που καλύπτει το τρίγωνο Α;Α�Α; . Γι αυτό, αρκεί να αποδείξουμε ότι: �β� i+S :::::: �iA�i+S και A2 i+2B�i :::::: �i+2A� i+J ' για κάποιους

δείκτες ί , J ε {Ο, 1 , 2 } . Έχουμε ΑιΒs �βs

-- = --

A4As AsBJ και ΑJΒι Α2Α3 -- = --

�Α ΑΟΒ5 ' οπότε λαμβάνουμε

ο , , π2 A2 i+2B�i --1 � Α2Β(; . Α4Β� . �β� --1 . (2) μοιως, προκυπτει και η ισότητα -.,....,.-i=O A21+2A�i+J Α2Α� Α4Α; �>Α;

Από τις ( 1 ) και (2) προκύπτει το ζητούμενο . Δεύτερη λύση Ας είναι A1A2B1B2C1C2 το δοσμένο εξάγωνο και ABC , A'B'C' τα δύο τρίγωνα που

σχηματίζουν οι τομές των εναλλάξ πλευρών του, όπως στο σχήμα 3 . Το ζητούμενο ισοδυναμεί με την ισχύ της σχέσης E(ABC) +.E(A'B'C') :::::: 3 Για την απόδειξή της ας ονομάσουμε BC = a, CA = b, ΑΒ = c τα μήκη των πλευρών του ABC , Ε = 1 το εμβαδόν του εξαγώνου και Ε; , ε; τα εμβαδά των έξι μικρών τριγώνων έξω από το

ΕΥΚΛΕΙΔΗΣ Β' 81 τ. t/29

Page 32: Ευκλειδης Β 81

--------- Μαθηματικοί Διαγωνισμοί - Μαθηματικές Ολυμπιάδες -------­

εξάγωνο όπως στο σχήμα 3 . Ισοδυνάμως, πρέπει να αποδείξουμε (Ε ι + Ε2 + Ε3 + Ε ) + ( Ε; + Ε; + Ε; + Ε) � 3 , δηλαδή ότι Ε ι + Ε2 + Ε3 + Ε; + Ε; + Ε; � 1 .

Α

ct'3=c( l -t ! "t2) bt'2=b( l -t3-t l ) I

Ε ι C ' ι

ι

,' Β' ι

t 1 a

c2 Ε

Ε2

Β

Σχήμα 3 Α' ', atΊ=a( l -t2-t3) c

Τα έξι μικρά τρίγωνα είναι όμοια με το ABC εξαιτίας της παραλληλίας των πλευρών του εξαγώνου. Αν t ι , tυ t3 οι λόγοι ομοιότητας των ΑΒ

2Cι , ΒC2Αι , CΑ

2Βι ως προς το ABC

αντιστοίχως, και t 1 ', t2 ', t3 ' οι λόγοι ομοιότητας των Α'ΑιΑ2 , Β 'Β 1Β2 , C'CιC2 ως προς το ABC ,

η αποδεικτέα γράφεται E(ABC) · ( t � + t� + t� + ς + t/ + t/ ) ;:::: 1 . Έχουμε ΒΑ, = t 2a και A2C = t3a οπότε ΑιΑ2

= BC - ΒΑι - A2C = (1 - t2 - t3 )a ,

Συνεπώς ι ; = 1 - ι2 - ι3 και ομοίως t� = 1 - ι ι - ι3 , t� = 1 - ι ι - ι2 •

'Ετσι η αποδεικτέα σχέση γράφεται E(ABC)·( t� + t; + t; +( 1 - t2 - ι3 )2 + ( 1 - t, - t3 )

2 + (1 - t, - t2 )2 ) � 1

Όμως E(ABC) � Ε , + Ε, + Ε, + Ε � E(ABCJ · ( ι; + ti + t/) + I � E(ABC) � ( 2 l 2 ') · 1 - ι ι + t2 + ι3

Άρα πρέπει ισοδυνάμως να αποδείξουμε ( αφού 1 - ( ι � + t; + t3 2 ) > Ο ) την 2 ( ι � + t; + ι; ) + ( ι ι ι 2 + ι2 ι3 + ι3 ι ι ) - 2(t ι + ι 2 + t3 ) + 1 ;:::: ο

Πράγματι, θεωρώντας το αριστερό μέλος ως τριώνυμο του t ι 2t� + ( t2 + t3 - 2) t ι + (2 ( t; + t/ ) + t2 t3 - 2(t2 + t3 ) + 1 ) � o

αυτο εχει θετικό συντελεστή μεγιστοβάθμιου όρου, οπότε το ζητούμενο ισχύει αφού το τριώνυμο έχει μη θετική διακρίνουσα D . Πράγματι είναι

ο ::; O <=> (t2 + t3 -2)2 -s ( 2 ( t; + t� )+ ι2ι3 -2(t2 + t3 ) + 1 ) ::; ο <=> Ι sι; + (6t3 - 12)t2 + ( t sι; - 1 2t3 +4) ;:::: ο, που αληθεύει, αφού αν θεωρήσουμε το αριστερό μέλος ως τριώνυμο του t2 , αυτό έχει θετικό συντελεστή μεγιστοβάθμιου όρου και μη θετική διακρίνουσα

Dι = ( 6ι3 - 1 2)2 - 4 · 1 5 · ( 1 5ι; - 1 2ι3 + 4) = -96 · (3ι3 - 1)2 ::; Ο .

Έστω ότι οι ευθείες AB, CD και EF ορίζουν το τρίγωνο ΑιC ιΕ ι και οι ευθείες BC,DE και FA ορίζουν το τρίγωνο ΒιDι Fι . (σχήμα 4). Έστω ακόμη ότι:

ΑΒ BC CD DE EF F Α -- = a -- = b -- = c -- = d -- = e και -- = f FιΒι

, ΑιC ι

, Β ιD ι ' C ι Ε ι ' D ι Fι Ε ιΑ ι .

Τότε, από την ομοιότητα των αντίστοιχων τριγώνων, λαμβάνουμε: Ε (ΑΒDι ) = a2Ε (ΒιD ι Fι ) , Ε (ΒCΕι ) = b2Ε (ΑιCιΕ ι ) , Ε ( CDFι ) = c2Ε (ΒιDι Fι ) , Ε (DΕΑι ) = d2Ε (ΑιCιΕ ι ) , Ε (ΕFΒι ) = e2Ε (ΒιD ι Fι ) , E ( FACι ) = f2Ε (ΑιCιΕ ι ) ,

ΕΥΚΛΕΙΔΗΣ Β ' 81 τ. l/30

Page 33: Ευκλειδης Β 81

-------- Μαθηματικοί Διαγωνισμοί - Μαθηματικές Ολυμπιάδες -------­

οπότε προκύπτουν οι ισότητες: 1 = E(ABCDE) = (1 - a2 - c2 - e2 )E(B1D1F1 ) (1)

1 = E(ABCDE) = (1 - b2 - d2 - f2 )E(A1C1E 1 ) (2) Α1

F

Σχήμα 4

Οι λόγοι b, d, f εκφράζονται συναρτήσει των λόγων a, c, e ως εξής (εξαιτίας της ομοιότητας των τριγώνων με πλευρές παράλληλες στις τρεις διευθύνσεις των πλευρών του εξαγώνου) :

ΑΒ ΑΒ DΙF; - -- · DΙFΙ - -- · DΙFΙ b = BC = D1F1 - D1B - F1C = B1F; B1F1 = D1F1 · (1 - a - c)

A1C1 A1E + EF + FC1 A 1E + e · D1F1 + FC1 A1E + e · D1F1 + FC1

F1C D 1F1 ( ) D1F; ( CD EF ) και Α E = - · ED = - F B - DF - ΕΒ = - · F B - -- · F B - - · ΕΒ I F D F B I I I I F B I I D Β I I D F I

I I I I I I I I I ( CD EF ) = D F · 1 - -- - - = D F (1 - c - e) I I D B D F I I

I I I I

και ομοίως FC1 = D1F1 (1 - a - c) , οπότε τελικά

b = D1F1 · (1 - a - c) = D1F1 · (1 - a - c) = -1_-_a_-_c_ A1E + e · D1F1 + FC1 D1F; · ( (1 - c - e) + e + (l - a - e) ) 2 - a - c - e

1 - c - e 1 - e - a και ομοίως d = f = ----2 - a - c - e ' 2 - a - c - e

Αν θέσουμε a + c + e = p , τότε έχουμε a2 + c2 + e2 :;::: .!.. p2 και 3

b2 d2 f2 _ 3 - 4p + p2 + a2 + c2 + e2 1 ( 3 - 2p

J2

+ + - > - --( 2 - p)2

-3 2 - p

Αν υποθέσουμε ότι E(A1C 1E 1 ) < i και E(B1D 1F1 ) < i , τότε από τις ( 1 ) και (2) παίρνουμε 2 2

' 3 2 2 2

3 2 2 2 αντιστοιχως 1 < 2 (1 - a - c - e ) και 1 < 2 (1 - b - d - f ) ,

1 1 1 2 1 1 ( 3 - 2p

J2 1 δηλαδή a2 + c2 + e2 < 3 και b2 + d2 + f2 < 3 , οπότε λαμβάνουμε 3p < 3 και3 2 _ p < 3 ,

που δε μπορεί να συναληθεύουν καθώς η πρώτη συνεπάγεται p < 1 , ενώ η δεύτερη 1 < p < 2 , (άτοπο).

ΕΥΚΛΕΙΔΗΣ Β' 81 τ.1/3 1

Page 34: Ευκλειδης Β 81

--------- Μαθηματικοί Διαγωνισμοί - Μαθηματικές Ολυμπιάδες

Επιτροπή Διαγωνισμών της Ε.Μ.Ε.

Από αυτό το τεύχος του ΕΥΚΛΕΙΔΗ Β ' , η Επιτροπή Διαγωνισμών της ΕΜΕ θα προτείνει στους μα­θητές ασκήσεις προς λύση επιπέδου μαθηματικών διαγωνισμών. Πιστεύουμε ότι οι ασκήσεις αυτές θα αποτελέσουν κίνητρο για τον μαθητή που αγαπάει τα Μαθηματικά και θέλει να συμμετάσχει στους μα­θηματικούς διαγωνισμούς. Πιστεύουμε ότι είναι μια καλή ευκαιρία να ασχοληθεί με τη λύση τους, ανα­ζητώντας παράλληλα, στην ευρύτερη βιβλιογραφία ή στο διαδίκτυο, υλικό για την καλύτερη θεωρητική του ενημέρωση . Στο επόμενο τεύχος θα δοθούν οι λύσεις των προτεινόμενων ασκήσεων, καθώς και άλ­λες ασκήσεις προς λύση . Στο παρόν τεύχος θα δώσουμε κάποιες εξαιρετικές λύσεις μαθητών στα προ­βλήματα της φετινής Ελληνικής Ολυμπιάδας Μαθηματικών «0 ΑΡΧΙΜΗΔΗΣ», καθώς και την πλήρη λύση του προβλήματος [ 1 ] , η οποία εκ παραδρομής δεν είχε περιληφθεί στο προηγούμενο τεύχος, όπως επίσης μια συμπλήρωση και μια άλλη λύση στο πρόβλημα [4] .

Την Επιτροπή Διαγωνισμών της Ελληνικής Μαθηματικής Εταιρείας (ΕΜΕ) πλαισιώνουν μαθηματι­κοί από όλη την Ελλάδα. Πρόεδρος της Επιτροπής είναι ο Αναπληρωτής Καθηγητής του ΕΜΠ Ανάργυ­ρος Φελλούρης και αντιπρόεδρος ο μαθηματικός Γιάννης Τυρλής. Μέλη της Επιτροπής είναι οι μαθη­ματικοί Αγγελική Βλάχου, Ευάγγελος Ζώτος, Ταμάρα Μτσεντλίτζε, Αλέξανδρος Παπαϊωάννου, Πα­ναγιώτης Πουλόπουλος και ο Ευάγγελος Ψύχας. Όλοι οι παραπάνω αποτελούν τον κεντρικό πυρήνα της Επιτροπής και ασχολούνται εκτός των άλλων και με την επιλογή των προβλημάτων των διαγωνισμών της ΕΜΕ. Επίσης συμμετέχουν τα μέλη των παραρτημάτων της ΕΜΕ: Γεώργιος Αποστολόπουλος, Αν­δρέας Βαρβεράκης, Δημήτριος Κοντοκώστας, Θεόδωρος Μπόλης, Κωνσταντίνος Παπαδόπουλος , Ανδρέας Πούλος και ο Αλέξανδρος Συγκελάκης. Επίσης, η Επιτροπή έχει συνεργάτες Ολυμπιονίκες παλαιότερων ετών, όπως ο Πέτρος Μπρέγιαννης και ο Σιλουανός Μπραζιτίκος. Την τεχνική υποστή­ριξη της Επιτροπής έχουν αναλάβει οι συνεργάτες της ΕΜΕ η Μαρία Γεωργούδη, ο Αθανάσιος Μαλα­φέκας και ο Στυλιανός Μαραγκάκης.

Βρείτε όλες τις συναρτήσεις f : ( Ο, +οο) � JR που ικανοποιούν την ισότητα

xy (3f (; ) + 5/ ( �)) = 3χ2 + 5y2 , για κάθε x, y ε IR .

Βρείτε όλες τις συναρτήσεις f : JR � JR που είναι γνησίως μονότονες και ικανοποιούν την ισότητα f (χ + f ( y ) ) = f (χ) + y, για κάθε χ, y ε JR .

Έστω Ρ( χ) πολυώνυμο με ακέραιους συντελεστές. Δίνεται ότι Ρ( α) - Ρ ( b) = 1 , για κάποι­ους ακέραιους a, b. Να αποδείξετε ότι οι ακέραιοι a, b διαφέρουν κατά 1 . ' ,,: _; , Να προσδιορισθούν όλα τα ζεύyη (mn) θετικών ακέραιωνπου είναι τέτοιοι ώστε: nl2m-1 και ml2n-1 .

Ένας θετικός ακέραιος ονομάζεται ισορροπημένος, αν ο αριθμός των ψηφίων του ισούται με τον αριθμό των διαφορετικών πρώτων παραγόντων του στην κανονική ανάλυσή του. Για παρά­δειγμα, ο αριθμός 2 1 = 3 · 7 είναι ισορροπημένος, ενώ ο αριθμός 25 = 52 δεν είναι. Να αποδείξε­τε ότι υπάρχει πεπερασμένος αριθμός ισορροπημένων ακέραιων.

Πάνω στις πλευρές CB και CD τετραγώνου ABCD θεωρούμε σημεία Μ και Κ, αντίστοι-χα, έτσι ώστε η περίμετρος του τριγώνου CΜΚ να ισούται με το διπλάσιο της πλευράς του τε­τραγώνου. Βρείτε τη γωνία LMAK .

- - - - Δίνεται τρίγωνο ABC με έκκεντρο Ι και περίκεντρο Ο τέτοια ώστε ΟΙ l_ ΑΙ . Έστω Α1 το δεύτερο σημείο τομής της ευθείας ΑΙ με τον περιγεγραμμένο κύκλο του τριγώνου ABC . Έστω ακόμη D το ση­μείο τομής των ευθειών ΒΑ1 και AC , και Ε το σημείο τομής των ευθειών CA, και ΑΒ . Να αποδείξετε ότι το περίκεντρο του τριγώνου ADE βρίσκεται πάνω στον περιγεγραμμένο κύκλο του τριγώνου ABC .

ΕΥΚΛΕΙΔΗΣ Β ' 81 τ. l/32

Page 35: Ευκλειδης Β 81

-------- Μαθηματικοί Διαγωνισμοί - Μαθηματικές Ολυμπιάδες --------Συμπληρωματικές λύσεις Π ΡΟ ΒΛΗ ΜΑΤΩ Ν « 0 ΑΡΧ Ι Μ Η Δ Η Σ » 201 1

Να λύσετε στους ακέραιους την εξίσωση x3y 2 ( 2y - χ) = x2y4 - 36 .

Μετά τις πράξεις διαπιστώνουμε ότι η δεδομένη εξίσωση είναι ισοδύναμη με την εξίσωση χ2/ (χ - y)2 - 62 = Ο, x, y Ε Ζ <=> [ xy (x - y) - 6} [ xy (x - y) + 6] = Ο, x, y Ε Ζ,

<=> xy (χ - Υ) = 6, χ, y Ε Ζ, ή xy (χ - y) = -6, χ, y Ε Ζ. <=> xy (x - y) = 6, x, y Ε Ζ, (1) ή xy (y - x) = 6, x, y Ε Ζ. (2)

Από τη μορφή των ( 1) και (2) προκύπτει ότι, αν (x0 , y0 ) είναι λύση της ( 1) , τότε το ζευγάρι (y0 , x0 ) είναι λύση της (2) και αντιστρόφως. Επομένως, αρκεί να λύσουμε μόνον την εξίσωση ( 1). Επειδή x, y Ε Ζ , η εξίσωση ( 1) είναι ισοδύναμη με :

{xy = 6, χ - y = 1} (Σ 1 ) ή {xy = -6, χ - y = - 1} (Σ2 ) ή {χy = 3, χ - y = 2} (Σ3 ) ή {xy = -3, χ - y = -2} (Σ4 ) ή {χy = 1, χ - y = 6} (Σ5 ) ή {xy = - 1 , χ - y = -6} (Σ6 ) ή {χy = 2, χ - y = 3} (Σ7 ) ή {xy = -2, χ - y = -3} (Σ8 ) .

Από τα 8 συστήματα μόνον τα (Σ 1 ) , (Σ3 ) , (Σ8 ) δίνουν τις ακέραιες λύσεις:

( x, y) = (3, 2) , ( x, y) = ( -2, -3) , ( x, y) = (3, 1) , ( x, y) = (-1, -3) , ( x, y) = ( -2, 1) και ( x, y) = ( -1, 2) . Σύμφωνα με όσα είπαμε παραπάνω, η εξίσωση (2) έχει στους ακέραιους και τις λύσεις

( x, y) = (2, 3) , ( x, y) = ( -3, -2) , ( x, y) = ( 1 , 3) , ( x, y) = ( -3, - 1 ) , ( x, y) = ( 1 , -2) και ( x, y) = (2, -1 ) .

Αν a, b, c είναι θετικοί πραγματικοί αριθμοί με άθροισμα 6, να προσδιορίσετε τη μέγιστη

τιμή της παράστασης: S = �a2 + 2bc + �b2 + 2ca + �c2 + 2ab .

Λύση με ανισότητα Holder (Π. Λώλας, Γ. Βλάχος, Μ. Αγγελής)

'Εχουμε �9 (α + b + c)2 = � ·�( α2 + 2bc ) + ( b2 + 2ca ) + ( c2 + 2αb ) =�1+ 1+ 1 ·ψ d +2bc) + ( b2 +2ca) + ( c2 +2ab) ?:.�d +2bc+�b2 +2ca+�c2 +2ab =S, (ανισότητα Holder).

Επομένως έχουμε S � �9 · 62 = 3� , όπου η ισότητα ισχύει όταν α2 + 2bc =b2 +2ca = c2 +2αb, α+b+c = 6 <=> ( α-b) ( α +b-2c) = 0, (b -c) (b + c -2α) = 0, α + b+c = 6 <=> (α-b) ( 6 -3c) = O, (b -c) ( 6-3α) = 0, α+b+c = 6 <=>α = b = c = 2.

Λύση με ανισότητα Cauchy - Schwarz (Χ. Τσαμπασίδης)

Θέτουμε χ = � α2 + 2bc , y = � b2 + 2cα , z = � c2 + 2αb , οπότε είναι χ, y, z > Ο και από την ανισότητα Cauchy - Schwartz έχουμε

( 13 + 13 + 13 ) ( 13 + 13 + 13 ) (χ3 + / + z3 ) ?:. ( 1 · 1 · x + l · l · y + l · l · z )3 <=> 9 (χ3 + / + z3 ) ?:. (x + y + z)3 • Όμως έχουμε χ3 + / + z3 = α2 + 2bc + b2 + 2ca + c2 + 2αb = (α + b + c ) 2 = 36 , οπότε η τελευταία

ανισότητα γίνεται χ + y + z � �9 ( χ3 + y3 + z3 ) = �9 · 36 = 3�. Η ισότητα ισχύει, όταν

χ = y = z, α + b + c = 6<::::> α2 + 2bc = b2 + 2cα = c2 + 2αb, α + b + c = 6 <=> α = b = c = 2. Λύση με ανισότητα Jensen (Κ. Αξιώτης, Ν Αθανασίου, Δ. Σωτηρίου)

Θεωρούμε τη συνάρτηση f (χ) = �' χ > Ο, η οποία είναι δύο φορές παραγωγίσιμη στο

ΕΥΚΛΕΙΔΗΣ Β ' 81 τ.l/33

Page 36: Ευκλειδης Β 81

-------- Μαθηματικοί Διαγωνισμοί - Μαθηματικές Ολυμπιάδες --------1 � 2 � 1 Γ (Ο, +οο ) με f' (x) = )x 3 και f' (χ) = -9χ 3 . Επομένως η συνάρτηση f (χ) = {ιχ , είναι κοίλη

στο διάστημα ( Ο, +οο ) , οπότε από την ανισότητα του Jensen λαμβάνουμε

Η ι ( α' + 2bc ) + ι ( b' + 2cα ) + ι ( c' + 2αb )J � ι ( α' + 2bc + b' +32cα + c' + 2αb J

=> !(α' + 2bc ) + ! ( b' + 2cα) + ! ( c2 + 2αb) � 3f ( ( α +�+ c ) ' J = 3/ ( 1 2 ) = 3�

� �α2 + 2bc + �b2 + 2cα + �c2 + 2αb � 3:ifli. Επειδή για α = b = c = 2 το πρώτο μέλος γίνεται ίσο με 3:ifϊ2 έπεται ότι η μέγιστη τιμή της

παράστασης S είναι 3:ifϊ2 . Λύση με ανισότητα δυνάμεων (Α. Μουσάτοβ)

Θέτουμε χ = Jα2 + 2bc , y = Jb2 + 2ca , 2 = Jc2 +2αb , οπότε έχουμε χ + y + 2 = ( α + b + c ) 2 = 36 . Άρα, αν θέσουμε u = �, ν = ifY, w = � , τότε έχουμε S = u + ν + w και από την ανισότητα

u3 + ν3 + w3 των δυνάμεων προκύπτει S = u + ν + w � 3 · 3 = 3 · :if1i, 3 αφού ισχύει ότι u3 + ν3 + w3 = χ + y + z = 36 . Η ισότητα ισχύει όταν u = ν = w <::::> α = b = c = 2.

Λύση με ανισότητα αναδιάταξης (Κ. Δερμεντζής)

Θέτουμε χ = � α2 + 2bc , y = � b2 + 2cα , 2 = � c2 + 2αb , οπότε έχουμε S = χ + y + z , χ3 + y3 + 23 = (α + b + c ) 2 = 36 .

Στη συνέχεια θέτουμε Sι = Χ3 + / + 23 = χ2χ+ /y+222, s2 = x2y + /2 +z2x, s3 = x2z + /x+22y και επειδή οι τριάδες ( χ2 , /, z2 ) , (χ, y, 2 ) διατάσσονται ομοίως, από την ανισότητα της αναδιά­

ταξης λαμβάνουμε S1 :2': S2 και S1 :2': S3 . Επιπλέον, αφού χ, y, 2 > Ο ισχύει S1 =χ3 + / + 23 :2-: 3.xyz , οπότε έχουμε S3 = χ3 + / + 23 + 3 ( χ2 y + / z + 22 χ) + 3 ( χ2 2 + /χ + 22 y) + 6xy2 = = S1 + 3S2 + 3S3 + 6xyz � S1 + 3S1 + 3S1 + 2S1 = 9S1 . Άρα έχουμε

S = χ + y + 2 � �9 ( χ3 + / + 23 ) = �./9 · 1 2 = 3 · :ifli. Επειδή για α = b = c = 2 η παράσταση S λαμβάνει την τιμή 3:ifϊ2 , έπεται ότι η μέγιστη τιμή

της παράστασης είναι 3:ifϊ2 .

Δίνεται οξυγώνιο τρίγωνο ABC ( με ΑΒ < AC ), εγγεγραμμένο σε κύκλο c(O, R) (με κέ­

ντρο το σημείο Ο και ακτίνα R ). Η προέκταση του ύψους AD τέμνει τον περιγεγραμμένο κύκλο στο σημείο Ε και η μεσοκάθετη (μ) της πλευράς ΑΒ τέμνει την AD στο σημείο

L . Η BL τέμνει την AC στο σημείο Μ και τον περιγεγραμμένο κύκλο c(O, R) στο σημείο

Ν . Τέλος η ΕΝ τέμνει τη μεσοκάθετη (μ) στο σημείο Ζ . Να αποδείξετε ότι:

ΜΖ l. BC <::::> ( CA = CB ή Ζ Ξ Ο) , δηλαδή ότι "η ΜΖ είναι κάθετη στην BC , αν, και

μόνο αν, το τρίγωνο ABC είναι ισοσκελές με CA = CB ή το σημείο Ζ ταυτίζεται με το κέ­ντρο Ο του περιγεγραμμένου κύκλου c(O, R) ".

Ευθύ. 'Εστω ΜΖ l_ BC. Τότε ΜΖ 1 1 AD , αφού AD l_ BC . Επομένως έχουμε: ZMC = EAC (ε-

ΕΥΚΛΕΙΔΗΣ Β' 81 τ. l/34

Page 37: Ευκλειδης Β 81

--------- Μαθηματικοί Διαγωνισμοί - Μαθηματικές Ολυμπιάδες

ντός εναλλάξ στις παράλληλες ΜΖ, AD , με τέμνουσα την Α C )

Ν Όμως, από το εγγεγραμμένο τετράπλευρο EANC έ-Λ Λ

χουμε: EA C = ENC , οπότε λαμβάνουμε τις ισότητες: ZMC = EA C = ENC = ZNC . Επομένως το τετράπλευ­ρο MNCZ είναι εγγράψιμο, οπότε θα είναι

"' ,.._ Λ Λ Λ ,.._

ACZ = MCZ = ΜΝΖ = ΒΝΕ . Όμως ΒΝΕ = ΒΑΕ , από το εγγεγραμμένο τετράπλευρο ΑΒΕΝ . Επομένως έχουμε

Λ Λ Λ

ACZ = ΒΑΕ = BAD . ( Ι ) Από το ορθογώνιο τρίγωνο ABD έχουμε ότι Λ Λ Λ

BAD = 90° - ABD = 90° - ABC , οπότε λαμβάνουμε Λ Λ

την ισότητα ACZ = 90° - ABC . (2) Επειδή το σημείο Ο είναι το περίκεντρο του οξυγώνιου τριγώνου ABC θα

Λ Λ

είναι A CO = 90° - ABC . (3) Από τις (2) και (3) λαμβάνουμε ότι Σχήμα Ι

ACZ = 90° - ABC = ACO και επειδή τα σημεία Ζ και Ο βρίσκονται στο ίδιο ημιεπίπεδο ως προ; την AC , έπεται ότι τα σημεία C, Ο και Ζ είναι συνευθεια-κά. Έτσι διακρίνουμε τις περιπτώσεις :

•· Ο = Ζ , οπότε ισχύει το ζητούμενο ·� Ο;/. Ζ , οπότε η ευθεία ΟΖ είναι η μεσ ο κάθετη της πλευράς ΑΒ και αφού το σημείο C ανήκει

στην ΟΖ θα είναι CA = CB .

/,, .,., � σ- ι- ρυφ; .• , Αν είναι CA = CB ή Ο = Ζ , τότε τα σημεία C, Ο και Ζ είναι συνευθειακά, αφού ανή-Λ Λ Λ

κουν στη μεσοκάθετη της πλευράς ΑΒ . Άρα έχουμε : Α CZ = Α CO = 90° - ABC =

= BAD = ΒΑΕ = ΒΝΕ = AJiιz => MCZ = AJiιz =>το τετράπλευρο MNCZ είναι εγγράψιμο, οπότε θα έχουμε ZMC = ZNC = ENC = EAC . Άρα θα είναι ΖΜ 1 1 ΑΕ και αφού ΑΕ l_ BC , έπεται ότι ΖΜ l_ BC .

. [Συμπλήρωση στη λύση του [4] από το προηγούμενο τεύιος 80]

Β

Σχήμα 5

ι ι ι

·---._L--------Σχήμα 6

c

Αν το τρίγωνο ABC είναι ισοσκελές με CA = CB και C = ψ = 45° , τότε τα τρίγωνα ΤΜΝ , ΤΜΑ και ΑΜΝ είναι ορθογώνια και ισοσκελή . Το τετράπλευρο ABCN είναι ισοσκελές τραπέζιο. Άρα η τΜ είναι μεσοκάθετη της BC . Στη περίπτωση αυτή και το σημείο Ζ ταυτίζεται με το σημείο Ο , ο­πότε η διάζευξη των προτάσεων ( CA = CB ή Ζ = Ο) είναι εγκλειστική, σχήματα 5 και 6 .

ΕΥΚΛΕΙΔΗΣ Β ' 81 τ. Ι/35

Page 38: Ευκλειδης Β 81

Η Homo Mathematicus είναι μια στήλη στο περιοδικό μας, με σκοπό την ανταλλαγή απόψεων και την ανάπτυξη προβληματισμού πάνω στα εξής θέματα: Ι ) Τι είναι τα Μαθηματικά, 2) Πρέπει ή όχι να διδάσκονται, 3) Ποιοι είναι οι κλάδοι των Μαθηματικών και ποιο το αντικείμενο του καθενός, 4) Ποιες είναι οι εφαρμογές τους, 5) Ποιες επιστήμες ή κλάδοι επιστημών απαιτούν καλή γνώση των Μαθηματι­κών για να μπορέσει κάποιος να τους σπουδάσει.

Για τους συνεργάτες της στήλης: παράκληση ! τα κείμενα της στήλης αυτής, ως προς το περιεχό­μενό τους και ως προς το επίπεδό τους, θα πρέπει να είναι συμβιβαστά με τα ενδιαφέροντα και το επίπε­δο κατανόησης από μέρους των παιδιών.

«Τα Μαθηματικά έχουν τη φήμη ότι είναι ένας ξεκάθαρος, αδιαμφισβήτητος και χωρίς περαιτέρω προβλήματα επιστημονικός κλάδος, το άκρως α­ντίθετο (από αυτήν την άποψη) της φιλοσοφίας. Εδώ τα πράγματα φαίνεται να έχουν τακτοποιηθεί, μια για πάντα, στη βάση συνηθισμένων και αναμε­νόμενων δραστηριοτήτων. Συμβαίνει, όμως, αυτό; Δεν υπήρξαν κάποιες επαναστάσεις στα Μαθημα­τικά όπου μακροχρόνιες πεποιθήσεις εγκαταλήφ­θηκαν; Ας θεωρήσουμε το βάθος των Μαθηματι­κών που χρησιμοποιούνται και απαιτούνται στις φυσικές και κοινωνικές επιστήμες.

Πως είναι δυνατόν τα Μαθηματικά, που εμφα­νίζονται πρωταρχικά ως μια διανοητική δραστη­ριότητα, να φωτίζουν τον φυσικό, τον ανθρώπινο και τον κοινωνικό κόσμο που μελετά η επιστήμη ; Γιατί, η βαθιά κατανόηση του κόσμου (με επιστη­μονικούς όρους) προϋποθέτει μια εξίσου βαθιά κατανόηση πολλών Μαθηματικών; Τι συμπεραίνουμε για τα Μαθηματικά από το γεγονός αυτό; Τι μας λέει αυτό για τον φυσικό, τον ανθρώπινο και τον κοινωνικό; . . . »

[ΠΗΓΗ: «ΣΚΕΨΕΙΣ ΓΙΑ Τ Α ΜΑΘΗΜΑ τΙΚΑ», Stewart Shapiro, Επιστημονική Επιμέλεια: Κώστας Αθ. Δρόσος, Εκδ. Πανεπιστημίου Πατρών, Πάτρα, 2006 ]

Τι είναι η Μαθηματική Ψυχολογία; [η απάντηση στο τέλος της στήλης]

«Καρδιοειδείς για συλλέκτες μανιταριών. Μαθήματα προσανατολισμού», [β 'μέρος]

υπενθυμίζουμε ότι το κείμενο τούτο είναι η συνέχεια του κειμένου που δημοσιεύτηκε στο προηγούμενο τεύχος του περιοδικού μας. Αποτελεί, τη δεύτερη και τελευταία συνέχεια του άρθρου.

« . . . Ας κάνουμε, για παράδειγμα, μια αριθμητική προσέγγιση για t =2h και ν= 2 km/h. Σε αυτή την περίπτωση, θ = 5π/6, ενώ η ακτίνα R και η απόσταση r είναι 8 km και 2,4 km, αντίστοιχα .

. -...... , '\

' i • •

\,, _ ___ /;'--� .. � � .. '.ι \ ί � ......... �;

( ,

Το Σχήμα 2 μας επιτρέπει να παραστήσουμε γραφικά τη θέση του τελικού σημείου Α" . Ας υποθέ­σουμε ότι την αρχική στιγμή οι κύκλοι Ο και Ο' εφάπτονται στο σημείο Α. Τότε, ο κύκλος Ο' αρχίζει να

ΕΥΚΛΕΙΔΗΣ Β' 81 τ.l/36

Page 39: Ευκλειδης Β 81

------------- ΗΟΜΟ MATHEMAτiCUS ------------­

κυλά χωρίς ολίσθηση κατά μήκος του κύκλου Ο. Τη χρονική στιγμή t, όταν οι κύκλοι εφάπτονται στο σημείο καμπής Α', το αρχικό σημείο επαφής Α βρίσκεται στη θέση Α" . Επομένως, το σύνολο όλων των δυνατών τελικών σημείων για διαδρομές σταθερής ταχύτητας ν και διαφορετικής διάρκειας t συμπίπτει με την τροχιά ενός σημείου του κύκλου Ο' που κυλά επί του κύκλου Ο.

Η αντίστοιχη καμπύλη ονομάζεται καρδιοει- κτικά μια πορεία τριών τμημάτων (Σχήμα 4β). Αυ­δής - το διάγραμμά της παρουσιάζεται στο Σχή- τή η στρατηγική διατηρεί το κύριο πλεονέκτημα, μα 3 - ενώ η εξίσωσή της περιγράφηκε προηγου- δηλαδή τον προσανατολισμό βάσει μόνο του Ήλι­μένως. Αυτή η καμπύλη μπορεί να κατασκευαστεί ου . με αυτοσχέδια όργανα, και το αντίστοιχο διάγραμ- Ολοκληρώνοντας θα ήθελα να επισημάνω ότι μα μπορεί να φανεί χρήσιμο στο δάσος όταν δεν η πιο σύνθετη περίπτωση όπου η αναχώρηση και η υπάρχει χάρτης ή ως συμπλήρωμα του χάρτη . επιστροφή γίνονται με διαφορετική ταχύτητα μπο-

Το σημείο C του Σχήματος 3 είναι το τελικό ρεί να αναλυθεί με παρόμοιο τρόπο. Μπορούμε σημείο όταν t=6h και η αντίστοιχη τροχιά παρου- εύκολα να δείξουμε ότι όλα τα τελικά σημεία θα σιάζεται στο Σχήμα 4α. ανήκουν στην ευθεία ΑΆ" . Η μεγαλύτερη ταχύτη-

Σε αυτή . την περίπτωση η αζιμουθιακή από- τα επιστροφής αυξάνει μόνο την απόσταση από το κλιση είναι μηδέν, αλλά για τις χαρακτηριστικές αρχικό σημείο Α. τιμές των παραμέτρων r= 1 6 km. Επομένως, ένας Μπορούμε επομένως από την ανάλυσή μας να ολοήμερος περίπατος (2t = 1 2 h) με τη στρατηγική συμπεράνqυμε ανεπιφύλακτα ότι, πηγαίνοντας στο «από τον Ήλιο, προς τον Ήλιο» θα είναι προφανώς δάσος, καλό είναι να έχουμε μαζί μας πυξίδα ! ;» ατυχής ή ακόμη και επικίνδυνος, όμως, για περί- [Πηγή: QUANTUM, τόμ. 6, τ. 5, εκδ. ΚΑΤΟΠΓΡΟ, Αθήνα 1999) πατο αυτής της διάρκειας θα προτείναμε εναλλα-

1 Μια χρήσιμη μετρική σχέση ι..._ __ ___,J Από to φίλο της στήλης Τηλέμαχο Μπαλτσαβιά (Κεφαλονιά) λάβα­με και ευχαρίστως δημοσιεύουμε ένα σημείωμα στο οποίο αναδεικνύει (και αποδεικνύει) το θεώρημα του Leibniz. Σε ορισμένες περιπτώσεις, η στήλη μας, φιλοξενεί και τέτοια θέματα.

Το Θεώρημα του Leibniz αποτελεί μία πολύτιμη μετρική σχέση για όσους ασχολούνται με πιο δύσκολα θέματα από τα συνήθη σχολικά της Ευκλείδιας Γεωμετρίας.

Το εν λόγω θεώρημα δε διδάσκεται στα σχολεία μας, ωστόσο εδώ θα δοθεί μια απόδει ξη με διανύσματα, εύκολα κατανοητή από κάπ οιον που έχει παρακολουθήσει το πρώτο κεφά­λαιο από τα Μαθηματικά Κατεύθυνσης της Β' Λυκείου.

Η διατύπωση του θεωρήματος είναι η εξής: <<Δίνεται τρίγωνο ABC , και έστω G το βαρύ­κεντρό του. Αν Μ είναι ένα τυχόν σημείο, τότε:

( ΜΑ)2 + ( ΜΒ)2 + ( MC)2 = 3( MG)2 + ( GA)2 + ( GB)2 + ( GC)2 » -2 -2 -2 - - - - - -

Η απόδειξη είναι η εξής : ΜΑ +ΜΒ + MC = (MG + GA)2 + (MG + GB)2 + (MG + GC)2

= 3MG2 + GA

2 + GB2 + GC

2 + 2MG (GA + GB + GC) - - -Όμως GA -+' GB + GC = Ο , όπως είναι γνωστό, και η απόδειξη τελειώνει εδώ.

I Μια πρόταση στα κανονικά πολύγωνα ,___ __ ____, Στο προηγούμενο τεύχος προαναγγείλαμε πως θα δημοσιεύσουμε ένα θέμα, που είναι έργο των Νίκου Μαλάμογλου* και του Παν. Οικονομάκου. Υπενθυμίζουμε πως ο Ν. Μαλάμογλου είναι από τα τρία αρχαιότερα, εν ζωή, μέλη της ΕΜΕ (οι άλλοι δύο είναι οι : Γιώργος Ω­ραιόπουλος και Γιώργος Ιωαννίδης).

Σε κάθε κανονικό πολύγωνο με Ν�5 εκείνες οι διαγώνιες (που ως χορδές του περιγεγραμμένου κύκλου, υποτείνουν τόξο διπλάσιο του τόξου που υποτείνει η πλευρά του),τεμνόμενες ανά δύο δια­δοχικά, σχηματίζουν κανονικό πολύγωνο όμοιο προς το αρχικό.

Εάν α1 ,α2 ,α3 , . . • ,αν, . . . είναι αντίστοιχα τα μήκη των πλευρών της ακολουθίας π 1 ,π2 , • • • ,Πγ, . . . των ομοίων κανονικών πολυγώνων της πρότασης (α), τότε η ακολουθία (αν) είναι μία φθίνουσα γεωμ. πρόοδος.

ΕΥΚΛΕΙΔΗΣ Β' 81 τ.l/37

Page 40: Ευκλειδης Β 81

------------- ΗΟΜΟ MATHEMAτiCUS -------------Α;η'ιi'ϊ ψ>η

Σημείωση : χωρίς να βλάπτεται η γενικότητα, η απόδειξη που θα ακολουθήσει, αναφέρεται σε κανο­νικό πεντάγωνο.

Έχω: τριγΑιΓ2Ε ι=τριγΑιΔ2Β ι , άρα: ΑιΓ2=Ε ιΓ2=Β ιΔ2=ΑιΔ2=Χι άρα: Γ2Δ2=Ε ιΒ ι-(Ε ι Γ2+Β ιΔ2 ) ή Γ2Δ2=δ 1 -2χ1 • Ομοίως, όλες οι πλευρές του

Α2Β2Γ2Δ2Ε2 είναι ίσες με δ 1 -2χ, , δηλ. έχουμε: α2= δ 1 -2χ1 Τα τρίγωνα Ε ιΑ ιΒ ι και Ε 1Α ι Γ2 είναι ισοσκελή

με κοινή γωνία της βάσης γωνΕ 1=γωνφ, τ ψ : : on

Από την ομοιότητα των τριγώνων Ε 1Α 1Β 1 και

Ε,Α, Γ2 , προκύπτει ότι � = � <::::> χ α,

2 Χ = � άρα θα έχουμε δι

α2 α = δ - 2-1 2 I δ I δ2 - 2α2

Άρα θα έχουμε: α2 = 1 1 δι

'Ε λ , α2 , α2 δ� - 2α � τσι, ο ογος - γινεται - = ----'------'--

α1 αι αιδ ι (i)

Στο τρίγωνο Α 1Ε 1Β 1 (από το θεώρημα των ημιτόνων), έχουμε (αν θέσουμε: γωνΕ 1Α 1Β 1=ω και γωνΕιΑ ιΓ2=φ):

� = α ι <::::> _δ_ι_ = _α_ι_ <::::> ημω ημ(90° - ;) ημω συν;

ω <::::> δ1 = 2α1ημω-2

(ίί) Απ' τις (ί), (ii) έχουμε (επειδή ω>90°) :

2 (ημ ω )2 - 1 � = 2 = -συνω

> Ο ω ω ημ- ημ-2 2

α Απομένει να δείξουμε ότι : -2 < 1 <::::>

Πράγματι έχουμε: α2 < 1 α ι

α ι

2 (ημ ω )2 - 1 2 2 ω ω -----'-----'--- < 1 <::::> 2ημ - - ημ- - 1 < ο . ω 2 2 ημ-2

Η τελευταία ανισότητα ισχύει, γιατί οι ρίζες του τριωνύμου φ:φ(χ)=2χ2-χ- 1 είναι οι χ1= 1 και

Χ2 = _ _.!._ . Και επειδή 90°<ω< 1 80° ή 45°< ω <90° 2 2

άρα είναι όμοια οπότε γωνΓ2=γωνΑ1=ω Για τον ίδιο λόγο θα έχουμε

γωνΔ2=γωνΕ2=γωνΑ2=γωνΒ2=ω, δηλαδή , στο πε­ντάγωνο Α2Β2Γ 2Δ2Ε2 αποδείξαμε πρώτα ότι όλες οι πλευρές του είναι ίσες με μήκος δ 1 -2χ1 και μετά όλες οι γωνίες του ίσες με μέτρο ω. Άρα αυτό είναι κανονικό πεντάγωνο και όμοιο προς το ΑιΒ ιΓ ιΔ ιΕ ι .

, J2 ω 1 , ( ω ) , ξ ' η l < ημl < , αρα το ημ l ειναι μετα υ των

ριζών του τριωνύμου φ. Αι

Δ ι α ι Γ ι

Άρα το πρόσημο αυτού για χ =ημ( �) θα είναι

ετερόσημο του 2 . Έτσι θα έχουμε (συνεχίζοντας) : α2 = � = . . . = -συνω = λ με Ο <

-συνω < 1

α 1 α2 ημ ω ημ ω 2 2

Συνεπώς η ακολουθία: α1 ,α2,α3 , • • • ,αν, . . . εί--συνω --λ ναι μία φθίνουσα Γ.Π. με λόγο τα δε ω ημ-

2 εμβαδά των κανονικών πολυγώνων

Π1 ,Π2,Π3 , • . • ,Πν, . . . αποτελούν φθίνουσα Γ.Π. , , συν2ω με λογο λ1 ισο με ---

2 ω η μ -2

Άρα το άθροισμα των εμβαδών των απείρων τούτων πολυγώνων θα είναι:

εφ2 ω ε ι 2 Σ =

1 - λ ι = . . . = 2εφ2 ω - 1 �

2 ε 1 όπου ε 1 το εμβαδό του αρχικού πολυγώνου Π ι

ΕΥΚΛΕΙΔΗΣ Β ' 81 τ.Ι /38

Page 41: Ευκλειδης Β 81

------------- ΗΟΜΟ MATHEMAτiCUS ------------­

Η δημιουργική ηλικία των μαθηματικών Ο φίλος της στήλης Δημητρίου Βαγγέλης [Σέρρες] μας έστειλε το πα­

ρακάτω κείμενο. Τον ευχαριστούμε. Λόγω έλλειψης χώρου, το δημοσιεύουμε σε δύο συνέχειες «Για τους μαθηματικούς λέγεται

ότι, περισσότερο ακόμη και από τους άλλους επι­στήμονες, μπορεί να είναι δημιουργικοί ως ερευ­νητές, μόνο στην πρώτη νεότητά τους. Όπως εί­δαμε ακόμη και πρόσφατα, βρέθηκαν κάποιοι βιο­λόγοι που προσπάθησαν να στηρίξουν αυτή την τρέχουσα αντίληψη πάνω σε δεδομένα που προέρ­χονται από την ανάπτυξη του κεντρικού νευρικού συστήματος και σε όσα αρχίσαμε να γνωρίζουμε για τη γήρανση του εγκεφάλου. Ωστόσο, παρά τις προόδους που επιτεύχθηκαν τα τελευταία χρόνια και τις προοπτικές που διανοίχθηκαν από τη νευ­ροβιολογία της ανάπτυξης, οι γνώσεις αυτές παρα­μένουν ακόμη πολύ αποσπασματικές, και δεν μου φαίνεται ότι έχουν προσκομιστεί έως σήμερα απο­δείξεις για συσχετισμούς ανάμεσα σε τούτο το κοινωνικό δεδομένο και σε κάποιο βιολογικό υπό­βαθρο .

ι.__ __ __JI Οφειλόμενες εξηγήσεις Στο προηγούμενο τεύχος [Νο 80] , η στήλη μας

φιλοξένησε το γραπτό, του συναδέλφου Βασίλη Χρ. Σαμαρά, με τίτλο «Η κρυφή γοητεία των Μα­θηματικών». Ο συνάδελφος μαθηματικός Βένιος Αγγελόπουλος μας ειδοποίησε πως μεγάλο μέρος του άρθρου αυτού ήταν, αυτολεξεί μεταφορά κει­μένου, από το βιβλίο του «Θέλουμε Παιδεία;», απ' όπου, όμως, έλλειπαν: α) βιβλιογραφική αναφορά

Α ναμφισβήτητα, η ιστορία των Μαθηματικών είναι πλούσια σε πρώιμες ιδιοφυΤες. Ο Eνariste Galois, που το θαυμάσιο έργο του (το οποίο συνι­στά αποφασιστική καμπή για τα σύγχρονα Μαθη­ματικά) διακόπηκε τραγικά και ανόητα σε ηλικία είκοσι ετών, αποτελεί σίγουρα το πιο εξέχον παρά­δειγμα. Όσο συναρπαστικές κι αν φαίνονται όμως παρόμοιες περιπτώσεις, δεν θα πρέπει να μας κά­νουν να λησμονούμε ότι υπάρχουν επίσης και α­ντίστροφα παραδείγματα. Έχουμε δει μαθηματι­κούς που η δημιουργικότητά τους αναπτύχθηκε με σχετική χρονική καθυστέρηση, ενώ σε άλλους η δημιουργικότητα της νιότης τους προεκτάθηκε και στην ωριμότητα»

[πηγή : «Η ΙΣΧΥΣ ΤΩΝ ΜΑΘΗΜΑ τΙΚΩΝ», Moshe Flato, εκδ . ΚΆΤΟΠΤΡΟ, Αθήνα 1 993]

και β) τα σύμβολα - εισαγωγικά που έπρεπε να περικλείσουν τα εδάφια - που δανείστηκε. Εμείς γνωστοποιήσαμε το συμβάν στον Β . Χ. Σαμαρά ο οποίος με μεγάλη προθυμία, μας έστειλε μήνυμα, στο οποίο εξηγεί πως, εκ παραδρομής έγινε ό,τι έγινε και ζητά συγγνώμη, για τη δυσαρέσκεια που προκλήθηκε στον Β. Αγγελόπουλο και στους ανα­γνώστες της στήλης.

«Η "Μαθηματική Ψυχολογία" είναι ένας συνοριακός κλάδος μεταξύ των Μαθηματικών και της Ψυ­χολογίας, που μελετά μοντέλα, κυρίως στοχαστικά, των διαφόρων φαινομένων και συστημάτων, που εμφανίζονται στην Ψυχολογία και ειδικότερα στην Πειραματική Ψυχολογία, Κλινική Ψυχολογία και στην Κοινωνική Ψυχολογία»

[πηγή : Σοφ. Καλπαζίδου - Χρυσ. Γκανάτσιου «ΣΤΟΙΧΕΙΆ ΜΑΘΗΜΑτΙΚΗΣ ΨΥΧΟΛΟΓΙΑΣ», εκδόσεις ΖΗΤΗ, Θεσσαλονίκη 1 996]

'

........... ,... ,'-

ΝΙΚ ΟΣ ΜΑΛΑΜΟΓ ΛΟΥ

Επί του τυπογραφείου* : Ενώ το περιοδικό ήταν να φύγει για έκδοση ήρθε η είδηση , ότι έφυγε πλήρης ημερών, ο συνάδελφος Μαθηματικός, Νίκος Μαλάμογλου [ 1 0/8 I 20 1 1 ] . Ο εκλιπών γεν­νήθηκε στην Μάδυτο [ 1 7/05/ 1 9 1 4] της Ανατολικής Θράκης και υπήρξε δραστήριο μέλος της μαθηματικής κοινότητας και της Ε.Μ.Ε. [ 1939) , σε όλες τις δραστηριότητες της, μέχρι και σήμερα .

Ενδεικτική είναι η άσκηση , που δημοσιεύουμε σε αυτό το τεύ­χος, μαθηματικής ανησυχίας και προβληματισμού σε γενικότερα θέματα Γεωμετρίας.

Στην επιμνημόσυνη δέηση , μεταξύ άλλων παρευρέθησαν από την Ε.Μ.Ε. και κατέθεσαν στεφάνι, ο Πρόεδρος της, Γρηγόριος Καλογερόπουλος και ο Α· Αντιπρόεδρος Γεώργιος Δημάκος.

ΕΥΚΛΕΙΔΗΣ Β' 81 τ. l/39

Page 42: Ευκλειδης Β 81

Α ' ΛΥ ΚΕΙΟΥ Θύμιος Διαμαντόπουλος

Δίνεται το σύνολο Α = {1, { 2, 3} ,0, 3} . Ποια από τα παρακάτω είναι σωστά και ποια είναι λάθος:

2 ε Α {1, 2} ς Α {0, 3} ς Α Ν(Α) = 5 { 2, 3} ς Α

Είναι φανερό ότι το 2 δεν είναι στοιχείο του συνόλου Α, άρα το 2 ε Α είναι λάθος.

Το Ι είναι στοιχείο του Α αλλά το 2 δεν είναι στοιχείο του Α, άρα το { 1 , 2} ς Α είναι λάθος.

Το σύνολο {0, 3} περιέχει δύο στοιχεία του Α, οπότε είναι υποσύνολο του Α, δηλαδή, ο συμβολισμός

{0 , 3} ς Α είναι σωστός.

Το σύνολο Α περιέχει τέσσερα στοιχεία και όχι πέντε. Άρα το Ν(Α) = 5 είναι λάθος.

Το σύνολο { 2, 3} είναι στοιχείο του Α και όχι υποσύνολο του Α, άρα το { 2, 3} ς Α είναι λάθος.

Δίνονται τα σύνολα Α, Β και ισχύουν Ν {Α - Β) = 3 , Ν {Β - Α) = 23 και Ν {Β) = 5 · Ν {Α) . Να βρεθούν τα Ν {Α r1 Β) και Ν {Α υ Β)

Έχουμε: Ν(Α - Β) = 3 � Ν (Α) - Ν(Α n Β) = 3 ( Ι ) και Ν(Β - Α) = 23 � Ν (Β) - Ν (Α n Β) = 23 (2)

Αφαιρώντας από την (2) την ( I ) έχουμε: Ν(Β) - Ν (Α) = 20 (3) . Επειδή Ν(Β) = 5 · Ν (Α) , από την (3)

παίρνουμε 5 · Ν (Α) - Ν (Α) = 20 � 4 · Ν (Α) = 20 � Ν(Α) = 5 , οπότε Ν(Β) = 25 . Από την ( 1 ) βρίσκουμε

Ν (Α n B) = 2 . Επίσης Ν (Α υ Β) = Ν ( Α) + Ν (Β) - N(A n B) = 5 + 25 - 2 = 28 .

Να γράψετε με αναγραφή τα παρακάτω σύνολα: Α = {χ ε ίΖ Ι - 4 < χ � 3} , Β = {χ ε Ν Ι χ � 5} , Γ = {χ ε ίΖ Ι χ3 - 4χ = Ο} Δ = {(χ , ψ) Ι χ ε Ν, ψ ε Ν και χ · ψ = 1 2} , Ε = {(χ , ψ) Ι χ ε Ζ, ψ ε ίΖ και χ · ψ = 6} .

Να γράψετε με περιγραφή τα παρακάτω σύνολα: Α = {-3, -2, - 1 , 0, 1 , 2, 3, 4} , Β = {0, 3 , 6, 9, . . . } , Γ = {-2, 3} ,

Δ = {3 , 8, 1 3, 1 8, . . . } Ε = {0, 2, 4, 6, 8, . . . } , Ζ = - , - , - , - , . . . . Να γραψετε το συνολο των διαιρετων του - 1 2 { 1 2 3 4 } ' ' ' 2 3 4 5

όταν σύνολο αναφοράς είναι: Το σύνολο των φυσικών αριθμών Το σύνολο των ακεραίων αριθμών. Αν Α είναι το σύνολο όλων των παραλληλογράμμων, Β το σύνολο των ορθογωνίων, Γ το σύνολο των ρόμβων, Δ

το σύνολο των τετραγώνων και Ε το σύνολο των τραπεζίων, να γράψετε τα αποτελέσματα των παρακάτω πράξεων: Β n Γ , Α υ Β υ Γ υ Δ , Δ n Ε .

Α ν Α = {χ ε ffi. I - 3 < χ � 2} , Β = {χ ε ffi. I -Ι � χ < 5} , Γ = {χ ε ffi. I χ < 4} και σύνολο αναφοράς είναι το

σύνολο ffi. των πραγματικών αριθμών, να βρείτε τα: Α υ Β , AnB, Β υ Γ , Β n Γ , Α' , Γ' , Α - Β , ΒnΓ . Αν Ω = { 1 , 2, 3 , 4, 5 } είναι το β�σικό σύνολο, να βρείτε τα σύνολα Α, Β, Γ όταν ισχύουν συγχρόνως:

A n B = {2 , 4} Α υ Β = {2, 3, 4, 5 } , Α n Γ = {2, 3} και Α υ Γ = {1 , 2, 3, 4} . Να βρείτε τα σύνολα Α και Β όταν ισχύουν συγχρόνως τα παρακάτω :

Α υ Β = {1, 2, 3 , 4, 5} , A n B = {3, 4, 5} , ] (!'Ο Α - Β και 2 (z'O B - A . Αν Ω = {-4, -3, -2, - 1 , 0, 1 , 2, 3, 4} , Α = {-3, - 1, 1 , 2} και Β = {-4, -2, 0, 1 , 2} .

Να βρείτε τα σύνολα: Α' , Β' , Ω' , (Α υ Β)' , Α' n Β' , Α - Β , Α n Β' Ποια από τα παραπάνω σύνολα είναι ίσα;

ΕΥΚΛΕΙΔΗΣ Β' 81 τ. l/40

Page 43: Ευκλειδης Β 81

-------------- Μαθηματικά για την Α ' Λυκείου -------------

Ένα κουτί περιέχει 4 σφαίρες αριθμημένες από 1 έως 4 . Να βρεθεί ο δειγματικός χώρος στις παρακάτω περιπτώσεις:

Επιλέγουμε τυχαία μία σφαίρα και σημειώνουμε τον αριθμό που έχει Επιλέγουμε τυχαία μία σφαίρα σημειώνουμε τον αριθμό που έχει, την τοποθετούμε στο κουτί

και επιλέγουμε πάλι μία σφαίρα και σημειώνουμε τον αριθμό της (επανατοποθέτηση) . Επιλέγουμε τυχαία μία σφαίρα σημειώνουμε τον αριθμό που έχει και κατόπιν επιλέγουμε πάλι

μία σφαίρα και σημειώνουμε τον αριθμό της (χωρίς επανατοποθέτηση).

Εύκολα συμπεραίνουμε ότι: Ω = { 1, 2, 3 , 4} Με την βοήθεια του διπλανού

πίνακα διπλής εισόδου βρίσκουμε ότι: Ω = { ( 1, 1 ) , ( 1, 2 ) , ( 1, 3 ) , ( 1, 4) , ( 2, 1 ) , ( 2, 2 ) , ( 2, 3 ) , ( 2, 4 ) , ( 3, 1 ) , ( 3, 2 ) , ( 3, 3 ) , ( 3, 4 ) , ( 4, 1 ) , ( 4, 2) , ( 4, 3 ) , ( 4, 4)} Με την βοήθεια του διπλανού

πίνακα διπλής εισόδου βρίσκουμε ότι: Ω = {( 1, 2 ) , ( 1 , 3 ) , ( 1, 4) , (2, 1 ) , ( 2, 3 ) , ( 2, 4) , (3, 1 ) , (3 , 2 ) , (3, 4) , ( 4, 1 ) , ( 4, 2) , ( 4, 3 )}

Ρίχνουμε ένα ζάρι δύο φορές και έστω χ , ψ είναι

� η

ι

2

3

4

� 1

2

3

4

οι ενδείξεις κατά την πρώτη και δεύτερη ρίψη αντίστοιχα. Ν α βρεθεί ο δειγματικός χώρος του πειράματος τύχης.

I 2 3

( ! , ι ) ( ι ,2) ( 1 ,3 )

(2, ι ) (2,2) (2,3)

(3 , ι ) (3 ,2) (3 ,3 )

(4, ι ) (4,2) (4,3 )

1 2 3

( 1 ,2) ( 1 ,3)

(2, 1 ) (2,3)

(3 , 1 ) (3 ,2)

(4, 1 ) (4,2) (4,3)

Να βρεθούν τα ενδεχόμενα: Α = {(χ,ψ ) ε Ω / χ · ψ = 12} και Β = {(χ,ψ ) ε Ω / χ < ψ} : Είναι τα ενδεχόμενα Α και Β ασυμβίβαστα;

4

( 1 ,4)

(2,4)

(3 ,4)

(4,4)

4

( 1 ,4)

(2,4)

(3 ,4)

, Με πίνακα διπλής εισόδου σε προηγούμενο παράδειγμα βρήκαμε ότι ο δειγματικός χώρος του πειράματος τύχης είναι: Ω = {( 1 , 1 ) , ( 1, 2 ) , ( 1, 3 ) , • • • , ( 6, 5 ) , ( 6, 6)}

Είναι: Α = {(2, 6) , ( 6,2) , (3, 4) , ( 4,3 )} και Β= {(1, 2) , (1, 3) , (1, 4) , ( 1, 5) , (1, 6) , (2, 3) , (2,4) , (2, 5) , ( 2, 6 ) , (3, 4) , (3, 5 ) , (3, 6 ) , (4, 5 ) , (4, 6) , (5, 6)}

Τα ενδεχόμενα Α και Β δεν είναι ασυμβίβαστα αφού έχουν κοινά τα στοιχεία (2, 6 ) και (3, 4) . Μεταξύ των οικογενειών με τέσσερα παιδιά επιλέγουμε τυχαία μία οικογένεια και εξετάζουμε τα παιδιά ως προς

το φύλλο και τη σειρά γέννησης τους. Να βρεθεί ο δειγματικός χώρος του πειράματος τύχης.

, ; Να βρεθούν τα ενδεχόμενα: Α: Το μικρότερο παιδί είναι αγόρι. Β: Το μεγαλύτερο παιδί είναι κορίτσι. Να βρεθούν και να περιγραφούν λεκτικά τα ενδεχόμενα: Α' , Β' , Α - Β , Β -Α , Α' υ Β' .

Ο Δειγματικός χώρος ενός πειράματος τύχης είναι: Ω = {Ο, 1, 2, 3, 4, 5 , 6, 7, 8, 9, 1 Ο} . Να βρεθούν και να παρασταθούν με διαγράμματα Venn τα ενδεχόμενα:

Α = {χ Ε Ω , όπου χ πολλαπλάσιο του 3} και Β = {χ Ε Ω , όπου χ < 6} · . Να βρεθούν τα ενδεχόμενα: Γ: Πραγματοποιείται ένα τουλάχιστον από τα Α , Β .

Δ: Πραγματοποιούνται ταυτόχρονα το Α και το Β . Ε: Δεν πραγματοποιείται το Β . Ένας μαθητής ξέχασε τον τετραψήφιο αριθμό ΡΙΝ που έβαλε στο κινητό του τηλέφωνο. Θυμάται ότι το πρώτο

ψηφίο είναι 4 , το δεύτερο ψηφίο μεγαλύτερο του 7 , το τρίτο ψηφίο είναι περιττός αριθμός και το τέταρτο ψηφίο είναι 6 .

Ποιο είναι το σύνολο των πιθανών αριθμών του ΡΙΝ; · Να βρεθούν τα ενδεχόμενα: Α : Το δεύτερο ψηφίο του αριθμού PIN είναι το 8 .

Β : Το τρίτο ψηφίο του αριθμού ΡΙΝ είναι το 3 ή το 7 . Δύο παίκτες θα παίξουν τάβλι και νικητής θα είναι εκείνος που πρώτος θα κερδίσει τρία παιχνίδια. Α ν α είναι το

ΕΥΚΛΕΙΔΗΣ Β ' 81 τ. Ι /41

Page 44: Ευκλειδης Β 81

-------------- Μαθηματικά για την Α' Λυκείου ------------­

αποτέλεσμα να κερδίσει ο πρώτος παίκτης ένα παιχνίδι και β είναι το αποτέλεσμα να κερδίσει ο δεύτερος παίκτης ένα παιχνίδι, τότε : ·. : Να βρεθεί ο δειγματικός χώρος του πειράματος τύχης.

Να βρεθούν τα ενδεχόμενα: Α: Νικητής είναι ο πρώτος παίκτης. Β: Ένας παίκτης κερδίζει δύο παιχνίδια διαδοχικά. Να παρασταθούν με πράξεις μεταξύ των συνόλων Α , Β , Γ και να βρεθούν τα ενδεχόμενα:

AυB, AnB, AnB' , Α'υΒ . Ένα άτομο βρίσκεται στη θέση Α , στο διπλανό σχήμα, και θέλει να πάει

στη θέση Ι . Μπορεί να κινηθεί μόνο μέσω των ευθυγράμμων τμημάτων που συνδέουν τις κορυφές Α, Β, Γ, Δ, Ε, Ζ, Η, Θ, Ι και κάθε φορά προς τα δεξιά ή προς τα πάνω.

Να βρείτε τον δειγματικό χώρο του πειράματος τύχης. Να παραστήσετε με αναγραφή τα παρακάτω ενδεχόμενα:

Α : Το άτομο περνά από τη θέση Ε , Β: Το άτομο δεν περνά από τη θέση Γ Γ: Το άτομο δεν περνά από τις θέσεις Δ και Θ .

Η�-----Θ�----�1

Δ t------�Ε;;;_ __ ---ι Ζ

Α Β Γ Δίνεται ο δειγματικός χώρος Ω= {ω1 ,ω2 ,ω3 ,ω4} . Αν ισχύουν Ρ(ω2 ) =3Ρ(ω1 ) , Ρ(ω3 ) = 2Ρ(ω1 ) και

Ρ( ω4 ) = 2Ρ( ω3 ) , να βρεθούν: Οι πιθανότητες των απλών ενδεχομένων του δειγματικού χώρου Ω . Να βρείτε τις πιθανότητες των ενδεχομένων: Α = {ω1 ,ωΛ , Β = {ω1 ,ω2 ,ω4 } .

Αν Ρ (ω1 ) = κ , έχουμε Ρ (ω2 ) = 3κ , Ρ (ω3 ) = 2κ και Ρ (ω4 ) = 4κ . Τότε 1

Ρ (ω1 ) + Ρ ( ω2 ) + Ρ ( ω3 ) + Ρ (ω4 ) = 1 � κ + 3κ + 2κ + 4κ = 1 � 1 0.κ = 1 � κ = ιο ·

1 3 2 4 Άρα Ρ (ω1 ) = - , Ρ (ω2 ) = - , Ρ (ω3 ) = - και Ρ (ω4 ) = - .

1 0 1 0 1 0 1 0 1 2 3 1 3 4 8

Έχουμε Ρ ( Α) = Ρ (ω1 ) + Ρ ( ω3 ) = - + - = - , Ρ (Β) = Ρ ( ω1 ) + Ρ ( ω2 ) + Ρ (ω4 ) = - + - + - = - . 1 0 1 0 1 0 1 0 1 0 1 0 1 0

Για τα ενδεχόμενα Α, Β ισχύουν: Ρ { Α') = � , Ρ { Α υ Β) = i και Ρ { Α Γ\ Β) = .!. . 4 5 5 Να βρεθούν οι πιθανότητες των ενδεχομένων: Α, Β, Α n Β' , Α' n Β και Α υ Β' .

Έχουμε Ρ ( Α) = 1 - Ρ (Α') = 1 - � = .!_ , Ρ (Α υ Β) = Ρ (Α) + Ρ (Β) - P (A n B) οπότε 4 4

4 1 1 1 3 Ρ (Β) = Ρ ( Α υ Β) + P (A n B) - P (A) = S + S - 4 = 1 - 4 = 4

Ρ (Α n Β') = Ρ ( Α - Β) = Ρ ( Α) - Ρ ( Α n B) = .!_ _ .!_ = 5 - 4 = -1 4 5 20 20

P (A' n B) = P (B - A) = P (B) - P (A n B) = � - .!_ = 1 5- 4 = .!..!.. .

4 5 20 20

Για τα ενδεχόμενα Α,Β ισχύουν: Ρ {Β) = � και Ρ{ Α - Β) = .!. . Να βρεθεί η Ρ{ Α υ Β) . 5 5

Έχουμε Ρ (Α υ Β) = Ρ (Α) + Ρ (Β) - Ρ (Α n Β) ( 1 ) και P (A - B) = P (A) - P (A n B) (2) .

Τότε Ρ (Α υ Β) = Ρ (Β) + Ρ (Α - Β) = � + .!_ = .± . 5 5 5

Στο ισόγεω ενός κτιρίου με 3 ορόφους μπαίνουν 3 άτομα. Να βρεθεί η πιθανότητα να κατέβουν: Στον ίδω όροφο δύο άτομα ακριβώς. Και τα τρία άτομα σε διαφορετικούς ορόφους.

Α ν συμβολίσουμε τα άτομα με τα γράμματα α , β , γ και τα στοιχεία του δειγματικού χώρου Ω με τριάδες της

μορφής ( ί, j, k ) με ί , j, k ε { 1, 2, 3} όπου το ί δηλώνει τον όροφο που κατεβαίνει το άτομο α , το j δηλώνει τον

ΕΥΚΛΕΙΔΗΣ Β' 81 τ. l/42

Page 45: Ευκλειδης Β 81

-------------- Μαθηματικά για την Α ' Λυκείου --------------­

όροφο που κατεβαίνει το άτομο β και το k δηλώνει τον όροφο που κατεβαίνει το άτομο γ , με δεντροδιάγραμμα βρίσκουμε ότι: Ω = { ( 1 , 1 , 1 ) , ( 1 , 1 , 2 ) , ( 1 , 1 , 3 ) , ( 1 , 2, 1 ) , ( 1 , 2, 2) , ( 1 , 2, 3 ) , ( 1 , 3, 1 ) , ( 1 , 3, 2) , ( 1 , 3, 3 ) , ( 2, 1 , 1 ) ,

( 2, 1 , 2 ) , ( 2, 1 , 3 ) , ( 2, 2, 1 ) , ( 2, 2, 2 ) , ( 2, 2, 3 ) , ( 2, 3, 1 ) , ( 2, 3, 2 ) , ( 2, 3, 3 ) , (3 , 1 , 1 ) , ( 3, 1 , 2 ) , (3 , 1 , 3 ) , (3, 2, 1 ) , (3 , 2, 2) , (3 , 2, 3 ) , (3, 3, 1 ) , ( 3 , 3, 2 ) , ( 3 , 3 , 3 )} , με Ν (Ω) = 27 .

( i ) Θεωρούμε το ενδεχόμενο Α : «Δύο ακριβώς άτομα κατεβαίνουν στον ίδιο όροφο» και τότε : Α = { ( 1, 1 , 2 ) , ( 1 , 1 , 3 ) , ( 1 , 2, 1 ) , ( 1 , 2 , 2 ) ( 1 , 3, 1 ) ( 1 , 3, 3 ) , ( 2, 1 , 1 ) , ( 2, 1, 2 ) , ( 2, 2, 1 ) , ( 2, 2, 3 ) ,

. Ν (Α) 1 8 2 (2, 3, 2) , (2,3,3) , (3, 1, 1 ) , (3, 1,3) , (3, 2, 2) , (3, 2,3) , (3, 3, 1) , (3, 3, 2)} , με Ν (Α) = 1 8 . Άρα Ρ ( Α) = -(-

)= - = - .

Ν Ω 27 3 ( i i ) Θεωρούμε το ενδεχόμενο Β : «Και τα τρία άτομα σε διαφορετικούς ορόφους» και τότε:

. Ν (Β) 6 2 Β = {( 1 , 2, 3 ) , ( 1 , 3, 2 ) , ( 2, 1 , 3 ) , ( 2, 3, 1 ) , (3, 1 , 2) , (3, 2, 1 )} , με Ν (Β) = 6 . Άρα Ρ (Β) = -(-

) = - = - .

Ν Ω 27 9 ΑΣΚΗΣ Ε Ι Σ ΠΑ Λ ΥΣ Η I . Μεταξύ των οικογενειών με τρία παιδιά επιλέγουμε τυχαία μία οικογένεια και εξετάζουμε τα παιδιά ως προς το φύλλο και τη σειρά γέννησης τους. ( i ) Να βρείτε τον δειγματικό χώρο του πειράματος τύχης. ( i i ) Να βρείτε τις πιθανότητες των ενδεχόμενων: Α : Τα δύο πρώτα παιδία είναι αγόρια και το τρίτο κορίτσι Β: Ένα τουλάχιστον παιδί είναι αγόρι Γ: Ένα μόνο παιδί είναι κορίτσι Δ: Το πολύ ένα παιδί είναι κορίτσι. 2. Ένα κουτί περιέχει 2 μπάλες κόκκινες (κ) και 3 πράσινες (π) . Βγάζουμε από το κουτί μια-μια τις μπάλες, χωρίς επανατοποθέτηση, μέχρι να βγάλουμε και τις δύο κόκκινες. Να βρείτε την πιθανότητα να διακοπή η διαδικασία στην εξαγωγή το πολύ της τρίτης μπάλας.

J . Για τα ενδεχόμενα Α , Β ισχύουν: Ρ (Α' ) = .!. , Ρ (Β') = .!_ και Ρ (Α' υ Β') = i . 3 2 4

(i) Να δείξετε ότι τα ενδεχόμενα Α , Β δεν είναι ασυμβίβαστα. ( i ί ) Να βρείτε τις πιθανότητες των: Α υ Β , Α n Β' , Α' n Β , (Α - Β) υ (Β - Α) , Α' n Β' , Α'υΒ και Α υ Β' . 4. Από το σύνολο Ω = { 1, 2, 3, 4} επιλέγουμε τυχαία δύο αριθμούς τον ένα μετά τον άλλο και με αυτούς σχηματίζουμε ένα κλάσμα. Ο πρώτος είναι ο αριθμητής και ο δεύτερος είναι ο παρανομαστής του κλάσματος. Να βρείτε την πιθανότητα ώστε το κλάσμα ( i ) Να εκφράζει ακέραιο αριθμό. ( i i) Να είναι μεγαλύτερο από την μονάδα. S . Δίνεται ο δειγματικός χώρος Ω = { 1 , 2, 3, 4, 5} . Η πιθανότητα των απλών ενδεχομένων { 2} , { 3} , { 4} , { 5}

1 είναι Ρ ( ν ) = - , όπου ν = 2, 3 , 4, 5 . ( i ) Να βρείτε την πιθανότητα του απλού ενδεχομένου { 1 } . 2v

( i i ) Να βρείτε τις πιθανότητες των ενδεχομένων: Α = { 2, 3} , Β = { 2, 4, 5} και Γ = { l, 2, 3} . 6. Έστω Α , Β είναι τα ενδεχόμενα ένας συγκεκριμένος γιατρός να βρίσκεται στις 8 πμ στο ιατρείο του ή στο σπίτι του αντίστοιχα. Α ν Ρ (Α) = Ο, 3 και Ρ (Β) = Ο, 6 , να βρείτε την πιθανότητα στις 8 πμ ο γιατρός να μη βρίσκεται στο ιατρείο ούτε στο σπίτι του . 7. Έστω Α , Β είναι δύο ενδεχόμενα ενός δειγματικού χώρου Ω για τα οποία ισχύουν Ρ( Α') � Ρ( Α n Β) και

1 Ρ(Β') �Ο,3 . Να αποδείξετε ότι: ( i ) Ρ ( Α) � - ( i i ) τα ενδεχόμενα Α , Β δεν είναι ασυμβίβαστα 2

8. Έστω Ω = { 1 , 2, 3, 4, 5 , 6} είναι ο δειγματικός χώρος ενός πειράματος τύχης τέτοιος ώστε :

Ρ (Α') 5 -- < -Ρ (Β) - 7

.

Ρ (1 ) = Ρ (2) = 2Ρ (3 ) = 2Ρ (4) = 3Ρ (5 ) = 3Ρ (6) . 'Εστω επίσης Α = {2, χ 2 + 1 } και Β = {3, 2χ} , χ ε � δύο

ενδεχόμενα του Ω . ( i ) Να βρείτε τις πιθανότητες Ρ ( 1 ) , Ρ (2 ) , Ρ (3 ) , Ρ (4) , Ρ (5) και Ρ (6) ( i i ) Αν ΑυΒ={2,3,4,5} , να βρείτε: ο. Τον πραγματικό αριθμό χ β. Τις πιθανότητες των: Α, Β, A n B και Α - Β .

Σz::ηκ11 ενδ�: ικτικi1 β ιβλ ιογραφία : 1 . 1000 Exercices Corriges de Mathematiques - Alain Gastineau, Ph i l ippe Po itrat 200 I , [Γαλλία] 2. Αλγεβρικά θέματα, τόμος Β-Γ, Παν. Μάγειρας, < Εκδ . : 1 975 [Αθήνα] . 3 . Άλγεβρα Α ' Λυκείου, Τόμος Α' , Θύμιος Διαμαντόπουλος, Βαγγέλης Ζώτος <Εκδ . : Ξιφαράς 201 1, [Αθήνα] . 4 . Problems in Higher Mathematics - Υ. Gονοrον , Ρ. Dyboν, Ν. Miroshin, S . Smimoνa, 1 994,[ Η.Π.Α.]

ΕΥΚΛΕΙΔΗΣ Β' 81 τ. l/43

Page 46: Ευκλειδης Β 81

------------ Μαθηματικά για την Α ' Λυκείου ------------

Βαγγέλης Ευσταθίου

Σ' αυτό το σύντομο σημείωμα, σας παραθέτουμε μερικές ενδεικτικές ασκήσεις από το σύνολο R των πραγματικών αριθμών, για εμπειρία και εξάσκηση

� , Α ν ψ :;ι: 1, z. :;ι: 1 , z :;ι: Ο , χ = ____!___ και ψ = _z_ τότε να αποδειχθεί ότι: 1 - ψ 1 - z

z 1 - 2z :;t: Ο και χ = --1 - 2z

χ + 2ψ 2χ - ψ :;ι: Ο και -- + 5z = 3

2χ - ψ.

z z

0 1 - 2z 0 , ψ 1 - z z ψ :;t: 1 � 1 -- :;t: � -- :;t: � 1 - 2z :;ι: Ο, οποτε χ = -- = -- = --1 - z 1 - z 1 - ψ 1 - 2z 1 - 2z

1 - z z 2z -- + -

; ; \ 2χ - ψ = � - -z- = z ; ο � χ + 2ψ + 5z = 1 - 2z 1 - z + 5z =

1 - 2z 1 - z (1 - z)(1 - 2z) 2χ - ψ z (1 - z)(l - 2z)

z · (1 - z) + 2z · (1 - 2z) + Sz = z- z2 + 2z - 4z2 + Sz = 3z

- 5z2 + Sz = 3z- 5z2 + 5z2 = 3z = 3 . z z z z z

Αν α + β + γ = Ο και α2 + β2 + γ2 = 1 να αποδειχθεί ότι: α4 + β4 + γ4 = .!. 2 /υΆ;σ η

2 . Έχουμε α4 + β4 + γ4 = (α2 + β2 + γ2 ) _ 2 (α2β2 + β2γ2 + α2γ2 ) = 1 _ 2 (α2β2 + β2γ2 + α2γ2 )

Όμως α2β2 + β2γ2 + α2γ2 = ( αβ + βγ + αγγ - 2αβγ ( α + β + γ) = ( αβ + βγ + αγ)2 , αφού α + β + γ = Ο

οπότε α4 + β4 + γ4 = 1 - 2 (αβ + βγ + αγ)2 Επειδή 2 ( αβ + βγ + αγ) = (α + β + γ )2 - ( α2 + β2 + γ2 ) = 0 - 1 = - 1 έχουμε αβ + βγ + αγ = _..!. , οπότε

2

4 4 4 1 1 ( )2 α + β + γ = 1 - 2 -

2 =

2 .

,_ '"-

1 Να γίνουν οι πράξεις: --=---­χ2 + 4χ + 4

' 4 4 1 4 4 Εχουμε - + = - + = χ2 + 4χ + 4 χ4 + 4χ3 + 4χ2 χ3 + 2χ2 χ2 + 4χ + 4 χ 2 ( χ2 + 4χ + 4) χ2 ( χ + 2)

4 4 Χ2 - 4 + 4 (χ + 2) χ2 - 4 + 4χ + 8 χ 2 + 4χ + 4 ( χ + 2)2

(χ + 2)2 χ 2 ( χ + 2)2 +

χ 2 (χ + 2 ) =

χ2 ( χ + 2)2 =

χ2 ( χ + 2)2 =

χ2 ( χ + 2)2-

χ2 ( χ + 2)2 χ2

Α ν α β > Ο , να αποδειχθεί ότι: � + ! � 2 β α

Αν α > Ο , β > Ο και γ > Ο , να αποδειχθεί ότι: (α + β + γ) · (! + ! + !) � 9 α β γ

Αφού αβ > Ο , αρκεί να δείξουμε ότι: ΕΥΚΛΕΙΔΗΣ Β' 81 τ. l/44

Page 47: Ευκλειδης Β 81

------------ Μαθηματικά για την Α ' Λυκείου -----------­

� + Q_ � 2, ή αβ� + αβQ_ � 2αβ, ή α2 + β2 � 2αβ, ή α2 - 2αβ + β2 � 0, ή (α - β)2 � 0 , η οποία β α β α

είναι αληθής 1 Από Ο ; έχουμε: � + Q_ � 2 και όμοια Q. + l � 2 και � + 1 � 2 β α γ β γ α

Προσθέτοντας τις ανισότητες αυτές κατά μέλη παίρνουμε : � + Q_ + Q. + r + � + l � 6 � Q. + r + � + l + � + � � 6 � β + γ + α + γ + α + β � 6 � β α γ β γ α α α β β γ γ α β γ

β + γ α + γ α + β α + β + γ α + β + γ α + β + γ ( 1 1 1 ) 1 + -- + 1 + -- + 1 + -- � 9 � + + � 9 � (α + β + γ) · - + - + - � 9 α β γ α β γ α β γ

Α x + z z χ λ , λ ' χ ψ z ν -- = - = -- να υπο ογισετε τους ογους: - , - και - . ψ Χ Ζ - ψ ψ Ζ Χ

Α ν α + β = 4 και αβ = 3 , να υπολογίσετε τις εκφράσεις : 2 2 3 3 1 1 1 1 1 1 α β α + β , α + β , - + - , -2

+ 2 , 3 + 3 και 2 + -2 . α β α β α β β α Αν α + β + γ = 4 και α2 + β2 + γ2 = 1 0 να αποδείξετε ότι: α3 + β3 + γ3 - 3αβγ = 28 Να aπλοποιήσετε τις παραστάσεις: ( 5α 5χ 1 0αχ ) ( α χ 2αχ ) , -- + -- + . -- + -- - 1 α + χ α - χ α2 - χ2 α + χ α - χ α2 - χ2 ·

, , , [( s r ) ( r2 s2 )] r4 _ 54 , , , Να δειχθει οτι η Α= - + -- · 2 + 2 - 2 : -2 -

2 ειναι ανεξαρτητη απο τα r και s

r - s r + s s r r · s

Αν Ρ (μ, ν ) = ( 2μ2

2 + 1) : (_!_ μ + ν ) , να υπολογίσετε το γινόμενο

ν - μ 2 2μ - 2ν Ρ (2, 1 ) · Ρ (3, 2) · Ρ ( 4, 3) · Ρ ( 5 , 4)

Αν οι αριθμοί χ , ψ με χ * -ψ είναι ακέραιοι, να αποδείξετε ότι ο αριθμός: χ5 - χ4ψ + χψ2 + χ3 + χ2ψ + ψ5 - χψ4 + ψ3 Α = είναι φυσικός.

χ3 + ψ3 + χ2ψ + χψ2 Για κάθε α,β ε � να αποδείξετε ότι: α2 + β2 + γ2 � αβ + αγ + βγ (α + β + γ)2 � 3 (α2 + β2 + γ2 ) Oli) α4 + β4 � α3β + αβ3

Για τους πραγματικούς αριθμούς χ , ψ , z, α δίνεται ότι χ + ψ + z = α .Να αποδείξετε ότι: 2 2 2 α2 α2 i H χ + ψ + z � 3 χψ + ψz + zχ �

3 Α ν α , β θετικοί αριθμοί και α + β = 1 να αποδειχθεί ότι:

\ ' ' αβ � � ( 1 +±} ( 1 + i) � 9 5 α4 + β4 � i

Αν α + β � Ο , να αποδείξετε ότι: \ � .ι α3 + β3 � α2β + αβ2

Α ν α , β είναι θετικοί αριθμοί να αποδείξετε ότι:

, α3 + β3 ( α + β )3 ι � --

2 2

( α4 + β4 ) ( αs + βs ) � 2 ( α9 + β9 ) αs - αs + α2 - α + 1 > Ο

ΕΥΚΛΕΙΔΗΣ Β ' 81 τ.l/45

Page 48: Ευκλειδης Β 81

Μαθηματικά για την Α ' Λυκείου

Α ' ΛΥΚΕΙΟΥ ΑΡΧΙΚΕΣ ΕΝΝΟΙΕΣ -ΙΣΟΤΗΤΑ ΤΡΙΓΩΝΩΝ - ΑΝΙΣΟΠΚΕΣ ΣΧΕΣΕΙΣ ­

ΣΧΕΠΚΕΣ ΘΕΣΕΙΣ ΔΥΟ ΚΥΚλΩΝ

Δίνεται ευθύγραμμο τμήμα ΑΒ και ση­μείο Μ εσωτερικό του ΑΒ ώστε .ΜΑ=�ΜΒ τότε:

4 Α ν Ρ εσωτερικό σημείο του ΜΒ να δειχθεί

. ΡΜ 4ΡΑ - 3ΡΒ οτι = -----

7 Αν Σ σημείο στην προέκταση του ΑΒ Β δ θ , , ΣΜ

4ΣΑ + 3ΣΒ προς το να ειχ ει οτι = 7

Α Μ Ρ Β Σ

Από την υπόθεση έχουμε 4ΜΑ=3ΜΒ ( 1 ) Έχουμε :ΡΜ=ΡΑ-ΑΜ� 4ΡΜ=4ΡΑ-4ΑΜ (2)

Αλλά και ΡΜ=ΜΒ-ΡΒ� 3ΡΜ=3ΜΒ-3ΡΒ (3)

Από (2) και (3) 7ΡΜ=4ΡΑ-4ΑΜ+3ΜΒ-3ΡΒ οπότε από . 4ΡΑ - 3ΡΒ την ( 1 ) προκυπτει: 7ΡΜ=4ΡΑ-3ΡΒ� ΡΜ = .

7 Έχουμε :ΣΜ=ΣΑ-ΑΜ�4ΣΜ=4ΣΑ-4ΑΜ (4)

Αλλά και ΣΜ=ΜΒ+ΣΒ�3ΣΜ=3ΜΒ+3ΣΒ (5) Από (4),(5) έχουμε: 7ΣΜ=4ΣΑ-4ΑΜ+3ΜΒ+3ΣΒ και από την ( Ι ) προκύπτει:

7ΣΜ=4ΣΑ+3ΣΒ� ΣΜ =

4ΣΑ + 3ΣΒ . 7

1 . Δίνεται αμβλεία γωνία χ Α ψ και στο εσωτερικό της οι ημιευθείες Αδ και Αζ ώ­

στε οι αμβλείες γωνίες χ Α δ και ψ Α ζ να εί­

ναι ίσες. Αν Αε η διχοτόμος της γωνίας δ Α ζ να δειχθεί ότι:

Οι γωνίες χ Α δ και ψ Α ζ έχουν κοινά εσωτερικά σημεία.

Η ημιευθεία Αε είναι και διχοτόμος της " γωνίας χ Α ψ.

Ισχύει χ Α δ + ψ Α ζ= χΑ ψ + δ Α ζ.

Οι γωνίες χΑ δ και ψ Α ζ είναι αμβλείες άρα ισχύει: Ι L < χΑ δ < 2 L ( I ) και Ι L < ψ Α ζ < 2 L (2). Με πρό­σθεση των ( 1 ) και (2) κατά μέλη.

Έχουμε: 2 L < χΑ δ + ψΑ ζ < 4 L (3) Αλλά χΑ ψ < 2 L (4) . Από (3), (4) � χΑ ψ < χΑ δ + ψ Α ζ και επειδή χΑ δ και ψ Α ζ βρίσκο­νται στο εσωτερικό της χ Α ψ άρα έχουν κοινά εσωτερικά σημεία.

Του Αποστόλη Κακαβά δ

ψ

Α iι Ισχύει: χΑ ζ = χΑ δ - ζ Α δ = ψΑ ζ -ζ Α δ = ψ Α δ

( 5) .Από υπόθεση έχουμε ότι Α ε διχοτόμος της γωνίας δ Α ζ οπότε ζ Α ε = ε Α δ ( 6). Από (5), (6) � χΑ ζ+ζΑ ε = ψ Α δ + εΑ δ�χΑ ε=ψ Α ε

"

άρα η ημιευθεία Αε είναι και διχοτόμος της γωνίας χΑ ψ . ι: ι χΑ δ + ψΑ ζ = (ζΑ δ + χΑ ζ ) + ψ Α ζ =

ζΑ δ + (χΑ ζ + ψΑ ζ)= ζΑ δ + χΑ ψ . Παρατήρηση: Ειδικά αν οι γωνίες χΑδ , yΑζ είναι ορθές, τότε προφανώς οι xAy , δΑζ είναι παραπληρωματικές.

, . Δίνεται τρίγωνο ΑΒΓ, με ΑΒ<ΑΓ και η δι­χοτόμος του ΑΔ .Στην πλευρά του ΑΓ έστω σημείο Ε ώστε ΑΕ=ΑΒ να δειχθεί ότι:

ι Το τετράπλευρο ΑΒΔΕ έχει κάθετες διαγώ-νιους.

:: ι Η ΑΔ διχοτομεί και την γωνία Β Δ Ε. ; ι ι ΔΓ > ΔΒ. i ' . Δεν υπάρχει τρίγωνο με πλευρές ΑΒ,ΑΓ,ΕΓ.

Λ \:ση : i Το τρίγωνο ΑΒΕ είναι ισοσκελές με κορυφή Α

και η ΑΔ είναι διχοτόμος άρα και διάμεσος και ύψος οπότε το τετράπλευρο ΑΒΔΕ έχει κάθετες διαγώνιους.

\ i. Από το προηγούμενο έχουμε ότι η ΑΔ είναι με-σοκάθετος του τμήματος ΒΕ άρα ΒΔΕ ισοσκελές με κορυφή Δ οπότε η ΑΔ διχοτομεί και την γωνία Β Δ Ε.

Α

Β Δ Γ

H i . Τα τρίγωνα ΑΒΕ και ΒΔΕ είναι ισοσκελή με κορυφές Α και Δ αντίστοιχα οπότε Α Β Δ=Α Ε Δ ως ά­θροισμα ίσων γωνιών, άρα και Β cξ = Δ Ε Γ( l ) ως πα­ραπληρωματικές των γωνιών Α Β Δ και Α Ε Δ αντί-

ΕΥΚΛΕΙΔΗΣ Β ' 81 τ. l/46

Page 49: Ευκλειδης Β 81

Μαθηματικά για την Α ' Λυκείου

στοιχα. Αλλά Β εξ > Γ (2) οπότε από ( l ), (2) :::>Δ Ε Γ> t ::::>ΔΓ >ΔΕ::::>ΔΓ>ΔΒ (επειδή ΔΒ=ΔΕ) .

Έχουμε ότι ΕΓ= ΑΓ-ΑΕ=ΑΓ-ΑΒ άρα δεν υπάρ­χει τρίγωνο με πλευρές ΑΒ,ΑΓ,ΕΓ γιατί λόγω της τριγ. ανισότητας έπρεπε να ισχύει: ΑΓ -ΑΒ<ΕΓ<ΑΓ+ΑΒ.

Δίνεται τρίγωνο ΑΒΓ, με ΑΒ < ΑΓ και στις προεκτάσεις των πλευρών του ΒΑ,Γ Α ση­μεία Ε και Δ αντίστοιχα ώστε ΑΔ=ΑΒ και Α­Ε=ΑΓ. Αν Κ το σημείο τομής των ΕΔ,ΓΒ, τότε να δειχθεί ότι η ΚΑ είναι μεσοκάθετος των ΒΔ και ΓΕ και διχοτόμος της ΕΚΓ .

Τα τρίγωνα ΑΒ Γ και ΑΔΕ έχουν: ΑΔ=ΑΒ (από την υπόθεση) ΑΕ=ΑΓ (από την υπόθεση) Β Α Γ= Δ Α Ε (Ως κατακορυφήν) Από το κριτήριο Π-Γ -Π τα τρίγωνα είναι ίσα επο-

μένως ΒΓ=ΔΕ ( l ) και Α Γ Β= Δ Ε Α (2) και Α Β Γ= Α Δ Ε (3) .

κ

Ε

Β r Τα τρίγωνα ΚΔ Γ και ΚΒΕ έχουν: Α Γ Β=Δ Ε Α (από

την (2)), ΒΕ=ΓΔ (άθροισμα ίσων ευθυγράμμων τμημάτων), ΚΒ Ε= κ Δ Γ (παραπληρωματικές ίσων γωwί:ιν (από την (3)) Από το κριτήριο Γ -Π -Γ τα τρίγωνα είναι ίσα επομένως ΚΒ=ΚΔ (4) => το σημείο Κ ισαπέχει από τα σημεία Δ και Β => το σημείο Κ ανήκει στην μεσοκάθετο του ΔΒ. Αλλά και ΑΒ=ΑΔ=> το σημείο Α ισαπέχει από τα ση­μεία Δ και Β => το . σημείο Α ανήκει στην μεσοκάθετο του ΔΒ. Επειδή τα σημεία Κ και Α είναι διαφορετικά και από δύο διαφορετικά σημεία διέρχεται μοναδική ευθεία η ΚΑ είναι μεσοκάθετος του ΔΒ. Εξάλλου: ( l ),(4)=>ΚΒ+ΒΓ=ΚΔ+ΔΕ::::>ΚΓ=ΚΕ τότε το ση μείο Κ ανήκει στην μεσοκάθετο του ΓΕ. Αλλά και το ση μείο Α ισαπέχει από τα σημεία Γ και Ε. Άρα η ΚΑ είνα μεσοκάθετος του ΓΕ και προφανώς διχοτόμος της γωνία ΕΚΓ του ισοσκελούς τριγώνου ΕΚΓ.

Δίνεται ισοσκελές τρίγωνο ΑΒΓ με ΑΒ=ΑΓ και τα μέσα Ε και Ζ των πλευρών του ΑΒ και ΑΓ αντίστοιχα. Α ν οι διχοτόμοι των ε-

� � ξωτερικών γωνιών του Β και Γ τέμνονται στο σημείο Κ να δειχθεί ότι :

� � Β Ε Κ=Γ Ζ Κ Τα σημεία Β και Γ είναι συμμετρικά ως

προς την ευθεία ΑΚ.

Έχουμε:

Κ Β Γ= Βεξ = 180° - Β 2 2

1 80° - Γ =Γεξ =Β r κ ( Ι )

2 2 Α

Τα τρίγωνα ΕΒΚ και ΓΖΚ έχουν: ΒΕ=ΓΖ (Ως μισά ίσων πλευρών) ΒΚ=ΓΚ (από την ( Ι ) το τρίγωνο ΚΒΓ ισοσκελές με κορυφή Κ) . Ε Β Κ=Ζ Γ Κ (Ως άθροισμα ίσων γωνιών) Από κριτήριο Π-Γ -Π τα τρίγωνα είναι ίσα επομέ-

νως Β Ε Κ=Γ Ζ Κ . Από το προηγούμενο ερώτημα έχουμε ΒΚ=ΓΚ

άρα το σημείο Κ ισαπέχει από τα άκρα του ΒΓ άρα α­νήκει στην μεσοκάθετο του ΒΓ. Αλλά και το τρίγωνο ΑΒΓ είναι ισοσκελές με κορυφή Α οπότε ΑΒ=ΑΓ δη­λαδή το σημείο Α ισαπέχει από τα άκρα του ΒΓ άρα ανήκει στην μεσοκάθετο του ΒΓ. Επειδή τα σημεία Κ και Α είναι διαφορετικά και από δύο διαφορετικά ση­μεία διέρχεται μοναδική ευθεία η ΚΑ είναι μεσοκάθε­τος του ΒΓ οπότε τα σημεία Β και Γ είναι συμμετρικά ως προς την ευθεία ΚΑ.

Δίνεται ευθύγραμμο τμήμα ΑΒ=α να βρεθεί ο γεωμ. τόπος των σημείων του επιπέδου που απέχουν από το σημείο Α απόσταση ίση με 4 1 - α και από το σημείο Β απόσταση ίση με - α . 5 3

Έστω σημείο Μ του γεωμετρικού τόπου επειδή

απέχει από το σημείο Α απόσταση ίση με iα ανήκει 5

στον κύκλο (Α, iα ) και επειδή απέχει από το σημείο 5

Β , , 1 , ,

λ (Β l ) αποσταση ιση με -α ανηκει στον κυκ ο , -α . 3 3

Α

ΕΥΚΛΕΙΔΗΣ Β ' 81 τ. l/47

Page 50: Ευκλειδης Β 81

Μαθηματικά για την Α ' Λυκείου

Οι παραπάνω κύκλοι έχουν διάκεντρο δ=ΑΒ=α άρα τέμνονται διότι ισχύει ρ 1 -ρ2 <δ<ρ 1+ρ2 πράγματι

4 Ι 7 ρ,

- ρ2 = Sα - 3α = 15 α < α=δ και

4 Ι Ι 7 ρ + ρ = -α + -α = -α < α=δ. (Ι) 1 2 5 3 Ι 5 Το σημείο Μ ανήκει και στους δύο τεμνόμενους

κύκλους· άρα είναι ένα από τα δύο σημεία τομής Γ, Δ των δύο κύκλων.

Έστω ότι: Μ ε {Γ, Δ} . Τότε :

ΑΜ=ΑΓ=ΑΔ= iα και ΒΜ=ΒΓ=ΒΔ= .!_ α . Άρα ο γεω-5 3

μετρικός τόπος των σημείων του επιπέδου που απέχουν , , Α , , 4 , απο το σημειο αποσταση ιση με S α και απο το ση-

μείο Β απόσταση ίση με .!.α είναι το σύνολο {Γ,Δ} . 3

1 •. Δίνεται οξυγώνιο τρίγωνο ΑΒΓ και ση­μείο Δ εσωτερικό της πλευράς του ΒΓ. Από το σημείο Δ φέρουμε τμήμα ΔΚ κάθετο στην ΑΒ και τμήμα ΔΛ κάθετο στην ΑΓ να δειχθεί ότι :

. β + γ - α ΑΔ ' ' i . < < τ, οπου τ η ημιπεριμετρος 2

του ΑΒΓ τριγώνου . , ΚΛ < ΒΓ .

Α

Β Δ Γ ι Εφαρμόζουμε τριγωνική ανισότητα στο τρίγωνο

ΑΒΔ οπότε ΑΔ < ΑΒ + ΒΔ ( Ι ) και στο τρίγωνο ΑΓ Δ οπότε ΑΔ < ΑΓ + ΓΔ (2) . Με πρόσθεση των σχέσεων ( Ι ) και (2) κατά μέλη έχου­με : 2ΑΔ < ΑΒ + (ΒΔ + Γ Δ ) + ΑΓ ::::> 2ΑΔ < ΑΒ + Β Γ + ΑΓ ::::>2ΑΔ < 2τ ::::> ΑΔ < τ .

β < ΑΔ + ΔΓ} γ < ΑΔ + ΔΒ

::::> β + γ < 2ΑΔ + α ::::>

β + γ - α β + γ - α < 2ΑΔ ::::> < ΔΑ 2

Σημείωση : Η απόδειξη που συναντάμε στα περισσότε-

• Το να αποδείξουμε ότι δύο κύκλοι με τις προϋποθέ­σεις (Ι) τέμνονται είναι ένα από τα πλέον δύσκολα προ­βλήματα της Γεωμετρίας και καλό είναι να μας απα­σχολήσει

ρα βιβλία: ::::>2ΑΔ>β+γ-α ::::>ΑΔ> -- , ΑΔ>β-ΔΓ} β+γ-α ΑΔ>γ-ΔΒ 2

δεν είναι σωστή, αφού πιθανόν να είναι: β<ΔΓ ή γ<ΔΒ. Εφαρμόζουμε τριγωνική ανισότητα στο τρίγωνο

ΚΛΔ οπότε ΚΛ < ΚΔ + ΔΛ (3) . Στο ορθογώνιο τρίγω­νο ΚΒΔ η υποτείνουσα ΒΔ είναι η μεγαλύτερη οπότε ΚΔ < ΒΔ (4) . Ομοίως και στο τρίγωνο ΔΛΓ ισχύει ότι ΔΛ < ΔΓ (5) Με πρόσθεση των σχέσεων (4) και (5) κατά μέλη έχουμε: ΚΔ+ΔΛ<ΒΔ+ΔΓ::::>ΚΔ+ΔΛ<ΒΓ (6) . Από τις σχέσεις (3), (6) λόγω της μεταβατικής ιδιό­τητας έχουμε : ΚΛ < ΒΓ .

(' Δίνεται κυρτό τετράπλευρο ΑΒΓ Δ και τα εσωτερικά σημεία Κ, Λ, Μ, Ν των πλευρών του ΑΒ , ΒΓ , ΓΔ , ΔΑ . Να δειχθεί ότι:

Η περίμετρος του τετραπλεύρου ΚΛΜΝ είναι μικρότερη από την περίμετρο του τετρα­πλεύρου ΑΒΓ Δ .

' · , ΜΚ < ΚΑ + ΑΔ + ΔΜ . �; Η περίμετρος 'tiJ του τετραπλεύρου ΑΒΓ Δ

είναι μεγαλύτερη από το άθροισμα ΚΜ + ΝΛ . Στα τρίγωνα ΑΚΝ , ΒΚΛ , Γ ΛΜ , ΔΜΝ εφαρμόζου­

με τριγωνική ανισότητα και έχουμε: ΚΝ < ΚΑ+ΑΝ ( Ι ) , ΚΛ < ΚΒ + ΒΛ (2), ΛΜ < ΛΓ + ΓΜ (3) , ΜΝ < ΜΔ + ΔΝ (4) .

Γ

Α Κ Β Με πρόσθεση των ν ( I ) , (2) , (3) , (4) κατά μέλη έχου­με: ΚΝ + ΚΛ + ΛΜ + ΜΝ < (ΑΚ + ΚΒ) + (ΒΛ + ΛΓ) + (ΓΜ + ΜΔ)+(ΔΝ+ΝΑ) ::::> ΚΝ + ΚΛ + ΛΜ + ΜΝ < ΑΒ + ΒΓ + ΓΔ + ΔΑ . ι ι . Στα τρίγωνα ΑΚΜ, ΑΜΔ εφαρμόζουμε τριγωνική ανισότητα και έχουμε: ΜΚ<ΚΑ+ΑΜ (5) και ΑΜ<ΑΔ+ΔΜ (6). Με πρόσθεση των (5), (6) έχουμε: ΜΚ+ΑΜ<ΚΑ+ΑΜ+ΑΔ+ΔΜ::::>ΜΚ<ΚΑ+ΑΔ+ΔΜ.

Γ

Α Κ Β Από το Β. α. έχουμε: ΜΚ < ΚΑ + ΑΔ + ΔΜ (7) .

Ομοίως ΜΚ < ΚΒ + ΒΓ + ΓΜ (8), οπότε :

( 7 ) , (8) ::::> 2ΜΚ < m ::::> ΜΚ < m . Ομοίως: 2

ΝΛ < tΙJ , οπότε ΜΚ+ΜΛ<tσ. 2

ΕΥΚΛΕΙΔΗΣ Β ' 81 τ.l/48

Page 51: Ευκλειδης Β 81

Β ' ΛΥΚΕΙΟΥ

Λεωνίδας Μαυρογιαννάκης Λ .. Τ

ί'Ί.σκψ:η� 1 11 Δίνεται η συνάρτηση f(x) = 3ημ( 23χ) . ί) Ν α βρεθεί η περίοδος, η μέγιστη και η ελάχιστη τιμή της. ίί) Να βρεθούν τα διαστήματα μονοτονίας στο διάστημα μιας περιόδου. ίίί) Ν α γίνει η γραφική παράσταση και να βρείτε για ποιες τιμές του χ παίρνει την μέγιστη

και την ελάχιστη τιμή στο ίδιο διάστημα.

ί) Η 'οδ • τ 2π 2π 6π 3π Η · · f · 3 �� _,_ · 3 περι ος ειναι: = - = -2 =- = . μεγιστη τιμη της ειναι ρ = και η i::J\Uχιστη ειναι -ρ = - .

ω 2 3

ίί) . Η f είναι γνησίως αύξουσα στα διαστήματα [Ο, 3:] και [ 9: , 3π] , και γνησίως φθίνουσα στο χ ο 3π/4 3π/2 9π/4

ημχ δ ' [ 3π 9π ] ' ' ' ιαστημα 4,4 , οπως φαινεται στον πινακα και

:m .,,.

ίίί) Η γραφική παράσταση της συνάρτησης φαίνεται στο σχήμα.

Γ 3π ' ' ' ια χ = 4 παιρνει την μεγιστη τιμη της

f ( 3π ) 3 9π ' λ ' 4 = και για χ = 4 παιρνει την ε αχιστη

f ( 9; ) = -3 .

Να λυθούν οι παρακάτω τριγωνομετρκές εξισώσεις:

ί) 4ημχ · συνχ - .J3 = 2.J3ημχ - 2συνχ ίί) η μ 2χ · εφ χ - συν2χ · σφχ = Ο ίίί) συνχ - ημχ + ημχ · συνχ = 1 iv) ημ4χ + συν4χ = 1 ν) εφ2χ · εφ3χ = 1 , στο διάστημα [-π, π] vi) 4ημ2 ( χ-�)+ 2J2ημ( χ-�)=J2 + 2συν(2; - χ) /\ ·{ιστ� i) 'Ε χουμε ισοδύναμα: 4ημχ · συνχ - 2.J3ημχ = -2συνχ + .J3 <::> 2ημχ ( 2συνχ -.J3) = -2συνχ + .J3 <::>

ΕΥΚΛΕΙΔΗΣ Β ' 81 τ. J /49

Page 52: Ευκλειδης Β 81

------------- Μαθηματικά για τη Β ' Λυκείου -------------

2ημχ ( 2συνχ -ν'3) + 2συνχ -ν'3 =0 <:::::> (2ημχ+ ι) ( 2συνχ -ν'3) =0 <:::::> 2ημχ + 1 = ο ( 1 ) ή 2συνχ -ν'3 =0 (2). 'Εχουμε: ( l ) <=> ημχ = -_!_ <:::::> ημχ = ημ(-�)<=> χ = 2κπ -� ή χ = 2κπ + π + �<=> χ = 2κπ -� ή 2 6 6 6 6

7π J3 π π χ = 2κπ +-, Κ Ε Ζ και (2) <=> συνχ =-<:::::> συνχ = συν- <:::::> χ = 2κπ ± - . 6 2 6 6

Για να ορίζεται η εξίσωση, δηλαδή οι εφ χ , σφχ πρέπει και αρκεί χ * κπ + � και χ * κπ, κ Ε Ζ . (I) 2 3 3 . 2 2 ? ημχ 2 συνχ ημ χ συν χ Τοτε: ημ χ · εφχ - συν χ · σφχ = Ο <=> ημ-χ ·---συν χ ·--= 0<:::::> -----=0<:::::> συνχ ημχ συνχ ημχ

ημ4χ - συν4χ = Ο <:::::> { ημ2χ - συν2χ ) { ημ2χ + συν2χ ) = Ο <:::::> ημ2χ - συν2χ = Ο ( 1 ) . Αφού ημ2χ + συν2χ = 1 , άρα: ( Ι) <=> ( ημχ -συνχ) ( ημχ +συνχ) = 0 <:::::> ημχ -συνχ = Ο (2), ή ημχ +συνχ =0 (3) (2) <:::::> ημχ = συνχ <:::::> ημχ = ημ (� - χ) <:::::> χ = 2κπ +� - χ ή χ = 2κπ + π - (� - χ)<=> , <=> 2χ = 2κπ + 2 ή Ο · χ = 2κπ + 2 (που είναι αδύνατη, γιατί; ) <:::::> χ = κπ + 4 , κ Ε Ζ Επίσης: (3) <:::::> ημχ = -συνχ <:::::> ημχ = -ημ (� - χ)<:::::> ημχ = ημ ( χ -�)<:::::> χ = 2κπ+ χ -� ή χ = 2κπ + π - (χ -�)<:::::> Ο · χ = 2κπ -�( αδύνατη) ή 2χ = 2κπ + }π <=> χ = κπ + }π , κ Ε z 2 2 2 4

Οι τιμές είναι δεκτές, αφού επαληθεύουν τους περιορισμούς (I) π. χ. κ 1π + � * κ2π + � ' για κάθε 4 2 Κ1 , κ2 Ε Ζ (γιατί;) κ.λ.π. (Ο τρόπος αυτός αποτελεί την γενική μέθοδο επίλυσης εξισώσεων της μορφής α ( συνχ ± ημχ) + βημχσυνχ + γ = Ο . Καλό είναι να προσεχθεί ιδιαίτερα διότι κρύβει παγίδες).

Παρατηρούμε ότι: ( συνχ - ημχ )2 = συν2χ + ημ 2χ - 2ημχσυνχ = 1 - 2ημχσυνχ, δηλαδή Ι 2 συνχ - ημχ = ω} ( 2 ) - ( συνχ - ημχ ) ημχσυνχ = (ί) . Άρα συνχ - ημχ + ημχ · συνχ = 1 ( 1 ) <:::::> 1 - ω2 _ 2 ω +-2- - 1 (3)

( 3) <:::::> ω2 - 2ω + 1 = Ο <:::::> ω = 1 , οπότε ( 2) <:::::> συνχ - ημχ = 1 και (i)�ημχσυνχ�ημχ=Ο ή συνχ=Ο�χ=κπ ή χ=κπ+� , κΕΖ. Δεκτές μόνον οι τιμές χ = 2κπ ή χ = (2κ + l) π +� (γιατί; ) 2 2

( 1) <:::::> συνχ - ημχ + ημχ · συνχ - 1 = Ο <:::::> ( συνχ - 1 ) + ημχ ( συνχ - 1) = Ο <:::::> <=> (συνχ - 1 ) (ημχ + 1 ) = 0 <:::::> συνχ = 1 ή ημχ = - 1 <:::::> χ = 2κπ ή χ = 2κπ + 3� , Κ Ε Ζ 2

ημ4χ + συν4χ = 1 <:::::> { ημ2χ + συν2χ )2 - 2ημ2χ · συν2χ = 1 <:::::> <:::::> 2ημ2χ · συν2χ = Ο <:::::> ημχ = Ο ή συνχ = Ο <:::::> χ = κπ ή χ = κπ + � , κ Ε Ζ 2 Για να ορίζεται η εξίσωση, δηλαδή οι εφ2χ , εφ3χ πρέπει και αρκεί 2χ * κπ + � και 2

3χ * κπ + � , δηλαδή χ * κ� + � και χ * κ� +� , κ Ε Ζ . Με επιπλέον: 3χ * κπ (γιατί;) έχουμε: 2 2 4 3 6

εφ2χ · εφ3χ = 1<:::::> εφ2χ =-1- <:::::> εφ2χ =σφ3χ <:::::> εφ2χ = εφ(� -3χ)<=> 2χ=κπ+�-3χ<:::::>χ = κ� +.2: , κ Ε Ζ . εφ3χ 2 2 5 10 ΕΥΚΛΕΙΔΗΣ Β ' 81 τ.l/50

Page 53: Ευκλειδης Β 81

------------ Μαθηματικά για τη Β ' Λυκείου ------------

Χ Ε [-π π] <:::::> -Π S Κ� +� S Π<:::::> -π -� S K� S π -� <:::::> _ Ι Ιπ S κ� S 9Π <:::::> _ _ !_!- < κ < 2_ <:::::> ' 5 1 0 1 0 5 1 0 1 0 5 1 0 2 2 <:::::> κ Ε {-5, -4, -3, -2, - Ι, Ο, Ι , 2, 3, 4} . Να ελέγξετε αν οι αντίστοιχες τιμές του χ είναι ή όχι δεκτές.

� i i 'Εστω : 4ημ2 (χ -�) + 2.J2ημ ( χ -�) = .J2 + 2συν ( 23π - χ) ( Ι ) Παρατηρούμε ότι: συν ( 23π - χ) = ημ ( χ -�) Επομένως: (1) <=:>4ημ2 (χ -�)+2ν'2ημ( χ -�)-2ημ( χ -� )-F2 =0(2) <:::::>

( π) } ημ(χ -�) = t ημ χ -- = t 6 ( π) Ι ( π) -J2 2 6 <:::::> { Ι ν'2 } <=:>ημ χ -6 ="2 (i) ήημ χ -6 =-2- (ii) . Τελικά: 4t + 2(ν'2-ψ -ν'2 =0 t E "2'-2

( i ) <=> χ = 2κπ + � ή χ = (2κ + Ι ) π, κ Ε Ζ και (i i ) <=> χ = 2κπ + Ι ?π ή χ = 2κπ +�, κ Ε Ζ . 3 Ι 2 Ι 2 l: !ιμ ι i ιυ<> η : Η ( 4) έχει δια κρίνουσα: Δ = [ 2 ( .J2 - Ι )2 J - 4 · 4 · ( -.J2 ) = 4 ( 2 - 2.J2 + Ι ) + I 6.J2 =

= Ι 2 + 8.J2 = 4 ( 3 + 2 .J2) = 4 ( .J2 + Ι ) 2 '.i-' ) \,' : Ι ; ;• , f'

Δίνεται το πολυώνυμο Ρ( χ) = χ3 - αχ2 + βχ + 4 . Να υπολογισθούν τα α, β Ε IR έτσι ώστε το Ρ( χ) να έχει παράγοντα το ( χ - 2) 2 • Ν α βρεθεί το πηλίκο της διαίρεσης Ρ( χ) : (χ - 2) 2 •

,\ Για να είναι το (χ - 2 )2 παράγοντας του Ρ(χ) πρέπει και αρκεί Ρ( χ) = (χ - 2 )2 π( χ) , για κάθε

χ Ε IR ( Ι ), όπου π( χ) είναι το πηλίκο της διαίρεσης. Αφού το Ρ( χ) είναι τρίτου βαθμού άρα το π( χ) είναι 1 ου βαθμού : π( χ) = κχ + λ ' Κ, λ Ε IR . Η ( Ι ) γράφεται ισοδύναμα:

Ρ(χ) = ( χ2 - 4χ + 4) (κχ + λ) <:::::> Ρ(χ) = κχ3 + λχ2 - 4κχ2 - 4λχ + 4κχ + 4λ <=:> Ρ( χ) = κχ3 + (λ - 4κ) χ2 + ( 4κ - 4λ) χ + 4λ , για κάθε χ Ε IR (2) Από (2) <=:> κ = Ι , λ - 4κ = -α , 4κ - 4λ = β , 4λ = 4 <=:> κ = Ι, λ = Ι , α = 3, β = Ο. Άρα: π(χ) = χ + l .

Λ σ κ ψπ� z · : Για το πολυώνυμο Ρ( χ) ισχύει Ρ(Ο) = 3 , Ρ(-1) = 13 , Ρ(2) = 1 . Να βρεθεί το υπόλοιπο της διαίρεσης Ρ( χ) : ( χ3 - χ2 - 2χ)

Από την ταυτότητα της διαίρεσης έχουμε Ρ( χ) = (χ 3 - χ 2 - 2χ ) π( χ) + υ( χ) , για κάθε χ Ε IR ( Ι ) όπου π ( χ) είναι το πηλίκο της διαίρεσης και υ ( χ ) το υπόλοιπο , που είναι της μορφής υ(χ) = αχ2 + βχ + γ , α, β, γ Ε IR . Επειδή χ3 - χ2 - 2χ = χ(χ2 - χ - 2) = χ(χ - 2)(χ + Ι) ,(2) η σχέση ( Ι ) γίνεται: Ρ( χ) = χ(χ - 2)(χ + Ι)π(χ) + αχ 2 + βχ + γ , για κάθε χ Ε IR (3),

Οπότε : ::;��)=; : : � + 3 = 1 3 }� ; = �7} . Επομένως υ ( χ ) = 3χ2 - 7χ + 3 . Ρ ( 2) = Ι f 4α + 2β + 3 = Ι γ = 3

; � Δίνεται το πολυώνυμο Ρ( χ) = αχ3 + βχ2 + γχ + 12 . Ο Να βρεθούν τα α, β, γ Ε IR έτσι ώστε το Ρ(χ) να έχει παράγοντα το τριώνυμο r-x-6 και το

υπόλοιπο της διαίρεσης Ρ( χ) : (χ - 4) να είναι υ = 36 .

ΕΥΚΛΕΙΔΗΣ Β ' 81 τ. 1/51

Page 54: Ευκλειδης Β 81

------------ Μαθηματικά για τη Β ' Λυκείου ------------

i i ) Για τις τιμές των α, β, γ του προηγούμενου ερωτήματος να βρείτε το πηλίκο της διαίρεσης Ρ(χ) : ( χ2 - χ - 6)

" ' , , υ ση i ) Για να έχει το Ρ(χ) παράγοντα το τριώνυμο χ 2 - χ - 6 , θα υπάρχει πολυώνυμο π( χ) τέτοιο ώστε

Ρ( χ) = ( χ2 -χ-6)π(χ) , για κάθε χ Ε JR ( 1 ) Αλλά: ,C-x-6=(x-3)(x+2) , οπότε: Ρ(χ) =(χ -3)(χ+2)π(χ) . Άρα:

Ρ(-2) = Ο => -8α + 4β - 2γ + 12 = Ο => -4α + 2β - γ = -6 => β = -4 Άρα: Ρ(χ) = 2χ3 -4χ2 -10χ+ 12 Ρ(3) = 0 } {27α + 9β + 3γ + 1 2 = 0 {9α + 3β + γ = -4 {α = 2 Ρ(4) = 36 64α + 1 6β + 4γ + 1 2 = 36 1 6α + 4β + γ = 6 γ = - 1 0 i i) Εκτελούμε την διαίρεση Ρ(χ) : ( χ 2 - χ - 6) και βρίσκουμε πηλίκο π(χ) = 2χ - 2 .

<::::> χ - 1 = (7 - χ)2 <::::> χ - 1 = 49 - 14χ + χ2 <::::> χ2 - 1 5χ + 50 = 0<::::> χ = 5 ή χ = 10 <::::> χ = 5 Άρα: (II)<::> x=5 . Λσκηση 4' 1 Να λυθούν οι εξισώσεις: ί) J2x2 - 3χ - 5 = x + l (I) ,\ίJση

ί i ) �� + � = .J8 - x (II)

Για να ορίζεται η (I) πρέπει και αρκεί 2χ 2 - 3χ - 5 2 Ο , δηλαδή χ Ε ( -οο, -1 ] υ [% , +οο) υ) Αν χ<- 1 , τότε η (Ι) είναι αδύνατη β) Αν χ 2 - Ι , δηλαδή x E {-1 } u [% , +oo} τότε ( Ι ) <=> (-i2χ2 - 3χ - 5γ = (χ + 1/ <=>

2χ2 - 3χ - 5 = χ2 + 2χ + Ι <::> χ2 - 5χ - 6 = 0 <::::> χ = -1 ή χ = 6 . Τελικά: ( Ι) <::::> χ Ε {- 1, 6}

i i ! �1 + -Γχ=ϊ = .)8 - χ Για να ορίζεται η (11) πρέπει και αρκεί: χ - 1 2 Ο , 1 + -Γχ=Ϊ 2 Ο , και 8 - χ 2 Ο , δηλαδή Ι � χ � 8 . Τότε (I) <=> 1 + -Γχ=ϊ = 8 - χ <=> -Γχ=ϊ = 7 - χ. ( 1 ) .

α) Αν 7 - χ <0 , δηλαδή 7 < χ � 8, τότε η ( 1 ) είναι αδύνατη !Η Α ν 7 - χ 2 Ο , δηλαδή 1 � χ � 7, τότε ( 1 ) <::>χ - Ι =(7 -χ)2 <::>χ - 1=49- Ι4χ +χ2 <::>χ2 -15χ +50=0<::>χ =5 ή χ = lO<::>x =5 Τελικά: (Π)<=> χ = 5 . ί\σκη ση 5 ' 1 Να υπολογίσετε τους α, β συναρτήσει του ν ώστε το πολυώνυμο Ρ(χ) =αχν+' + βχ'' + l

να έχει παράγοντα το { χ + 1 ) 2 •

\.1.)ση Γι αυτό πρέπει και αρκεί το Ρ( χ) να έχει παράγοντα χ + Ι , δηλαδή Ρ( -1) = Ο και το πηλίκο Π( χ) της διαίρεσης Ρ( χ) : (χ + Ι ) να έχει επίσης παράγοντα το χ + Ι , δηλαδή Π( -1) = Ο . Διακρίνουμε τις περιπτώσεις: "' Αν ν άρτιος έχουμε: Ρ(- Ι) = Ο <::> α(- Ι)ν+ Ι + β(-Ι)ν + Ι = Ο <::> -α + β + 1 = 0 ( 1 ) και με σχήμα

Hormer βρίσκουμε : χ ν+ Ι Χ ν Χ ν- Ι

α β ο -α -β + α

ο β - α

0 . . . . . . . . . . . . . . . ο -β + α . . . . . . . . . -β + α

α β - α -β + α β - α -β + α λ

ΕΥΚΛΕΙΔΗΣ Β ' 81 τ. l/52

1 -λ

p = - 1

Page 55: Ευκλειδης Β 81

------------ Μαθηματικά για τη Β ' Λυκείου ------------

Όπου λ = ± (α - β) Οι όροι χ0 , χ ι , χ2 , . . . , χ ν-ι του πηλίκου Π( χ) είναι πλήθους ν, δηλαδή αρτίου πλήθους και επειδή οι συντελεστές τους εναλλάσσονται (+, -, +, -), οι όροι χ0 και χ ν-ι θα έχουν αντίθετους συντελεστές. Άρα λ=α-β, οπότε Π ( χ ) = αχ ν + (β - α) χ ν- ι + ( α - β) χν-2 + . . . . . . . + (β - α) χ + (α - β) και Π(-1) = 0 � α(-1)ν + (β - α) (-1)ν-ι + (α - β) (- Ιγ-2 + . . . . . . . + (β - α ) (-Ι ) + (α - β) = Ο � � α + (α - β) + ( α - β) + . . . + ( α - β) + ( α - β) = Ο � α + ν ( α - β) = Ο � (ν + Ι ) α - νβ = Ο , (2) Λύνοντας το σύστημα των ( Ι ) και (2) βρίσκουμε: α = -ν, β = -ν - Ι • Αν ν περιττός τότε έχουμε: Ρ(-Ι) = Ο � α(-Ιγ+ι + β(-Ι)ν + 1 = 0 � α - β + Ι = Ο (3) και με σχήμα Hormer βρίσκουμε :

α β -α

ο ο 0 . . . . . . . . . . . . . . . ο -β + α β - α -β + α . . . . . . . . . β - α

α β - α -β + α β - α -β + α λ

1 -λ

ρ = -1

Εντελώς όμοια οι όροι χ0 , χ ι , χ 2 , . . . , χν- ι του Π(χ) είναι περιττού πλήθους ν και επειδή οι συντελεστές τους εναλλάσσονται (+, -, +, - , +) , οι όροι χ0 και χν-ι θα έχουν τώρα ίδιους συντελεστές. Άρα λ=β-α, οπότε Π(χ) = αχν + (β - α) χν-ι + (α - β) χν-2 + . . . . . . . + (α - β) χ + (β - α) και Π(-1) = 0 � α(-Ι)ν + (β - α) (- 1γ-ι + (α - β) (- 1γ-2 + . . . . . . . + (α - β ) (- Ι ) + (β - α) = Ο � � -α + (β - α) + (β - α) + . . . . . . . + (β - α) + (β - α) = Ο � -α + ν (β - α) = Ο � � -( ν + Ι ) α + νβ = Ο , (4) Λύνοντας το σύστημα των (3) και (4) βρίσkουμε: α = ν, β = ν + 1 . • Εφ' όσον έχει προηγηθεί το υπόλοιπο Ρ( -Ι) της διαίρεσης, μπορεί να υπολογιστή το λ ευκολότερα

ως εξής. Έχουμε: -λ + 1 = Ρ (- Ι ) (I) Στην πρώτη περίπτωση θα είναι: ( 1 ) � -λ + 1 = -α + β + 1 � λ = α - β και στην δεύτερη : ( 1 ) � -λ + 1 = α + β + 1 � λ = β - α Εφαρμογή : Να βρεθεί το πηλίκο της διαίρεσης Ρ( χ) : ( χ+ 1)2 , όταν Ρ( χ) = (2μ- 1) χ2μ + 2μχ2μ-ι + 1, όπου μεΝ*. ΛίJση Έχουμε: ν = 2μ - Ι, α = ν = 2μ - 1, β = ν + Ι = 2μ, οπότε: β-α= Ι . Και σύμφωνα με τη δεύτερη περίπτωση το πηλίκο της διαίρεσης Ρ ( χ ) : (χ + 1 ) θα είναι:

Π ι (χ ) = ( 2μ - 1) χ 2μ-ι + χ 2μ-2 - χ 2μ-3 + . . . + χ 2 - χ + 1 Εκτελώντας τη διαίρεση Πι ( χ ) : ( χ + Ι ) με το σχήμα Hormer βρίσκουμε ως νέο πηλίκο :

π2 (χ ) = (2μ - 1 ) χ2μ-2 - (2μ - 2) χ2μ-J + 0 0 0 + 3χ2 - 2χ + 1 . Για παράδειγμα αν ν=5 , τότε Ρ ( χ ) = 5χ6 + 6χ5 + Ι , οπότε:

Π ι (χ ) = 5χ5 + χ4 - χ3 + χ2 - χ + 1 και Π2 ( χ ) = 5χ4 - 4χ3 + 3χ2 - 2χ + Ι .

ΕΥΚΛΕΙΔΗΣ Β ' 81 τ. l/53

Page 56: Ευκλειδης Β 81

-------------- Μαθηματικά για τη Β ' Λυκείου --------------

ΜΕτΡΙΚΕΣ ΣΧΕΣΕΙΣ ΣΕ ΤΡΙΓΩΝΟ ΚΑΙ ΚΥΚΛΟ

Α' ΜΕΡΟΣ Δύο κύκλοι με κέντρα Κ και Λ τέμνο­

νται στα σημεία Α και Β. Αν το τμήμα ΚΑ τέ­μνει τον ένα κύκλο στο σημείο Γ και το τμήμα ΛΑ τον άλλο κύκλο στο σημείο Δ και οι Γ Δ και ΚΛ τέμνονται στο σημείο Μ, τότε:

Να αποδείξετε ότι τα ΑΚΔ και ΑΓ Λ είναι όμοια. Να αποδείξετε ότι το τετράπλευρο ΚΓΔΛ είναι εγγράψιμο. Αν είναι ΚΛ=lΟ, ΓΔ=4 και ΔΜ=8 να υπολο­γίσετε το μήκος του τμήματος ΛΜ.

\\I

Τα τρίγωνα ΑΚΔ και ΑΓΛ είναι ισοσκελή γιατί ΑΚ=ΚΔ ως ακτίνες του ίδιου κύκλου και ΑΛ=ΓΛ για τον ίδιο λόγο. Επιπλέον τα ισοσκελή τρίγωνα έχουν την γωνία Α κοινή, επομένως έχουν δύο γωνίες ίσες άρα είναι όμοια.

Από την ομοιότητα των παραπάνω τριγώνων έχουμε την ανα­ΑΔ ΑΚ λσγία - = - από όπου προκύπτει ότι ΑΔΑΛ=ΑΓΑΚ, ΑΓ ΑΛ

συνεπώς το τετράπλευρο ΚΓ ΔΛ είναι εγγράψιμο. Από το εγγράψιμο τετράπλευρο ΚΓ ΔΛ έχουμε ότι:

ΜΔ·ΜΓ=ΜΛ·ΜΚ και αν ονομάσουμε την ΜΛ=χ, από την προηγούμενη ισότητα προκύπτει ότι: 8 · 1 2=χ·(χ+ Ι Ο):::::>χ2 + l Ox-96 =Ο:::::>χ=6

Δίνεται κύκλος (0, R) και μια χορδή του ΑΒ. Αν είναι ΟΜ το απόστημα της χορδής και τα τετράπλευρα Γ ΔΕΖ και ΚΛΡΝ είναι τε­τράγωνα με πλευρές 2α και 2β, τότε:

Να υπολογίσετε την ακτίνα R του κύκλου ως συνάρτηση του αποστήματος ΟΜ=χ και του α Να υπολογίσετε την ακτίνα R του κύκλου ως συνάρτηση του αποστήματος ΟΜ=χ και του β. Α ν το μήκος του τμήματος ΛΔ είναι 4, να βρεθεί το μήκος του αποστήματος χ της χορδής ΑΒ. Έστω Τ, Η τα σημεία που η ΟΜ τέμνει τις ΝΡ και ΖΕ

αντίστοιχα. Αφού η ΟΜ είναι κάθετη στην ΑΒ θα είναι τα

Καρδαμίτσης Σπύρος - Στρατήγης Γιάννης ΟΤ και ΟΗ aποστήματα (άρα Τ, Η μέσα των ΝΡ και ΖΕ). Από το Π. Θ. στα ΟΗΕ και ΟΤΡ προκύπτει:

ΟΗ2 = ΟΕ2 - ΗΕ2 ή (2α - χ )2 = R2 - α2, οπότε, R2=x2 -4αχ+5α2 .

Όμοια έχουμε: ΟΤ2 = ΟΡ2 - ΤΡ2, ή (χ - 2β)2 = R2 - β2, οπότε, R2 = χ2 - 4βχ + 5β2

Από τα παραπάνω έχουμε: χ2-4αχ+5α2=χ2-4βχ+5β2 :::::> :::::>4χ( α+β)=5( α-β)( α+β):::::>χ=5( α-β) Όμως, ΛΜ=pτ=β, ΜΔ=ΗΕ=α και ΛΔ=ΜΔ-ΜΛ=α-β,

' β '

5 οποτε α- =4, τοτε : χ= - · 4=5 . 4

Δίνεται τραπέζιο ΑΒΓΔ με ΑΒ//ΓΔ και ΑΕ η κάθετη στην ΒΓ. Αν οι γωνίες ΔΑΕ, ΕΑΒ και ΔΕΓ είναι ίσες, τότε:

Να δείξετε ότι το ΑΔΕ είναι ορθογώνιο. Αν είναι ΔΓ=3 και ΔΕ=4, να υπολογίσετε το μήκ�ος του τμήματος ΕΒ.

Αν ονομάσουμε τις ίσες γωνίες του σχήματος ω έ­χουμε ότι στο τρίγωνο ΑΔΕ η γωνία ΔΕΑ είναι 90-ω και ΔΕΑ+ΔΑΕ=90-ω+ω=90 επομένως το τρίγωνο ΑΔΕ είναι ορθογώνιο στο Δ.

Φέρνουμε την ΕΗ κάθετη στην ΑΒ, τα τρίγωνα ΑΔΕ και ΑΕΗ είναι ορθογώνια με την ΑΕ κοινή πλευρά και έχουν και από μια γωνία ίση με ω επομένως είναι ίσα, οπότε ΕΗ=ΔΕ=4. Επιπλέον αν φέρουμε την ΔΗ αυτή τέμνει την ΑΕ στο σημείο Κ και τα τρίγωνα ΑΔΚ και ΑΚΗ έχουν την ΑΚ κοινή και από την προηγούμενη ισότητα είναι ΑΔ = ΑΗ.

-"

Β

ΕΥΚΛΕΙΔΗΣ Β ' 8 1 τ. l/54

Page 57: Ευκλειδης Β 81

--------------- Μαθη ματικά για τη Β' Λυκείου ---------------

Επιπλέον έχουν τις περιεχόμενες γωνίες ίσες, άρα τα τρίγωνα είναι ίσα, τότε ΑΚΔ = ΑΚΗ και είναι οι γω­νίες αυτές παραπληρωματικές άρα η ΔΗ είναι κάθετη στην ΑΕ, συνεπώς παράλληλη της ΒΓ. Λόγω της παραλληλίας αυτής το τετράπλευρο ΔΓΒΗ είναι παραλληλόγραμμο, άρα ΗΒ = ΓΔ = 3. Τέλος από Π. Θ. στο ΕΗΒ έχουμε: ΕΒ2 = 32 + 42 => ΕΒ = 5

Δίνεται οξυγώνιο τρίγωνο ΑΒΓ με πλευ­ρές α, β, γ και τα ύψη του ΑΔ, ΒΕ και ΓΖ. Εξωτε­ρικά του τριγώνου κατασκευάζουμε τα τετράγω­να ΑΒΗΘ, ΒΓΙΚ, ΑΓΛΜ. Να δείξετε ότι:

2ΑΒ·ΒΖ+2ΑΓ·ΑΕ+2ΒΓ·ΓΔ=α2+β2-tf Από τη γενίκευση του Πυθαγόρειου θεωρήματος,

στο οξυγώνιο τρίγωνο ΑΒΓ, έχουμε: α2=β2+γ2-2ΑΓ-ΑΕ β2=γ2+α2-2ΑΒ·ΒΖ γ2=α2+β2-2ΒΓΤΔ.

Ι\ Ι

Κ Ι Προσθέτοντας κατά μέλη, έχουμε α2 + β2 + γ2 = 2α2 + 2β2 + 2γ2 - 2ΑΓ-ΑΕ - 2ΑΒ·ΒΖ - 2ΒΓΓΔ από όπου προκύπτει ότι: 2ΑΒ·ΒΖ +2ΑΓ·ΑΕ+2ΒΓ·ΓΔ=α2+β2+γ2

Δίνεται κυρτό τετράπλευρο ΑΒΓ Δ. Α ν οι διαγώνιοι του τέμνονται στο σημείο Ο και Κ, Λ, Μ και Ρ είναι τα έγκεντρα των τριγώνων ΑΟΒ, ΒΟΓ, ΓΟΔ και ΔΟΑ αντίστοιχα, να δείξε­τε ότι: ΚΛ2 + ΜΡ2 = ΡΚ2 + ΜΛ2•

Λόγω των διχοτόμων και των κατακορυφήν γωνιών θα ισχύει: 4(χ + y) = 360 => χ + y = 90 δηλαδή

Α Β οι διαγώνιες ΜΚ και Ρ Λ του τετραπλεύρου ΚΛΜΡ τέ­μνονται κάθετα στο σημείο Ο, συνεπώς σχηματίζονται τέσσερα ορθογώνια τρίγωνα με ορθή γωνία την Ο. Από Π.Θ.έχουμε: ΚΛ2+ΜΡ2=(0Κ2+0Λ2)+(0Μ2+0Ρ2) =

=(ΟΚ2 + ΟΡ2) + (ΟΛ2 + ΟΜ2) = ΡΚ2 +ΜΛ2 . Δίνεται τρί γωνο ΑΒΓ με πλευρές α, β, γ

για τις οποίες ισχύει: α=2γ και β= J7 γ. 'ξ , αf3 Να αποδει ετε οτι μα = --2

Να βρείτε το είδος της γωνίας Β. Αν ΑΕ κάθετη στην ΒΓ να υπολογίσετε το τμήμα ΒΕ ως οονάρτηση του γ. Να υπολογίσετε σε μοίρες το μέτρο της γωνίας Β.

'Ε χουμε β = J7 γ = α J7 επομένως από τον τύπο 2

, , 2 2(βz + γz ) - αz της διαμεσου εχουμε : μα = ---'-----'----

4

Ε Β 1\Ι Γ Από τη γενίκευση του Π.Θ.: για την αμβλεία γωνία

Β θα έχουμε:

β2=α2+γ2+2α ΕΒ=>7γ2=4γ2+γ2+2 ·2γ·ΕΒ=>ΕΒ= l 2

Από το ορθογώνιο τρίγωνο ΑΕΒ έχουμε ότι ΕΒ= l = ΑΒ , 2 2

Λ

άρα η γωνία ΕΒΑ είναι 60°, οπότε Β = 1 20° Δίνεται τετράγωνο ΑΒΓ Δ και τα τόξα

(Β, ΑΓ) και (Α, ΔΒ). Αν η γωνία ΑΕΖ είναι ορθή και ΑΕ = α, ΕΖ = β, ΒΖ = χ, να δείξετε ότι:

χ=�α 2 + 2β 2 - pJ3α 2 + 4β 2

Ονομάζουμε R την πλευρά του τετραγώνου και φ την γωνία ΕΑΒ. Από το ορθογώνιο τρίγωνο ΕΑΖ με

εφαρμογή του Π. Θ. έχουμε: R= �α 2 + β2 ( Ι )

R=Vιi+ μ'

ΕΥΚΛΕΙΔΗΣ Β ' 81 τ. l/55

Page 58: Ευκλειδης Β 81

------------- Μαθηματικά για τη Β ' Λυκείου -------------Στο ισοσκελές τρίγωνο ΑΒΕ φέρνουμε το ύψος του

ΒΗ το οποίο είναι και διάμεσος. Είναι ΑΕΒ=φ και ΒΕΖ=90°-φ. Από νόμο συνημιτόνων στο τρίγωνο ΒΕΖ έχουμε: χ2=R2+β2-2Rβ·συν(90°--φ )::::>χ2=R2+β2 -2Rβ·ημφ (2)

Από το ορθογώνιο τρίγωνο ΑΗΒ έχουμε ότι: J4R2 - α2 "

ημφ= (3) Από τις ( 1 ), (2), (3) : χ2=α2+2β2-β 2R

J3α2 + 4β2 και εφόσον το χ >Ο ως μήκος ευθυγράμ­

μου τμήματος, έχουμε: χ= �α2 + 2β2 - βJ3α2 + 4β2 χ ;·; . Δίνεται τετράγωνο ΑΒΓΔ πλευράς α και

τα τόξα ΑΓ και ΒΔ των κύκλων (Δ, α) και (Α, α).Να δείξετε ότι η ακτίνα R του κύκλου που εφάπτεται στα τόξα ΑΓ, ΒΔ και στην πλευρά ΑΔ είναι ίση με α/6.

Αρχικά από το ορθογώνιο τρίγωνο ΑΚΕ έ-

χουμε ότι: συν(90° - φ) = ημφ = ___!:____ Εφαρμόζουμε α - R

νόμο συνημιτόνων στο τρίγωνο ΑΕΒ και έχουμε: ( α+R)2=( α-R)2+α2 -2·α·( α-R)-συν(90° -φ) ( α+R)2=

=(α-R)2+α2-2·α{α-R)· ___!:____ ::::>(α+R)2=(α-R)2+α2-2·α·R α - R

::::>α2+2αR+R2=α2-2αR+R2+α2-2 ·α·R::::>

Α Β

2 α α - 6αR = Ο::::> α (α-6R) = Ο, με α:;t:Ο R = - . 6

Β ' ΜΕΡΟΣ (/\ ;�·\) ι,,,, Σ . \IJ . Δίνεται ορθογώνιο ΑΒΓΔ (ΑΒ>ΓΔ). Αν ΑΕ και Γ Δ είναι οι κάθετες στην διαγώνιο ΒΔ του ορθογωνίου, τότε:

ΑΒ2 - ΑΔ2 Να δείξετε ότι: ΕΖ = .J

(1) ΑΒ2 + ΑΔ2

f. 'i ; Πότε είναι ΕΖ = ΒΔ ; και πότε ΕΖ = ΕΑ; 3 �'

,;; ;, Από τα ορθογώνια τρίγωνα ΑΒΔ, ΓΒΔ έχουμε ΑΒ2 = ΒΔ·ΒΕ, ΒΓ2 = ΒΔ·ΒΖ αντίστοιχα, συνεπώς ΒΕ = ΑΒ2 ΒΓ2 -- , ΒΖ = -- , από όπου προκύπτει ΒΔ ΒΔ

ΑΒ2 ΒΓ2 ΑΒ2 - ΒΓ2 ΒΕ-ΒΖ=--- ή ΕΖ = (2) . ΒΔ ΒΔ ΒΔ

Αλλά στο ορθογώνιο τρίγωνο ΑΒΔ έχουμε: ΒΔ= J ΑΒ2 + ΑΔ 2 (3), αφού ΒΓ=ΑΔ (από το ορθογώ­

ΑΒ2 - ΑΔ2 νιο ΑΒΓΔ) η (2) σχέση γράφεται: ΕΖ= 1 νΑΒ2 + ΑΔ2

ΒΔ Για να είναι ΕΖ=- οφείλει λόγω της (3) να είναι: 3

�m+M2 ΑΒ2 - ΑΔ2 .J ΑΒ2 + ΑΔ2 ΕΖ = ή ή 3 .JAB2 + ΑΔ2 3 3(ΑΒ2 - ΑΔ2) = ( .J ΑΒ2 + ΑΔ2 ) 2 ή 2ΑΒ2 = 4ΑΔ2 ή

ι;; , ΑΔ · ΑΒ , ΑΒ= \1 L ΑΔ. Ισχύει: ΑΕ·ΒΔ=ΑΔ-ΑΒ η ΑΕ= η ΒΔ

ΑΔ · ΑΒ ΑΕ = (4) από το ορθογώνιο τρίγωνο

.JABz + ΑΔz

ΑΒΔ. Έτσι ΕΖ=ΕΑ ή ΑΒ2 - ΑΔ2 ΑΔ · ΑΒ

JAB2 + ΑΔ2 .JAB2 + ΑΔ2

ή ΑΒ2 - ΑΔ2=ΑΔ-ΑΒ ή (AB J2 - ΑΒ - 1 = 0 ή ΑΔ ΑΔ

ΑΒ 1 + .J5 , , ΑΒ - = -- = Φ η ΑΒ=φ·ΑΔ γιατι - > 0 ΑΔ 2 ΑΔ

Δίνεται ορθογώνιο τρίγωνο ΑΒΓ Λ

( Α = 90° ). Φέρνουμε την ΑΔ κάθετη στην ΒΓ, την ΔΕ κάθετη στην ΑΒ και την ΔΖ κάθετη στην ΑΓ. Να αποδείξετε ότι:

ΒΕ2 + ΕΑ2 + ΑΖ2 + ΖΓ2 = ΒΔ2 + ΓΔ2 και ΕΒ·ΕΑ + ΖΑ·ΖΓ = ΔΒ·ΔΓ.

Το τετράπλευρο ΒΕΖΓ είναι εγγράψιμο . l l l ΒΕ3 + ΓΖ3 = ΒΓ και

(ΔΒ·ΑΓ)2+(ΔΓ·ΑΒ)2=(ΔΑ·ΒΓ)2

: Είναι ΒΕ2 + ΕΑ2 + ΑΖ2 + ΖΓ2 = = ΒΕ2 + ΔΖ2 + ΕΔ2 + ΖΓ2 = = (ΒΕ2 + ΕΔ2) + (ΔΖ2 + ΖΓ2) = ΒΔ2 + ΓΔ2 αφού είναι ΑΕ = ΔΖ και ΑΖ = ΕΔ από το ορθογώνιο ΑΕΔΖ και με εφαρμογή του Π. Θ. στα ορθογώνια τρίγωνα ΒΕΔ, ΓΖΔ.

Β

Γ

ΕΥΚΛΕΙΔΗΣ Β ' 81 τ.l/56

Page 59: Ευκλειδης Β 81

------------- Μαθηματικά για τη Β ' Λυκείου ------------­

Επιπλέον από τα ορθογώνια τρίγωνα ΑΒΔ και ΑΓ Δ έ­χουμε: ΕΒ·ΕΑ+ΖΑ·ΖΓ=ΕΔ2+ΖΔ2= ΕΖ2 = ΑΔ2 = ΔΒ·ΔΓ. β) Στα ορθογώνια τρίγωνα ΑΒΔ, ΑΓΔ είναι: ΑΔ2=ΑΒ·ΑΕ, ΑΔ2=ΑΓΑΖ αντίστοιχα, οπότε ΑΒ·ΑΕ=ΑΓ-ΑΖ, που σημαίνει ότι τε τετράπλευρο με κορυφές τα σημεία Β , Ε, Γ, Ζ είναι εγγράψιμο. J ) Χρησιμοποιούμε μετρικές σχέσεις ορθογωνίου τρι­γώνου. Έχουμε από το τρίγωνο ΑΒΓ γ2=α·ΒΔ και 2 β2 β2=α·ΓΔ ή ΒΔ = L και ΓΔ= - ( 1 ) και από τα ΑΒΔ α α και ΑΓΔ έχουμε: ΒΔ2=γ·ΒΕ και ΓΔ2=β·ΓΖ, συνεπώς εί-

ΒΔ2 ΓΔ2 ναι ΒΕ= -- και ΓΖ= -- και λόγω των σχέσεων ( 1 ) γ β

_ (γ2 : α)2 γ3 (β2 : α)2 β3 έχουμε: ΒΕ- - και ΓΖ= = -γ α2 β α2 (2), επομένως

Εξάλλου ( ΔΒλΓ)'-1{ ΔΓ λ Β)' ':' ( � + ί� · γ J" (� )2 · ( γ2 +β2(�r ( α:α )2 · α2 =( υα · α)2 =(ΔΑ·ΒΓ)2 .

[Μπορείτε να αποδείξετε αυτό το ερώτημα, αν το ση­μείο Δ είναι τυχαίο, στην πλευρά ΒΓ;] . . \ J l . Θεωρούμε κύκλο (O,R) και την εφαπτομένη του, σε διάμετρο ΑΒ στο Α. Πάνω στην εφαπτο­μένη σημειώνουμε σημείο Γ τέτοιο ώστε ΑΓ=R. α) Να συγκριθούν τα τμήματα ΑΒ. ΑΓ, ΒΓ, ΓΔ και ΒΔ, όπου Δ η τομή του κύκλου με την ΒΓ.

2R Να δείξετε ότι ΓΑ+ΓΔ=- · φ και .J5

ΑΔ =

[ΓΔ =

Γ Δ όπου φ= .J5 + 1

ΑΒ νrn ΓΑ 2

Μπορεί ΓΒ·ΓΔ=ΒΔ2; Πότε είναι ΓΔ=φ; . '\ ::,: ) Με το Π.Θ. στο ΑΒΓ είναι: ΒΓ2 = ΒΑ 2 + Γ Α 2

ή ΒΓ2 = (2R)2 + R2 ή ΒΓ2 = 5R2 ή ΒΓ = J5 R ( 1 ) . Η ΑΓ είναι εφαπτομένη του κύκλου, οπότε

ΑΓ2 = ΓΔ·ΓΒ ή R2 = ΓΔ· ( J5 R) ή ΓΔ = J5

R(2) . 5 Άρα ΒΔ=ΒΓ-ΓΔ ή ΒΔ= J5 R-

J5 R ή ΒΔ= ifs R (3) 5 5 ' J5 4J5

Ισχυει - R <R< --R <2 R< JS R γιατί 5 5 ' J52 < 52 < ( 4J5 )2 < 102 < ( sJS )2 ' επομένως είναι: ΓΔ<ΓΑ<ΒΔ<ΒΑ<ΒΓ.

Γ

Β

<2 > J5 R β) Είναι Γ Α + Γ Δ = R + - R = R +- = 5 J5

= R J5 + 1 =

2R . J5 + 1 =

2R J5 J5 2 J5 φ .

Η εγγεγραμμένη γωνία ΑΔΒ είναι ορθή και στο ορθο­γώνιο τρίγωνο ΑΒΓ έχουμε ΑΔ·ΒΓ = ΑΒ-ΑΓ ή

AB · AΓ <1 > (2R) · R 2JS ΑΔ= = -R (4)

ΒΓ J5R 5 , ΑΔ <4> 2J5R : 5 J5 <2 ) ΓΔ

Ειναι - = = - = - και ΑΒ 2R 5 ΓΑ

Μ "�")��5 "Jf � Js � � "�

γ) Θέλουμε ΓΒ·ΓΔ=ΒΔ2 ή ΓΑ2=ΒΔ2 ή ΓΑ=ΒΔ, που δεν μπορεί να συμβεί γιατί από το (α) ερώτημα είναι Γ Α <ΒΔ.

Πρέπει Γ Δ=φ από την (2) J5

R=φ ή R= J5 φ. 5 ,Λ,, J 2 " Δίνεται ΑΒΓ με ΑΒ=5μ, ΒΓ=6μ, Γ Α=4μ, μ>Ο.

Να βρείτε το είδος του, ως προς τις γωνίες του. f;) Να υπολογίσετε τη προβολή της ΑΒ πάνω στην ΒΓ .

Α ν η εφαπτομένη του περιγγεγραμμένου του κύκλου στο Α, τέμνει την ευθεία ΒΓ στο Ε, να βρείτε τα μήκη ΕΓ και ΕΑ. . \.

Β

,η Επειδή ΒΓ=6μ>5μ=ΒΑ, ΒΓ=6μ>4=ΓΑ και ΒΓ2=( 6μ)2=36μ2<4 1 μ2=(5μ)2+( 4μ)2=ΑΒ2+ΑΓ2 είναι

Λ

Α < 1 ορθή . Επομένως το τρίγωνο ΑΒΓ είναι ορθογώ­νιο, αφού η μεγαλύτερη γωνία του Α είναι οξεία. ; \ : Από την γενίκευση του Π. Θ. στο ΑΒΓ, επειδή Β < 1 ορθή έχουμε: ΑΓ2=ΑΒ2+ΒΓ2-2ΒΓ·ΒΔ ή

(4μ)2=(5μ)2+(6μ)2-2·6μ·ΒΔ ή 12μ ΒΔ=45μ2 ή ΒΔ=.!2_ μ ( Ι ) 4 Γ Έστω ΕΓ = χ > Ο. Εφαρμόζουμε το θεώρημα της ο­ξείας γωνίας Β στο τρίγωνο ΑΒΕ. Είναι ΑΕ2 = ΑΒ2 + ΒΕ2 - 2ΒΕ· ΒΔ

ή ΑΕ2 = (5μ)2 + (6μ +χ)2 - 2(6μ + χ) .!2_ μ (2) . 4 ΕΥΚΛΕΙΔΗΣ Β ' 8 1 τ. l/57

Page 60: Ευκλειδης Β 81

------------- Μαθη ματικά για τη Β ' Λυκείου -------------

A)..)JJ. ΕΑ εφαπτομένη του κύκλου, οπότε ΕΑ2=ΕΓΕΒ (3) και

λfyyω της (2) έχουμε: (5μ)2+(6μ +χ)2-2(6μ+χ) � μ=χ-(χ+6μ) 4 1 5 3

ή 25μ2+36μ2+χ2+12μχ-45μ2-- μχ=χ2+6μχ ή 1 6μ2 = - μχ 2 2 , Γ

32 , , 2 32 32 η χ = Ε = - μ. Άρα απο την (3) : ΕΑ =- μ-( - μ+6μ) 3 3 3

1 6 · 4 · 25 40 ή ΕΑ2 = μ2 ή ΕΑ = - μ.

9 3 Θεωρούμε τετράπλευρο ΑΒΓ Δ με ΑΒ=7,

Λ Λ Λ ΒΓ=8, Α = Γ = 90° και Β = 120° Να προσδιοριστεί το μέτρο της διαγωνίου ΒΔ. Να δείξετε ότι: ΑΒ(ΑΓ2-ΑΒ2)=ΒΓ(ΑΓ2-ΒΓ2)

ΑΓ .J3 .J3 · ΒΓ και - = - = --==-----ΒΔ 2 .J3ΑΔ - ΑΒ

Α ν Ε είναι η τομή των ευθειών ΔΓ και ΑΒ, Από το

τρίγωνο ΒΓΕ είναι ΑΒΓ =90°+ Ε ή Ε = 1 20°-90°=30°, επομένως ΒΕ = 2ΒΓ = 2 · 8 ή ΒΕ = 1 6 ( 1 ) . Στο ορθογώ-

Λ

νιο τρίγωνο ΕΑΔ επειδή Ε = 30° είναι ΕΔ = 2ΑΔ (2) και από το Π.Θ. : ΑΕ2 + ΑΔ2 = ΒΕ2 ή (7+ 1 6)2 + ΑΔ2 =

=(2ΑΔ)2 ή 3ΑΔ2 = 232 ή ΑΔ = 23J3 (3) . 3

Με το Π.Θ. στο ΑΒΔ βρί­σκουμε ΒΔ2 = ΑΒ2 + ΑΔ2 =

=72 + 232 · 3 = 2028 ή ΒΔ 32 32

= 2613

(4). 4 Με το θεώρημα αμβλείας

γωνίας στο τρίγωνο ΑΒΓ, ε-πειδή Β = 1 20°> Ι ορθή προ­κύπτει: ΑΓ2=ΑΒ2+ΒΓ2+ 2ΑΒ· ΒΖ=

72+82+2·7 -4= 1 69 ή ΑΓ= Ι 3 (5), γιατί είναι Λ Λ

ΒΓΖ = ΒΕΓ = 30° (οξείες γωνίες με πλευρές κάθετες) και ΒΖ = ΒΓ: 2=4. Έχουμε: ΑΒ(ΑΓ2-ΑΒ2) = 7( 1 32 - 72) = 7 ·6 ·20, ΑΒ(ΑΓ2 - ΒΓ2) = 8( 1 32 - 82) = 8 · 5 ·2 1 = 7 ·6 ·20 = ΑΒ(ΑΓ2 - ΑΒ2) και ΑΓ 1 3 13 13 · ΒΓ <J J 13 · 8 13 = = -- - -

ΒΔ 2613 : 3 2 ' 13 · ΑΔ - ΑΒ - 27 · 3 2 θεωρούμε τρίγωνο ΑΒΓ και ΒΔ.lΑΓ. Να

αποδείξετε ότι: ΑΔ · ΑΓ = μ; - ( i) 2 ΑΔ · ΑΓ < β·γ.

α2 + β2 + γ2 ΑΔ·ΑΓ+ΒΓ·ΒΑ+ΓΕ·ΓΒ= σε οξυ-2

γώνιο τρίγωνο με ΑΕ .l ΒΓ, ΓΖ .l ΑΒ.

Β

Διακρίνουμε περιπτώσεις για την γωνία Α. Λ

Αν Α < 1 ορθή . Από το θεώρημα οξείας γωνίας στο τρίγωνο ΑΒΓ είναι: α2=β2+γ2-2β· ΑΔ ή 2ΑΓ· ΑΔ = β2+γ2-α2 ( 1 ) Επίσης από το 1 ° θεώρημα διαμέσων στο τρίγωνο

2 2(β2 + 2 ) - α 2 ΑΒΓ είναι μ = γ α 4

(α )2 β2 + γ 2 - α2 ή μ� - "2 = 2 (2), συνεπώς από τις

ισότητες ( l ) και (2) βρίσκουμε: ΑΓ ΑΔ= � -( � )2 (3) .-\

Γ Λ

Αν Α > 1 ορθή . Από το θεώρημα αμβλείας γωνίας στο τρίγωνο ΑΒΓ είναι:

α2 = β2 + γ2 + 2β· ΑΔ ή 2ΑΓ· ΑΔ = - (β2 + γ2 - α2) και

λόγω της (2) 2ΑΓ ΑΔ = - 2[ μ� - (�)2 ]

ή ΑΓ· ΑΔ = - [ μ� - ( � )2 ] (4)

Αν Α = 1 ορθή , είναι ΑΔ = Ο, � = α και ΑΓ-ΑΔ 2 = Ο = μ� - ( � )2 (5), επομένως από τις ισότητες

(3),(4),(5) είναι ΑΓ ΑΔ= μ� - ( � )2 Από το (�) ερώτημα είναι:

ΑΔ-ΑΓ= μ� - (�)2 =Ι β2 + γ� - α2 1 ·

Ι β2 + 2 _ α2 1 Άρα πρέπει να δείξουμε γ2 <β·γ

ή l β2+γ2 - α2 1 <2β ·γ ή -2β·γ < β2 + γ2 - α2 < 2β·γ ή (α2 < β2 + γ2 +2β·γ και β2 + γ2 -2β·γ < α2 ) ή [α2 <(β+γ)2 και (β-γ)2<α2 ] ή α<β+γ και l β-γl<α, που αληθεύουν (τριγωνική ανισότητα στο τρίγωνο ΑΒΓ) .

Για το οξυγώνιο τρίγωνο ΑΒΓ, προκύπτουν οι ανά­λογες σχέσεις: 2ΒΑ·ΒΓ=γ2+α2-β2 (6), 2ΓΒ·ΓΕ=α2+β2-γ2 (7) και με πρόσθεση κατά μέλη

των ( 1 ), (6) και (7) έχουμε: 2(ΑΓΑΔ+ΒΑ·ΒΖ +ΓΒ·ΓΕ)=α2+β2+γ2 .

ΕΥΚΛΕΙΔΗΣ Β ' 81 τ.l/58

Page 61: Ευκλειδης Β 81

------------- Μαθηματικά για τη Β ' Λυκείου -------------

Β � λΥΚΕΙΟΥ

Εφαρμογές στην Θεωρητική Γεωμετρία

Σε ένα ισοσκελές τρίγωνο ΑΒΓ με βάση ΒΓ, το τμήμα ΑΔ είναι διχοτόμος. Να δείξετε διανυσματικά ότι είναι ύψος και διάμεσος. ί. Α ν σε ένα τρίγωνο ΑΒΓ το τμήμα ΑΔ είναι

διάμεσος και ύψος, να δείξετε διανυσματικά ότι το τρίγωνο είναι ισοσκελές.

ί. Για να είναι το ΑΔ ύψος, αρκεί ΑΔ ..l ΒΓ , δη λαδή ΑΔ · ΒΓ = Ο ή ΑΔ · (ΑΓ - ΑΒ) = Ο , - - - - - - Α Μ·ΑΓ=Μ·ΑΒ. Πράγματι: Μ·ΑΓ=υ· β ·συν- , 2 - - Α Α ΑΔ · ΑΒ = υ · γ · συν- = υ · β · συν-2 2

Α I 2

γ β υ

Β Δ Γ Για να είναι το ΑΔ διάμεσος , αρκεί

IBΔI = IΔΓI , ή 1�2 =1&12 , ή (ΑΔ-ΑΒγ = (ΑΓ-ΑΔ)2 , ή

- - - _ , υ2 + γ2 - 2ΑΔ · ΑΒ = β2 + υ2 - 2ΑΓ · ΑΔ- , ή ΑΒ · ΑΔ = ΑΓ · ΑΔ που το δείξαμε πριν.

- -- ΑΒ + ΑΓ - -ii. Έχουμε: ΑΔ = 2 ( 1 ) και ΑΔ · ΒΓ = Ο (2).

ΑΒ + ΑΓ (- -) Από ( 1 ) και (2) έχουμε 2 ΑΓ - ΑΒ = Ο => ΑΓ2 - ΑΒ2 = 0 ::::> β2 - γ2 = 0 => β = γ .

Δίνεται τρίγωνο ΑΒΓ και το ύψος του -2 - - Α π ΑΔ. Να δείξετε ότι: ΑΔ = -ΔΒ · ΔΓ <=> Α = -

2 Σε κάθε τρίγωνο ΑΒΓ έχουμε:

Δ:λ Β Δ Γ ΑΔ2 = ( ΑΒ + ΒΔ ) ( ΑΓ + ΓΔ) = ΑΒ · ΑΓ + ΑΒ · Γ Δ + ΑΓ · ΒΔ + ΒΔ · Γ Δ = ΑΒ · ΑΓ +προβrΔ ΑΒ · ΓΔ+ προβ8Δ ΑΓ · ΒΔ+ ΒΔ · ΓΔ

- - - - - - - -= ΑΒ · ΑΓ + ΔΒ · Γ Δ + ΔΓ · ΒΔ + ΒΔ · Γ Δ = ΑΒ · ΑΓ - ΔΒ · ΔΓ - ΔΓ · ΔΒ + ΔΒ · ΔΓ

Α ν αστασία Φανέλη = ΑΒ · ΑΓ - ΔΒ · ΔΓ , οπότε -2 - - - - - π ΑΔ = -ΔΒ · ΔΓ <=> ΑΒ · ΑΓ = Ο <=> Α = -2

Αν το τρίγωνο ΑΒΓ είναι ορθογώνιο με , ΒΓ , δ 'ξ , 1 1 1 υποτεινουσα , τοτε να ει ετε οτι: 2 = 2 + .:z

υα β r ,; ' '' � Έστω ΒΔ = λ · ΒΓ ( 1 ) . Τότε ( 1 )

::::>Μ-ΑΒ=λ(ΑΓ -ΑΒ) => ΑΔ = λΑΓ + ( 1 - λ) · ΑΒ - - -2 - - -2 => ΑΓ · ΑΔ = λ · ΑΓ και ΑΒ · ΑΔ = ( \ - λ) · ΑΒ ,

- - - - -2 αφού ΑΒ · ΑΓ = Ο . Αλλά ΑΓ · ΑΔ = ΑΔ , ΑΒ · ΑΔ = ΑΔ 2 (προβολές επί της ΑΔ ) . Άρα

υα2 = λ - β2 } �2 = υ�2 ) ι Ι Ι υα2 = ( 1 - λ) · γ2

=>_!_ =

Ι -λ => β2 + γ 2 = υα2 β2 υα2 Σε ένα ισοσκελές τρίγωνο ΑΒΓ με βάση

ΒΓ, οι διάμεσοι ΒΝ και ΓΜ τέμνονται κάθετα. ί. Ν α .υπολογιστεί η γωνία Α .

ί.

ίί. Να δείξετε ότι ΒΝ2 +rM = ΑΙ<! όπου ΑΚ διάμεσος. ίίί. Να δείξετε ότι ΑΒ2 + ΑΓ2 = 5ΒΓ2

Α

Αρκεί λοιπόν να υπο-λογίσουμε το γινόμενο ΑΒ . ΑΓ ως 8 r

συνάρτηση του β . Παρατηρούμε ότι ΒΝ ..l ΓΜ => ΒΝ · ΓΜ = Ο => ( ,ΔΝ-ΑΒ) ( ΑΜ-ΑΓ) =0 => ( �

-ΑΒ Ι�

-ΑΓ )=ο => ΑΒ� ΑΓ �2 �

+ΑΒ· ΑΓ =Ο=>

�β2 5ΑΒ·ΑΓ=4β2 => ΑΒ·ΑΓ=�� ( 1 ) . Άρα συνΑ=� =� . 5 β 5

2 2 ΑΓ - ΑΒ -(- )2 (- )2 ίί. ΒΝ + ΓΜ = 2- ΑΒ + 2 - ΑΓ =

β2 γ2 - - β2 4β2 9β2 -+-+ β2 +γ2 -2ΑΒ · ΑΓ =- + 2β2 -2 · - = -4 4 2 5 1 0

AKz =(AB;Af )2

= � (βz + / + 2ΑΒ ·ΑΓ) = ΕΥΚΛΕΙΔΗΣ Β ' 81 τ. l/59

Page 62: Ευκλειδης Β 81

------------- Μαθηματικά για τη Β ' Λυκείου -------------β2 1 - - β2 1 4β2 9β2 -+-ΑΒ · ΑΓ =- +- ·- =- . 2 2 2 2 5 1 0 Άρα ΒΝ2 + ΓΜ2 = ΑΚ2

ίίί. Αρκεί 2β2 = 5α2 , ή λόγω της ( 1 ), 5 - - 2 ' ' 2 · 4ΑΒ · ΑΓ = 5α , η τελος ΑΒ · ΑΓ = 2α 2 Για

να υπολογίσουμε το γινόμενο ΑΒ · ΑΓ ως συ­νάρτηση του α , παρατηρούμε ότι: ΒΝ ΤΜ = Ο => {ΓΝ - ΓΒ ) (ΒΜ - ΒΓ) = Ο =>

=> (-� + ΒΓ J(-� - ΒΓ) = Ο =>

- - -2 ΑΒ ΑΓ -2 ΒΓ - -. ΒΓ +-- = 0 => ΑΒ · ΑΓ = 2α2 4 2 Λ5. Δίνεται τραπέζιο ΑΒΓ Δ με Α = Δ = �

2

και Μ το μέσο της πλευράς ΑΔ. Α ν ΒΜΓ = � να 2

υπολογιστεί με τη χρήση συντεταγμένων το ύ­ψος του τραπεζίου ως συνάρτηση των βάσεων του Γ Δ=α και ΑΒ=β.

Λύση : Θεωρούμε ορθοκανονικό σύστημα xAy με τα Β,Δ επί των θετικών ημιαξόνων Αχ, Ay αντιστοίχως. Τότε οι συντεταγμένες των κο-ρυφών είναι: Α ( Ο, Ο) , Β (β, Ο) , Γ ( α, υ)

χ

Υ

Δ(Ο,υ

Μ

Α(Ο,Ο) Ο Β(β,Ο) χ

y'

και Δ (Ο, υ) όπου υ το μήκος του ύψους. Επομέ­νως για το μέσο Μ της ΑΔ έχουμε Μ (Ο,�) , ο-πότε : ΒΜΓ = � => 2 ΒΜ · ΜΓ = Ο => ( 0 - β,� - 0 }(α - Ο, υ - �) = Ο =>

( -13,�)( ��)=ο => -13α+ � =0 => υ2 =4αβ => υ=l[cψ. Λ6. Δίνεται τετράγωνο ΑΒΓΔ. Αν Ε εσωτε­

ρικό και Ζ εξωτερικό σημείο του τετραγώνου, ώστε τα τρίγωνα ΑΒΕ και ΒΓΖ να είναι ισό­πλευρα, να δείξετε διανυσματικά ότι τα σημεία Δ, Ε, Ζ είναι συνευθειακά.

Λ \ι ση : Αρκεί λοιπόν να δείξουμε ότι τα διανύ­σματα u = ΔΕ = ΑΕ - ΑΔ και � = ΔΖ = ΓΖ - ΓΔ εί­ναι παράλληλα και μάλιστα ομόρροπα, δηλαδή αρκεί U · � = lu l · l �l ( 1 ) .

Πράγματι: u · ν =ΑΕ ·ΓΖ-ΑΕ· ΓΔ-ΑΔ·ΓΖ+ΑΔΤΔ .

Αλλά ΑΕ · ΓΖ = Ο διότι Η + Γ = 60° + 30° = 90° I ' 1 1

=> ΑΕ ..l ΓΖ . Επίσης ΑΔ ΤΔ = Ο . Άρα U · V = -ΑΕ · ΓΔ - ΑΔ · ΓΖ = -α2συνω - α2συνφ z = -α2συνl20° -α2συν120° Α = 2α2συν60° = α2 • Και �---�Β lul 2 =ΑΕ2 +ΑΔ2 -2ΑΕ·ΑΔ = 2α2 -2α2συν30° =α2 (2-f3) , ��� 2 = ΓΖ2 + ΓΔ2 - 2ΓΖΤΔ = 2α2 -2α2συν15σ' =α2 ( 2+f3) => lul2 · 1 �1 2 = α4 ( 4-3) = a4 => lul · l �l = a2 • Άρα ισχύει η ( 1 )

ι\7. Δίνεται τετράπλευρο ΑΒΓΔ με λ λ π Α = Γ = - . Αν Κ και Λ είναι τα μέσα των δια-

2 γωνίων του ΑΓ και ΒΔ αντιστοίχως, να δείξετε με την χρήση συντεταγμένων και διανυσματικά ότι: ΚΑ ..l ΑΓ .

Λ ίJση : Θεωρούμε ορθοκανονικό σύστημα συ­ντεταγμένων με αρχή των αξόνων το Α και άξονα χ 'χ επί της ευθείας ΑΒ , οπότε ο άξονας y'y θα είναι επί της ευθείας ΑΔ .

Τότε οι συντεταγμένες των κορυφών είναι: Α(Ο, Ο) , Β (β, Ο) , Γ (μ, ν) και Δ (Ο, δ) . Επομένως: ΒΓ ..l ΓΔ ::::> ΒΓ ΤΔ = Ο => (μ - β, ν) (-μ, δ - ν) = Ο => ( μ - β) (-μ) + ν (δ - ν ) = Ο => -μ2 + μβ + νδ - ν2 = 0 => μ2 - μβ - νδ + ν2 = 0 ( 1 )

'Εχουμε : κ(� ·Ξ) και Λ (% ·%) . Επομένως - ( μ ·- β ν - δ ) ΛΚ = -- -- . 2 ' 2

Υ Γ(μ,ν)

y'

, - - ( μ - β ν - δ ) Εχουμε ΛΚ · ΑΓ= -2- ,-2- (μ, ν) = μ(μ-β) ν(ν-δ) μ2 - μβ+ν2 -νδ ( ι )

=--+--= = Ο ::::> ΚΛ ..l ΑΓ . 2 2 2

Δ ιανυσματι κά ( β τριΊπος ) : Αρκεί ΚΛ · ΑΓ = Ο Έχουμε:

ΚΛ=ΚΑ+ΑΒ+ΒΛ) - - - - ΑΒ+ΓΔ _ _ _ _ ::::> 2ΚΛ= ΑΒ+ΓΔ ::::> ΚΛ =---ΚΛ=�+�+Μ 2

Επομένως ΚΛ · ΑΓ = � (ΑΒ + Γ Δ) ( ΑΒ + ΒΓ)

= � (ΑΒ2 + ΑΒ · ΒΓ + ΓΔ · ΑΒ + ΓΔ · ΒΓ)

= �ΑΒ (ΑΒ + ΒΓ + ΓΔ) = �ΑΒ · ΑΔ = Ο

ΕΥΚΛΕΙΔΗΣ Β ' 8 1 τ. l/60

Page 63: Ευκλειδης Β 81

ΜΑΘΗΜΑΠΚΑ ΓΕΝΙΚΗΣ ΠΑΙΔΕΙΑΣ r · ΛΥΚΕΙΟΥ

Διαφορικός Λογισμός Παπαχρήστος Σωτήριος

Ι : Δίνεται η συνάρτηση f : R � R μέ τύπο f (χ) = χ3 - 6χ2 + 9χ + α, όπου α παράμετρος στο R. ' α. Να βρεθούν η πρώτη και η δεύτερη παράγωγος της f.

β. Να μελετηθεί η f ως προς τη μονοτονία και να δείξετε ότι παρουσιάζει ένα τοπικό μέγιστο και ένα τοπικό ελάχιστο.

γ. Αν το τοπικό μέγιστο της f είναι τριπλάσιο από το τοπικό ελάχιστό της, να βρεθεί ο αριθμός α. δ. Να βρεθεί η ελάχιστη τιμή του ρυθμού μεταβολής της f .

Λt)ση α. f ' { x ) = 3x2 - Ι 2χ + 9, f " { x ) = 6x - I 2 , β. f ' { x ) = 0 � 3x2 - I 2x + 9 = 0 � x = I ή x = 3 (* J f ' { x ) > 0 � 3x2 - I 2x + 9 > 0 � x < I ή χ > 3 , f' { x ) < O � I < x < 3

χ -00 Ι 3 + οο f' { χ ) + ο - ο +

f { x ) γν. αύξουσα γν. φθίνουσα γν. αύξουσα Η f είναι γν. αύξουσα στο (---<ς Ι] , γν. φθίνουσα στο [ Ι , 3 ] και γν. αύξουσα στο [3 , +οο) . Παρουσιά­

ζει τ. μέγιστο στο Χη =Ι και τ. ελάχιστο στο χ1 =3 . γ .. Παρατηρούμε ότι: f(I) =3f(3) ::::}α+4=3α::::}α=2. {Ι , f " { x ) = 0 � 6x - I 2 = 0 � x = 2 . Αν f " {x ) > 0 � 6x - I 2 > 0 � x > 2 , f " {x ) < O � x < 2 .

χ -00 2 + οο f" { χ ) - ο + f' { χ ) γν. φθίνουσα γν. αύξουσα

Η f ' είναι γν. φθίνουσα στο ( -οο, 2] και γν. αύξουσα στο [2 , +οο) Παρουσιάζει ελάχιστη τιμή στο χ0 = 2 η οποία είναι ίση με f ' {2 ) = -3 .

2 . Δίνεται η συνάρτηση f με τύπο r( χ) = χ; 1 . α. Να βρεθούν η πρώτη και η δεύτερη παράγωγος της f. β. Να δείξετε ότι e'f' (χ) + e•r" (χ) = -1 .

γ. Να βρεθεί η εξίσωση εφαπτομένης της Cr στο σημείο της Μ { Ο, r (o)) . δ. Ν α βρεθεί η μέγιστη τιμή της f και να δείξετε ότι f (χ) � 1, για κάθε χ ε R. . ' { ) e' - ( x + I ) e' χ f" { x ) = e' - xe' χ - Ι Λυση υ. f χ = e2x = e' ' e2x = e' χ Ε R .

'f ' { ) 'f" { ) χ -χ χ χ - Ι β. e χ + e χ = e ·-+ e · -- = -χ + χ - Ι = - 1 . ex ex γ. Έστω (ε) : y= λχ + β η εφαπτομένη της Cr στο σημείο της Μ ( Ο, f { O)) <* *) .

(*J Σημείωση : Ο μηδενισμός της παραγώγου στο χ0=3 δεν είναι απαραίτητος, αλλά ούτε καν η ύπαρξη της. Αυτό που χρειάζεται είναι η συνέχεια της f στο χ0 και η αλλαγή προσήμου της Γ στο χ0. Με αυτά μόνον θα ασχοληθούμε στα επόμενα παραδείγματα. <**) Η ύπαρξη της εφαπτομένης της Cr στη θέση χ0 με κλίση λ=Γ(χ0) είναι εξασφαλισμένη όταν η f παραγωγίζεται στο χ0 , όπως στα παραδείγματα ακολουθούν. Γι αυτό δεν θα αναφέρεται ρητά σε κάθε περίπτωση.

ΕΥΚΛΕΙΔΗΣ Β ' 81 τ.1/61

Page 64: Ευκλειδης Β 81

------------ Μαθηματικά για τη Γ ' Λυκείου ------------

Είναι λ = f1 {0) = - � = 0 . Επίσης Μ Ε (ε) <:::::> f {0) = 0 + β <=:> β = 1 . Συνεπώς (ε) : y= 1 . e

e·� >0 'Εχουμε: f 1 { χ ) > Ο <:::::> -� > Ο <:::::> χ < Ο , f 1 { χ ) < Ο <:::::> χ > Ο και f συνεχής στο χ0=0. e•

Η f είναι γν. αύξουσα στο ( -οο, Ο] και γν. φθίνουσα στο (0 , +οο) . Παρουσιάζει μέγιστη τιμή στο χ0 = Ο η οποία είναι f {O) = 1 . Θα ισχύει λοιπόν f { χ ) � f { 0) , δηλαδή f { χ ) � 1, για κάθε χ Ε R . Δ, , r , r ( ) χ2 - 3χ + 2 ινεται η συναρτηση με τυπο χ = . χ - 1

Να βρεθεί το πεδίο ορισμού της f. Να βρεθεί η f1 (χ) . Να βρεθεί το όριο lim f (χ) ·�· Να βρεθεί ο πραγματικός αριθμός λ ώστε να ισχύει: λ 2f1 ( 20ι2) - 2λf ( 3) + ι = Ο.

f (3 + h) - ι Να βρεθεί το όριο lim . h�O h Για να ορίζεται η f(x) πρέπει και αρκεί χ - 1 1= Ο, δηλαδή χ 1= 1 . Άρα A=R- { 1} .

χ2 - 3χ + 2 (χ - 1 ) ( χ - 2) 1 Παρατηρώ ότι: f (χ ) = = = χ - 2 . Συνεπώς f1 ( χ ) = (χ - 2) = 1 . χ - 1 χ - 1 Iim f (χ ) = Iim ( χ - 2) = 1 - 2 = - 1 . Είναι f1 ( 20 12 ) = Ι , f ( 3) = 3 - 2 = 1 . Επομένως: Χ --> 1 Χ --> 1

λ 2f1 ( 20 12 ) - 2λf ( 3) + Ι = Ο <:::::> λ 2 · 1 - 2λ · 1 + 1 = Ο <:::::> (λ - 1 γ = Ο <:::::> λ = 1 Iim f { 3 + h) - Ι = Iim f { 3 + h ) - f { 3) = fl { 3) = 1 . h �O h h � O h

Δίνονται οι συναρτήσεις f, g : R ---+ R με f (χ) = χ2 - Sx + 6, g ( χ) = χ2 - 2χ.

Δείξτε ότι f1 ( χ) = g1 (χ) - 3 για κάθε χ Ε R . Να βρεθεί το όριο lim f ((x )) ·�2 g χ

Να βρεθεί η πρώτη παράγωγος της συνάρτησης h( χ) =:�:� για κάθε χ του πεδίου ορισμού της. Ν α βρεθεί η παράγωγος της συνάρτησης φ (χ) = f { g (χ)) για χ = 2 . Να βρεθεί η τιμή A= lOOO · f" (200) - 22 · g' ( �) . Ν α βρείτε την εξίσωση εφαπτομένης της Ch στο σημείο της Μ ( ι, h (ι)) .

f1{ χ) =2χ-5 , gl( χ) =2χ -2 . Είναι λοιπόν, gl ( χ) - 3 = 2χ - 2 - 3 = 2χ - 5 = f1{ χ) , για κάθε χ Ε R. , f { x ) χ2 - 5χ + 6 ( χ - 2) { χ - 3) χ - 3 , . f { x ) . χ - 3 1 Εχουμε: -{ ) = 2 = ( ) =- , οποτε Ιιm-( ) = Ιιm-= -- . g Χ Χ - 2χ Χ Χ - 2 Χ Η2 g Χ Χ --> 2 Χ 2 Για να ορίζεται η h(x) πρέπει και αρκεί g ( χ ) 1= Ο δηλαδή χ 1= Ο και χ 1= 2 .

Τότε έχουμε h' ( x ) = (:�:�J ' = ( ( x:(2J�x2) 3) ) ' = ( χ: 3 ) ' = χ -��- 3) = :, . I Ισχύει ψ1 ( x ) = (f (g ( x))) = f1 (g (x )) g1 ( x ) , για κάθε x E R.

Επίσης είναι: g (2) = 4 - 4 = 0, g1 ( 2) = 4 - 2 = 2 και f {0) = 6 . Συνεπώς: ψ1 {2 ) = f1 (g (2) ) · g1 ( 2 ) = f1 {0) · 2 = -5 · 2 = - 10 . Είναι f" (χ ) = 2 , για κάθε χ Ε R , οπότε: f" ( 200) = 2 . Είναι λοιπόν:

ΕΥΚΛΕΙΔΗΣ Β ' 81 τ.l/62

Page 65: Ευκλειδης Β 81

------------- Μαθηματικά για τη Γ ' Λυκείου -------------

A= 1 000 · f" ( 200) - 22 · g( (%) = 1000 · 2 - 22 ( 2 · % - 2) = 2000 - 22 · ( -�) = 2000 + 1 1 = 20 1 1 . Έστω (ε) : y= λχ + β η εφαπτομένη της Ch στο σημείο της Μ ( 1, h ( 1 )) . Είναι λ = h ' ( 1 ) = � = 3 . Επίσης Μ ε (ε) � h (1 ) = 3 · 1 + β � 1 � f ((

1 )) = 3 + β � � = 3 + β � β = -5 . Συνεπώς είναι (ε) : y= 3x - 5 . g 1 - 1

χ Δίνεται η συνάρτηση f με τύπο f (χ) = !.._ .

χ Να βρεθεί η πρώτη παράγωγος της f. Να μελετηθεί η f ως προς τη μονοτονία. Ν α δείξετε ότι f ( �) < f ( �) < f ( �) . f (α) + f (β) + f (γ) Α ν α, β , γ > Ο να δείξετε ότι ;;:: e . 3

χ χ ex ( χ - Ι ) Για να ορίζεται η f(x) πρέπει και αρκεί χ =1- Ο . Τότε έχουμε: f' ( χ ) = e · χ 2- e = 2 • χ χ

'Εχουμε: f' ( χ ) > Ο � ex ( \- 1) > Ο χ '>�χ ..

ο ex (χ - 1 ) > Ο� χ - Ι > Ο � χ > 1 , f' ( χ ) < Ο � χ < 1 και χ f συνεχής στο χο= 1 . Η f είναι γν. φθίνουσα στο ( -οο, Ο) , γν. φθίνουσα στο (Ο , 1] και γν. αύξουσα στο [ 1, +οο) . Παρατηρώ ότι Ι >� >� >� >0 και η f είναι γν. φθίνουσα στο (Ο , Ι) . Θα είναι f( �) < f( υ < f( �} Παρατηρώ ότι στο (Ο, + οο ) η f παρουσιάζει ελάχιστο στο χ0 = Ι το f ( Ι ) = e . Θα ισχύει λοιπόν

f ( χ ) ;;:: e, για κάθε χ > Ο. Αφού λοιπόν είναι α, β, γ >Ο θα έχω:

f (β) ;;:: e .;;) f (α ) + f (β) + f (γ) ;;:: 3e => f (α) + f �) + f (γ) ;;:: e , που είναι και το ζητούμενο. {f ( α) ;;:: e f ( γ) ;;:: e Δίνεται η συνάρτηση f : R � R με τύπο, f (χ) = αχ 2 + β χ + 2 όπου α, β παράμετροι στο R.

Να βρεθεί η f' (x ) .

Αν είναι lim f ( χ) = 2 και η εφαπτομένη (ε) της Cr στο σημείο της Μ(� , r (�)) σχηματίζει με χ�2 2 2 τον άξονα χ ι χ γωνία ω = 45° ' να βρεθούν οι αριθμοί α, β.

Για α = 1 και β = -2 , να βρεθεί η εξίσωση της (ε). f' (χ) f" ( χ) 2f (χ) Για α = Ι και β = -2 , να δείξετε ότι f" ( x) + f' (x ) = f' (x ) .

f' (χ) = 2αχ + β . Γνωρίζω ότι: limf( χ) = 2, οπότε: Χ-+2 lim( αχ2 +βχ + 2) = 2=>4α+ 2β + 2 = 2=> 2α+β=Ο (Ι). Γνωρίζω επίσης ότι η εφαπτομένη της cf στο Χο = � Χ-+2 2 σχηματίζει με τον άξονα χ ' χ γωνία � = 45° , οπότε: f' (%) = εφ45 => 2α · % + β = 1 => 3α + β = Ι (2) . Λύ­νοντας το σύστημα των ( 1 ) και (2) βρίσκουμε α = 1 και β = -2.

Για α = Ι και β = -2 έχουμε: f (x ) = x 2 - 2χ + 2 και f(x) = 2x - 2 .

ΕΥΚΛΕΙΔΗΣ Β ' 81 τ. l/63

Page 66: Ευκλειδης Β 81

--------�--- Μαθη ματικά για τη Γ Λυκείου ------------'Εστω (ε) : y= λχ + κ η εφαπτομένη της Cr στο σημείο της Μ ( % • r (%)) . Προφανώς λ = εφ45 = 1 . Επίσης Μ ε (ε) => r (%) = 1 · % + κ => => � - 2 . l + 2 = l + κ => � - Ι = l + κ => κ = _.!_. Συνεπώς είναι (ε) : y= x _.!_ . 4 2 2 4 2 4 4

δ .. Για α = Ι και β = -2 είναι επίσης f' ( x ) = 2x - 2 = 2 ( x - I ) και f" ( x ) = 2 . , f'(x) f"(x) 2(χ-1) 2 1 χ2 - 2χ + 2 f ( x ) f ( x ) 2f (x ) Ειναι· --+--=-- +-- =χ-1+-= = =--=--. f"(x) f'(x) 2 2(χ-1) χ-1 χ - 1 2 ( χ - Ι ) f' ( x ) f' ( x ) ·

2 2 7 . Δίνεται η συνάρτηση f : R � R με τύπο r(x) = χ2 - αχ + Ιοgβ , όπου α, β παράμετροι στο R με β>Ο.

π .. Να βρεθεί η f' (χ) . ,�ι Αν η f παρουσιάζει ελάχιστο στο χ0 = 2 , να βρεθεί ο αριθμός α. γ, Αν α = 4 και η ελάχιστη τιμή της f είναι ίση με -ι , να βρεθεί ο αριθμός β. -:ι Α ν α = 4 και β = ι 000 , να βρεθεί εφαπτομένη της Cr παράλληλη προς την ευθεία y= 2χ .

ο . f' ( χ ) = 2χ - α. β . 'Ε χουμε: f' ( χ ) > Ο <::::> 2χ - α > Ο <::::> χ > � , f' ( χ ) < Ο <::::> χ < � και f συνεχής στο 2 2 χ0 = � . Η f είναι γν. φθίνουσα στο ( -οο, �] και γν. αύξουσα στο [�, -too) . Παρουσιάζει ελάχιστη τιμή

α , α 2 4 στο χ0 = - , οποτε: - = =>α= . 2 2 γ. Για α=4 η f γίνεται: f(x) = x2 -4x + logβ . Η ελάχιστη τιμή της f είναι f{2) = 4 -8 + logβ =--4+ logβ ,

οπότε έχω: -4 + log β = - Ι => logβ = 3 => β = 1 000 και f ( χ ) = χ 2 - 4χ + 3 . Έστω Μ (χ0 , f ( x0 ) ) το σημείο επαφής και (ε) : y= λχ + γ η εφαπτομένη της Cr στο Μ. Για να εί­

ναι η (ε) παράλληλη y= 2x πρέπει και αρκεί λ=2 και γ:;t:Ο. Έχουμε : λ = 2 <::::> f' ( x0 ) = 2 <::::> 2χ0 - 4 = 2 <::::> χ0 = 3 . Άρα f ( x0 ) = f (3) = 9 - 1 2 + 3 = Ο , οπότε Μ (3, Ο) και Μ ε (ε) <=> 0 = 2 · 3 + γ <::::> γ = -6 :;t:Ο. Τελικά: (ε) y = 2x - 6 . { χ3 - χ ' αν χ :;t: ι Η. Δίνεται η συνάρτηση f με f (χ) = χ - ι όπου α>Ο, παράμετρος.

,J;. ' αν χ = ι Η Να βρεθεί το πεδίο ορισμού της f. ;; Να βρείτε το όριο lim f (χ) .

χ-->1

γ. Να βρεθεί ο αριθμός α, αν γνωρίζουμε ότι η f είναι συνεχής στο χ0 = ι . �·� . Να βρεθεί η γωνία ro που σχηματίζει η εφαπτομένη της Cr στο σημείο της Μ (Ο, f (Ο)) με τον

άξονα χ ' χ . r: . Για α = 4 , να βρεθεί η τιμή f ' (ι) . /\ !Ί ση υ . Η f έχει προφανώς πεδίο ορισμού το A=R

xJ - x x (x2 - l ) x (x - l) ( x + l ) β. x :;t: I => f ( x ) =--= = = (χ2 + χ ) , οπότε: limf ( x ) = lim (x2 + x ) = 2 Χ - 1 Χ - 1 Χ - 1 Χ --> 1 Χ --> 1

: . Η f είναι συνεχής στο χ0 = Ι => lim f ( χ ) = f ( Ι ) => 2 = .Γα => α = 4 Χ --> 1

Λ , x :;t: 1 => f' ( x ) = (x2 + x ) ' = 2x + I . Άρα: εφω = f' ( 0) = 2 · 0 + 1 = 1, οπότε ω = 45° . f ( l + h) - f ( I ) ( I + h)2 + ( I + h ) - 2 h2 + 3h Με h :;t: Ο , δηλαδή Ι + h :;t: Ι έχουμε : λ (h ) = h = h = h = h + 3 ,

ΕΥΚΛΕΙΔΗΣ Β ' 81 τ.l/64

Page 67: Ευκλειδης Β 81

------------- Μαθηματικά για τη Γ Λυκείου -------------οπότε: limλ (h ) = lim (h + 3) = 3 => f ' ( 1 ) = 3 . Τελικά f ' ( x ) = 2x + 1 για κάθε χ ε R , αφού 2 · 1 + 1 = 3 .

h --t O h --t O

9 . Δίνονται οι συναρτήσεις f, g : R � R με f( χ} = ( α2 + β2 ) χ2 - 2αχ , g ( χ) = 4βχ + 5. όπου α,β πα-ράμετροι στο R. υ . Αν ισχύει lim f ( χ} + lim g( χ} = Ο , να δείξετε ότι α = 1 και β = -2 .

x--tl x--tl

f (x) - � μ. Για τις παραπάνω τιμές των α, β να βρεθεί το όριο li"? ( }

5 . X--t- g χ + 13χ - 8 5

γ. Για τις παραπάνω τιμές των α, β να βρεθεί θετικός αριθμός λ ώστε: f'( -% }n2 λ = g' ( 2012) (Ι)

\ υ σ ι 1 υ. Έχω: limf( x) + limg( χ) =0=> α2 + β2 - 2α+ 4β + 5 =0 => ( α- 1)2 + (β + 2)2 = Ο=>α = 1 και β = -2 . Χ---+-1 Χ--71

β. Για α = 1 και β = -2 οι f, g γίνονται: f ( x ) = 5x2 - 2χ , g ( x ) = -8x + 5 . f ( x ) - � 5χ2 - 2χ - � 5χ2 - 2χ - �

Είναι: h ( x) = 5 5 5 = g ( x ) + 1 3x - 8 -8χ + 5 + 1 3χ - 8 5χ - 3 5 (x - �J (x + _!_ J ( 5x - 3) (x + _!_J 5 5 = 5 = χ + _!_ , οπότε : lim h ( x) = lim(x + _!_) = -± 5χ - 3 5χ - 3 5 χ->� χ->� 5 5

5 5

Για α = 1 και β = -2 έχουμε: f' ( x ) = 1 0x - 2, g' ( χ ) = -8 , οπότε: f' (-%) = 1 0(-%) - 2 = -8 και ( 1 ) <=> -8 lη2 λ = -8 <=:> lη2 λ = 1 <=:> 1ηλ = 1 ή ln λ = - 1 <=:> λ = e ή λ = ! . e

i 1 . Ένας αγρότης θέλει να πάρει νερό από μια πηγή που απέχει 640 μέτρα από το κτήμα του. Θα χρησιμοποιήσει γι' αυτό σωλήνες μήκους χ μέτρων ο καθένας.

Το κόστος αγοράς κάθε σωλήνα είναι 2 + 5χ2 € και τα εργατικά (σκάψιμο και σύνδεση σωλήνων) 96

είναι .± € για κάθε σωλήνα. Να βρεθεί το μήκος κάθε σωλήνα για να έχει ο αγρότης το ελάχιστο 3

κόστος κατασκευής . . \ υ :Υ η Έστω ότι ο αγρότης θα χρησιμοποιήσει y σωλήνες. Αφού ο καθένας έχει μήκος χ, θα είναι

640 τ , ( , , ) . θ λ , . 2 5 χ 2 4 1 0 5 χ 2 € Σ , y =- . ο κοστος αγορα και εργατικα για κα ε σω ηνα ει ναι +-+- =-+- . υνεπως το χ 96 3 3 96 ' κ € ( ' ' ) λ ' ' κ ( 1 0 5 χ 2 ) ( 1 0 5 χ 2 ) 640 κοστος σε αγορα και εργατικα για τους y σω ην ες ει ναι, = - +- · y = -+- ·-= 3 96 3 96 χ = 6400 + 3200χ2 = 6400 + 1 00χ = = _!_( 6400 + 1 οοχ) = Κ ( χ ) με χ > ο . 3χ 96χ 3χ 3 3 χ Ε, Κ' ( ) - 1 ( 1 00 6400 J - 1 ( 1 00x2 - 6400 ) , . ι ναι χ - - --- -- οποτε. 3 χ2 3 χ2 ' κ' ( ) ο 1 ( 1 00χ2 - 6400 ) ο 1 00χ2 - 6400 ο 1 00 2 6400 ο 2 64 χ>0 8 χ > <:::> - 2 > <:::> 2 > <:::> χ - > <:::> χ > <=:>χ > ' 3 χ χ

Κ' ( χ ) < Ο <:::> Ο < χ < 8 και Κ(χ) συνεχής στο χ0 = 8 . χ ο 8 Κ' ( χ ) - ο Κ ( χ) γν. φθίνουσα

+ οο +

γν. αύξουσα

Παρατηρούμε ότι το κόστος μειώνεται στο διάστημα (Ο, 8) και αυξάνει στο [ 8, +οο) .

Το κόστος λοιπόν γίνεται ελάχιστο μόνον όταν κάθε σωλήνας έχει μήκος 8 μέτρα. ΕΥΚΛΕΙΔΗΣ Β' 81 τ. l/65

Page 68: Ευκλειδης Β 81

------------- Μαθηματικά για τη Γ ' Λυκείου -------------

Εισαγωγικό σημείωμα για τους γεωμετρικούς τόπους στο μιγαδικό επίπεδο Ως γνωστόν η απόσταση (ΑΒ) δύο σημείων A(z1 ) , B(z2), που είναι εικόνες των συγκεκριμένων μιγαδικών αριθμών Ζ1 = χ 1 + Υ ι i , z2 = χ2 + y 2i , δίνεται από τον τύπο ( ΑΒ) = l zι - z2 1 . Αν M(z) είναι η εικόνα του μεταβλητού μιγαδικού αριθμού z = χ + yi , τότε είvαι προφανές ότι: lz - z1 1 = (ΜΑ) , lz - z2 1 = (ΜΒ) . Έτσι λοιπόν εύκολα αντιλαμβανόμαστε το σχήμα που παριστάνει κάθε μια από τις παρακάτω σχέσεις, όταν z1 ::f:. z2 : 1 . Η εξίσωση lz - z1 1 = lz - z2 1 περιστάνει τη μεσοκάθετο (ε) του ΑΒ. Ενώ η aνίσωση lz - z1 j > l z - z2 1 το ανοιχτό ημιεπίπεδο ((ε), Β). 2. Η εξίσωση lz - z1 1 = ρ , όπου ρ σταθερά με ρ>Ο παριστάνει κύκλο με κέντρο Α και ακτίνα ρ, ενώ η

aνίσωση lz - z1 1 < ρ το εσωτερικό αυτού. 3 . Η εξίσωση lz - z1 1 + lz - z2 1 = c , όπου c σταθερά με c > lz 1 - z2 1 παριστάνει έλλειψη με εστίες τα Α, Β

c και α = - . 2 4. Η εξίσωση l l z - z1 j - j z - z2 1 1 = c , όπου c σταθερά με Ο < c < lz1 - z2 1 παριστάνει υπερβολή με εστίες

c Α, Β και α = - . 2 5 . Τέλος η εξίσωση : lz -z, l ��� ( ! ) περιστάνει παραβολή με εστία Α και διευθετούσα την αz + βz

ευθεία (δ) :αχ+βy+γ =Ο , όπου φυσικά α,β,γ σταθερές στο IR , με lαJ + I� ::f:. O , αφού : ( 1 ) <=> (ΜΑ) = d(Μ, (δ)) . Ασκήσεις επί των Μιγαδικών Αριθμών

Θ , δ , , , I I ι 2z - i εωρουμε τους μιγα ικους z, w τετοιους ωστε z = , w = -. -- . ιz + 2 (α) Να αποδειχθεί ότι lwl = ι .

Μπουρνής Απόστολος

(β) Να αποδειχθεί ότι δεν υπάρχουν μιγαδικοί αριθμοί z, w με εικόνες Μ, Ν, τέτοιες ώστε (ΜΝ)=2

Είναι φανερό ότι z ::f:. -2i , διότι lzl = 1 . Άρα ο w ορίζεται για κάθε z με lzl = 1 . , I 1 2 1 2z - i 1 2 l2z - i l 2 ( 2z - i ) · ( 2� + i ) 4z� + 2iz - 2i� + 1 5 + 2iz - 2i� (α) Εχουμε w = -- = = = = = 1 iz + 2 l iz + 21 2 ( iz + 2) · {-iz + 2) zz + 2iz - 2iz + 4 5 + 2iz - 2iz

β) Τα σημεία Μ, Ν ανήκουν στον κύκλο με κέντρο την αρχή Ο και ακτίνα ρ= 1 . Αν λοιπόν υπάρχουν τέτοια σημεία, τότε αυτά θα είναι aντιδιαμετρικά, δηλαδή συμμετρικά ως προς την αρχή Ο. Οπότε αν z = χ + iy , τότε w = -χ - iy με χ2 + y2 = 1 ( 1 ) και -x - iy = �/x + i�; - i = 2( + (2) -.l ) i � (-x - iy) {(2 - y) + ix ] = 2x + (2y - l ) i � ι χ + yι + 2 2 - y + ιχ

� -χ (2 - y) + xy + i [ -χ2 - y(2 - y)] = 2χ + (2y - l ) i � -2χ + 2xy + i{ -χ2 + y2 - 2y) = 2χ + (2y - l ) i { -2x + 2xy = 2x } { x (y - 2x ) = O } { χ = Ο } , { y = 2x } � -x2 + y2 - 2y = 2y - 1 � -x2 + y2 - 4y + 1 = 0 � y2 - 4y + 1 = 0 (2) η 3χ2 - 8χ + 1 = 0 (3) ΕΥΚΛΕΙΔΗΣ Β ' 81 τ. l/66

Page 69: Ευκλειδης Β 81

-------------Μαθηματικά για τη Γ Λυκείου ------------­

Αν χ = Ο, τότε : ( Ι ) :::::? y = ±1 και (2 ):::::? 2 ± 4 = 0 , άτοπο. Αν y=2χ, τότε ( 1 ) :::::? 5χ2 = 1 :::::? χ = ± � και (3 ):::::? � ± � + 1 = 0 :::::? 5 ±J5 = 0, άτοπο. ν5 5 ν5

J : ι , , , . Αυτό που διαπιστώσαμε είναι ότι η χορδή ΜΝ του μοναδιαίου κύκλου στο παραδειγμά μας αποκλείεται να γίνει διάμετρος. Ένα πιθανό λάθος λοιπόν θα ήταν να γράψει κάποιος: max (MN) = max Jz - w J = 2ρ = 2

(α) Να βρεθούν οι ακέραιοι αριθμοί ν ώστε ί3ν+s = ι (I) (β) Να βρεθούν οι ακέραιοι αριθμοί ν ώστε ί3ν+s = i (II) (γ) Αν z1 , z 2 οι ρίζες της z 2 + z + ι = Ο , τότε να δείξετε ότι z ; = z2 , z ; = z 1 και να βρείτε τις

δυνάμεις z ν , ν Ε Ν* συναρτήσει του z Ε {z 1 , z 2 } .

(α), (β) Είναι γνωστό ότι ί ν = ί υ ' όπου ν = 4 κ + υ με κ Ε z και υ Ε {Ο, 1, 2, 3} . Άρα: Α= ί3ν+S = ί3 ν · ί5 = { ίν γ i4i = { ί υ )3 ί = ί 3υ+ Ι , διότι ί4 = 1 . υ = Ο :::::? Α= ί υ = 1:::::? Α= ί4 = 1 .

υ = 2:::::? Α = i7 = ί4ί3 = ί 3 = -ί . υ = 3 :::::? Α = ί1 0 = ί8ί2 = ί2 = - 1 . Τελικά: (Ι)<=> ν = 4κ+ 1 , και (Π)<=> ν = 4κ, κ ε Ζ,

(γ) Έχουμε: z1 z2 = 1 ( 1 ) και (z 1 - 1 ) { z� + z1 + 1) = Ο=? z; - 1 = Ο :::::? z; = 1 (2), οπότε ( 1 ) 3 2 (2J 2 Ο , 2 Α 3 , ν 3κ ( 3 )κ 1κ 1 ο

, ο :::::? z1 z2 = z1 :::::? z2 = z1 • μοιως z1 = z2 • ν ν = κ, τοτε z = z = z = = = z , αφου z =Ι=

(γιατί;) . Αν ν = 3κ + 1 , τότε zν = z3κ+Ι = z3κz = z . Αν ν = 3κ + 2, τότε zν = z3κ+2 = z3κz2 = z2 = -z - 1 . Τελικά λοιπόν οι δυνάμεις zν είναι το πολύ πρωτοβάθμιες εκφράσεις του z .

(α) Αν Jz l < ι και Jw l < ι , να δείξετε ότι: Jz + wJ < jι + � · wj (β) Α ν Jz - 2 - i J :S; 5 , να δείξετε ότι: 8 :S; Jz - ι4 - 6i J :S; ι8

(α) Για να δείξουμε ότι ισχύει: J z + w J < j 1 + � · w j , αρκεί να δείξουμε μια από τις παρακάτω σχέσεις. Jz + wJ2 < j1 + � · wj2 , ( z + w) · (� + w) < ( 1 + � · w) · ( 1 + z · w ) , z� + zw + w� + ww < 1 + zw +� +� · zw, Jz J 2 + JwJ 2 < 1 + jz2 j · Jw J2 , 0 < 1 + Jz J 2 · Jw J 2 - JzJ 2 - Jw J2 , 0 < (1 - Jz J 2 ) - Jw J 2 ( 1 - Jz ι z } , 0 < (1 - Jz ι z } · ( 1 - Jw J2 ) .

Η τελευταία όμως είναι αληθής, διότι Jz J < 1 και Jw J < 1 (β) Θέτουμε w = z - 2 - ί , τότε η παράσταση Α = z - 14 - 6ί γράφεται

A = (z - 2 - i ) + (- 1 2 - 5i ) = w + w1 , όπου w1 = - 1 2 - 5ί , οπότε JAJ = Jw + wι l · Από την τριγωνική I Jw J - Jw ι J I :S; Jw + wι J :S; Jw J + Jw ι J

ανισότητα και αφού

έχουμε: Jw J :S; 5 και

Jw , J = Jl 22 + 52 = 1 3 .Θα έχουμε I JwJ - Jwι J I � J 1 3 - 5J = 8 και JwJ + Jw, J :S; 5 + 13 = 1 8 . Άρα 8 :S; JAJ :S; 1 8 (Ι)

χ

Υ

Γεωμετρική αντιμετώπιση : Παρατηρούμε ότι τα σημεία M(z) σχ. α ανήκουν στον κυκλικό δίσκο με κέντρο Κ(2, 1 ) και ακτίνα ρ=5 , ' ·

ενώ Jz - 1 4 - 6i J = (ΛΜ) , όπου Λ( 1 4,6), (σχ α) και (ΚΛ) = �( 1 4 - 2) 2 + ( 6 - 1 ) : = 1 3 . Π.λ•: ::-. -

ΕΥΚΛΕΙΔΗΣ Β ' 81 τ. Ι/67

χ

Page 70: Ευκλειδης Β 81

------------ Μαθηματικά για τη Γ Λυκείου ------------(ΛΜ , ) s (ΛΜ ) s (ΛΜ2 ) => (ΚΛ) - ρ s (ΛΜ ) s (ΚΛ) + ρ => 1 3 - .5 s I AI s 1 3 + 5 => 8 s IAI s 1 8 Προσοχή: Έτσι δικαιολογείται ότι max iAI = 1 8 και min iAI = 8 κάτι που δεν μπορούμε να συμπεράνουμε απευθείας από τη σχέση (Ι) στην προηγούμενη απόδειξη .

Λ4 (α) Να βρεθεί ο γ. τόπος (c) των σημείων M(z) ώστε: lz - ι οΙ = 3 lz - 21 (β) Αν z, , z2 μιγαδικοί με εικόνες στην γραμμή (c), να δείξετε ότι ο αριθμός

z1 + z2 - 2 w = είναι πραγματικός. ιO + z1z2 - z, - z2

Λύση (α) Έχουμε: lz - 1 01 = 3 lz - 21 <:::> lz - 1 01 2 = 9 lz - 21 2 <:::> (z - 1 0) · (� - 1 0) = 9 (z - 2) · (� - 2) <:::> - - - - - - - - - -<:::> zz - 1 Oz - 1 Oz + 100 = 9zz - I 8z - I 8z + 36 <:::> 8zz - 8z - 8z = 64 <:::> zz - z - z = 8 <:::> zz - z - z + Ι = 9 <:::>

<:::> z{� - Ι) - {� - Ι) = 9 <:::> ( z - 1) · {� - Ι) = 9 <:::> l z - I I 2 = 9 <:::> lz - II = 3 <:::> Μ Ε ( c) ,

όπου ( c ) ο κύκλος με κέντρο το K( l ,0) και ακτίνα ρ=3 (β) Θέτουμε w, = z, - I και w2 = z2 - I , οπότε lw , l = lw2 1 = 3 , δηλαδή wι =.-2_ , W2 =� και ο w w1 w2 εκφράζεται πλέον εύκολα συναρτήσει των w 1 , w2 • Πράγματι: w 1 + w 2 = z1 + z2 - 2 και

w, ·W2 =(Ζι -Ι) · (� -Ι) =Ζι� -Ζι -� +1 . Άρα ΙΟ+Ζι� -Ζι -� =9+w1 ·w2 . Οπότε θα έχουμε: w = w, + w2 9 + w1w2

και w 9+Wι · W2

_2_+_2_ { w1 +w2 ) w1 +w2 w,9 w29

w, · w2 - w, ·w2 w, +w2 w. Αφού w =w, θα έχουμε: w E IR . 9+- ·- 9(Ι +-9-) - W1 ·W2 +9 9+w1 ·w2 w, w2 w,w2 w, ·w2

- Ν δ θ , , ' λε 'ζ ξ' ι 2 ιο ο , , Λ ::ι α απο ειχ ει οτι ο ς οι ρ ι ες της ε ισωσης -- + --+ ... + -- = ει ναι πραγματικες.

z + 2 z + 2 z + ιΟ Λί1ση 'Ε ίζ ξ' τ , 1 2 ιο ο ( 1) , 1 2 10 ο (2) στω z0 ρ α της ε ισωσης. οτε --+--+ .. . +--= , οποτε ---+---+ .. . +---- = . Ζσ +Ι Ζσ +2 Ζσ +ΙΟ z.o +I z.o +2 z.o +lO Αφαιρούμε τις σχέσεις ( 1 ) και (2), οπότε: (-I---1-)+(-2---2-)+ . . . +(__!Q_ _ __!Q_)=O::::> Ζσ + Ι z0 + I z0 +2 z0 +2 z0 + 10 z0 + I0 ; -z 2(; -z) 10(; -z) (- ) [ Ι 2 10 ] => ( ) + ( ) ( ) + . . . + ( ) ( ) =0=> Ζο -Ζσ . --2 +--2 + . . . + 2 =0 (3) (Ζσ + Ι) · z.o + I z.o +2 · z.o +2 z.o +lO · z.o + IO lz0 + 11 IZo +� IZo + Iq

Κάθε προσθετέος της αγκύλης είναι θετικός, οπότε: ( 3) => �ο - z0 = Ο => �ο = z0 => z0 Ε IR . Λ.6 (α) Να αποδειχθεί ότι l z , + z2 1

2 + l z , - z2 1

2 = 2{ 1 z , 1

2 + l z2 1

2 ) (Νόμος του παραλληλογράμμου). (β) Αν z, + z2 + z3 = Ο και l z , l = lz 2 1 = lz3 1 = ι , να αποδειχθούν τα παρακάτω: ί. l z , - z2 1 = l z2 - z3 1 = lz3 - z, l = .J3 ίί. z; = z2 • Ζμ z; = z, · z3 και z; = z, · z2 ίίί. Οι μιγαδικοί w 1 = z: · z� , w 2 = z� · z� και w 3 = z� · z: , έχουν εικόνες που είναι κορυφές

ισόπλευρου τριγώνου •

. \()ση (α) Έχουμε lz , + z2 1 2 + lz, - z2 1 2 = (z1 + z2 ) · (�ι + �2 ) + (z1 - z2 ) · (�ι - �2 ) =

= z1 �ι + z 1 �2 + z2 �ι + z2 �2 + Ζ1 �ι - Ζ1 �2 - z2 �ι + z2 �2 = 2z1 �ι + 2z2 �2 = 2 ( l z 1 1 2 + lz2 1 2 ) (β) ί. Εφαρμόζουμε το (α) και θέτουμε z1 + z2 = -z3 , οπότε:

ΕΥΚΛΕΙΔΗΣ Β' 81 τ.l/68

Page 71: Ευκλειδης Β 81

------------ Μαθηματικά για τη Γ Λυκείου -----------

l-z3 1 2 + lz , - z2 1 2 = 2 lz , l 2 + 2 lz2 1 2 => lz, - z2 1 2 = 3 => lz, - z2 Ι = J3 με όμοιο τρόπο βρίσκουμε lz2 - z3 1 = J3 και lz , - z3 1 = J3

2 - Ι - Ι - Ι - - -ίί. l z , l = Ι => zι =- . Όμοια Ζ2 =- και ZJ =- . Εξάλλου : z1 + z2 + z3 = Ο => zι + Ζ2 + ZJ = Ο => z1 z2 z3

Ι Ι Ι ( ) 2 2 =>-+-+- = Ο => z1z2 + z1z3 + z2z3 = Ο => z1 z2 + z3 + z2z3 = Ο => -z1 + z2z3 = Ο => z1 = z2z3 . z1 z2 z3 ο ' 2 2 μοιως: z2 = z1z3 , z3 = z1 z2

ίίί. Αρκεί να δείξουμε ότι lw , - w2 1 = lw , - w3 1 = lw2 - w3 1 * Ο . 'Εχουμε : w, -w2 = z�z; - z�z; = z; ( z� - z2z; ) = z; [(z� )2 - z2z; J = z; [ z;z; - z2z; J = z� · z� (z2 - z3 ) . Άρα lw , - w2 1 = lz� l · l z� l · l z2 - z3 1 = I · I · J3 = J3 . Όμοια lw , - w3 1 = J3 = lw2 - w3 1

ΛΊ Έστω z1 * z2 και z1 ,z2 ε C' ρίζες της εξίσωσης lzl 2 · w+ l� i · z2 =0 (1), όπου w * Ο σταθερά. (α) Να αποδειχθεί ότι ο αριθμός α = z 1 + z2 είναι ρίζα της (1) (β) Η εξίσωση (1) έχει άπειρες ρίζες.

(α) Αφού z, ρίζα της (1) θα έχουμε: lz, l 2 w + lw l i · z� = Ο => z, · �� · w + lw l i · z� = Ο => => z, (�ι · W + lw l i · z , ) = 0 ::::> �ι · w + lw l i · z1 = 0 (2) (z, :;e O) Όμοια έχουμε �2 · w + lw l i · Ζ2 = 0 (3) με z, , z2 * Ο . Προσθέτουμε τις (2) και (3), οπότε (�ι + �2 ) · w + lw l i ( z, + z2 ) = Ο ( 4). Πολλαπλασιάζουμε με z1 + z2 οπότε: (4) => (z, + z2 ) · (z1 + z2 )w + lwl i ( z, + z2 )2 = O=> Iz, + z2 1 2 w + lwl i ( z, + z2 )2 = 0=> iαl 2 w + lwl i · α2 = Ο => Ο αριθμός α είναι ρίζα της ( 1 )

(β) Είναι z, * z2 και α * z1 , α * z2 , αφού z1 , z2 * Ο . Από το (α) ερώτημα ο ι αριθμοί β 1=α+ z1 =2 z 1 + z2 και γι = α + z2 = z, + 2z2 είναι ρίζες της (1) . Επαγωγικά αποδεικνύεται ότι οι αριθμοί βν = νz1 + Zz , γ ν = z1 + νz2 , είναι ρίζες της ( Ι ) για κάθε ν ε Ν . Η ( Ι ) λοιπόν έχει άπειρες ρίζες. \ :'. Αν ΑΒΓ Δ τετράπλευρο, να αποδειχθεί ότι ( ΑΔ) · (ΒΓ) + ( ΑΒ) · (Γ Δ) � ( ΑΓ) · (ΒΔ) .

(Γενικευμένο θεώρημα του Πτολεμαίου) Στις κορυφές Α, Β, Γ, Δ aντιστοιχούμε τους μιγαδικούς z1 , z2 , z3 , z4 . Τότε (ΑΔ) = lz4 - z, l , (ΒΓ) = lz3 -z2 1 , (ΑΒ) = lz2 - z, l , (ΓΔ) = lz4 -z3 1 , ( ΑΓ) = lz3 - z, l και (ΒΔ) = lz4 - z2 1

Παρατηρούμε ότι: ( z4 - z1 ) • ( z3 - z2 ) + ( z2 - z1 ) • ( z4 - z3 ) = z4z3 - z4z2 - z1 z3 + z1z2 + z2z4 - z2z3 --z 1z4 + z1z3 = z4z3 + z1 z2 - z2z3 - z1z4 = z4 ( z3 - z1 ) - z2 ( z3 - z1 ) = ( z3 - z1 ) • ( z4 - z2 )

Οπότε i( z4 - z, ) · ( Ζ3 - Ζ2 ) + ( z2 - z, ) · ( Ζ4 - Ζ3 )I = I ( z3 - z, ) · ( Ζ4 - Ζ2 )I · Όμως I( Ζ4 - z, ) · ( z3 - z2 ) + ( z2 - z, ) · ( Ζ4 - Ζ3 ) I � l l z4 - z, l · l z3 - z2 1 + lz2 - z, l · l z4 - z3 1 1 Άρα lz3 - z, l · l z4 - zz i � Ιz4 - z, l · l z3 - z2 1 + 1 zz - z, l · l z4 - z3 1 ' Συνεπώς (ΑΓ) · (ΒΔ) � (ΑΔ) · (ΒΓ) + (ΑΒ) · (ΓΔ)

. \ 9 Αν z μιγαδικός, να βρεθεί η ελάχιστη τιμή της παράστασης: S = lzl + lz - 4l + lz - 3il + lz - 4-3il

Έστω M(x,y) η εικόνα του μιγαδικού z και Α, Β, Γ οι εικόνες των μιγαδικών z , = 4 . Ζ: = 3 ί . z3 = 4 + 3i αντίστοιχα. Τότε Α(4,0), Β(Ο,3) και Γ(4,3) . Παρατηρούμε ότι (ΜΟ) = l z l . (\l-\ 1 = z - .:. .

(ΜΒ) = lz - 3i l και (ΜΓ) = lz - 4 - 3i l Ακόμη έχουμε (ΜΟ) + (ΜΓ) � ( ΟΓ) και (ΜΒ) + (ΜΑ) � ( ΑΒ) .

ΕΥΚΛΕΙΔΗΣ Β ' 81 τ.l/69

Page 72: Ευκλειδης Β 81

------------ Μαθηματικά για τη Γ Λυκείου ------------

Άρα (ΜΟ) + (ΜΓ) + (ΜΒ) + (ΜΑ) � 2 (0Γ ) , διότι (ΑΒ)=(ΟΓ)=�32 + 42 = 5 . Άρα: S = lzl + lz - 41 + lz - 3i l + lz - 4 - 3i l � Ι Ο ( Ι ) . Παρατηρούμε ότι η ισότητα στην (1 ), ισχύει μόνον

όταν το Μ συμπέσει με το σημείο τομής Κ των διαγώνιων του ορθογωνίου ΟΑΓΒ. Μ • I Β(Ο,3) ·.

I ···. Γ(4,3) Άρα το ελάχιστο της παράσταση S είναι Ι Ο και θα

β ' ' ' 2 3 . συμ αινει μονον οταν z = + - ι . 2

ο I Α(4,0) -Να βρεθεί ο γεωμετρικός τόπος των σημείων M(z), ώστε: lαz + βz l = γ (Ι) , όπου α, β, γ e JR .

Έστω z = χ0 + y 0i Ι ) Αν γ<Ο, τότε η (Ι) είναι ψευδής, δηλαδή ο γ . τόπος είναι το κενό σύνολο. 2) Αν γ=Ο, τότε (Ι ) <=> αz + β� = Ο <=> α( χ0 + y0i ) + β ( χ0 - y0i ) = 0 <=> (α + β) χ0 + (α - β) y0 = Ο 2α) Αν Ια + βl + Ι α - βl = Ο, δηλαδή α=β=Ο, τότε η (Ι) ισχύει για κάθε χ0 , y0 ε JR , δηλαδή ο γ. τόπος είναι ολόκληρο το επίπεδο. 2β) Αν Ια + βi + Ια - βl :;t: Ο, δηλαδή α :;t: Ο ή β :;t: Ο, τότε ο γ. τόπος είναι ευθεία της μορφής (α + β) χ + (α - β) y = Ο , που προφανώς διέρχεται από την αρχή των αξόνων. 3) Αν γ>Ο, τότε: ( Ι ) <::> (α + β )2 χ2 0 + (α - β)2 y2 0 = γ2 (11) . 3α) Αν Ια+� + Ια-� =0, δηλαδή α=β=Ο, τότε η (11) είναι ψευδής, δηλαδή ο γ. τ. είναι το κενό σύνολο.

α + β :;t: Ο} γz 3β) Αν , δηλαδή α = β :;t: Ο, τότε: ( Ι ) <=> 4α2χ 2 0 = γ2 <=> χ� = -2 , οπότε ο γ. τόπος είναι α - β = Ο 4α οι κατακόρυφες ευθείες: χ = ± l. 2α 3γ) Αν α + β = Ο} , δηλαδή α = -β :;t: Ο, α - β :;t: Ο

2 τότε: ( 11 ) <=> 4α2y20 = γ2 <::> y� =� , οπότε ο γ. τόπος 4α είναι οι οριζόντιες ευθείες: y = ± l . 2α

α + β :;t: Ο} 3δ) Αν (ΠΙ) , δηλαδή α :;t: β και α :;t: -β α - β :;t: Ο τότε: χ2 y2 ( 11 ) <=> ο 2 + ο 2 = 1 . Στην (α: β ) ( α� β ) περίπτωση (α + β γ = (α - β )2 , δηλαδή αβ=Ο που λόγω των σχέσεων (III) γίνεται (α = 0) ή (α :;t: 0) 0 γ. τόπος είναι ο κύκλος με κέντρο το 0(0,0) και ακτίνα ρ = -1 _

γ_Ι . β :;t: Ο β = Ο α + β Στην περίπτωση όμως (α + β )2 :;t: (α - β γ , δηλαδή αβ :;t: Ο , ο γ. τόπος θα είναι έλλειψη με τις εστίες στον άξονα χ ' χ, όταν (α + β )2 < (α - β )2 , δηλαδή αβ<Ο και τις εστίες στον άξονα y ' y, όταν αβ>Ο π.χ. Η εξίσωση 19z - � 1 = 4 , όπου α=9. β=- Ι , δηλαδή (α + β)2 < (α - β)2 παριστάνει την έλλειψη χ2 y2 χ2 y2 --2 +--2 = Ι , δηλαδή την έλλειψη -2 +-2 = Ι που έχει προφανώς εστίες τα σημεία Ε(-3 ,0) (�ο) (:�) 5 4

και Ε(3 ,0) του άξονα χ 'χ, της οποίας η εξίσωση μπορεί να δοθεί και με μια από τις παρακάτω μορφές: Ι z + 3l + Ιz - 31 = 1 0, Ιz + 3l + I� - �1 = 1 0, ι� + 3 1 + Ιz - 31 = 1 0, ι� + 3 1 + 1� - 3 1 = 10, (γιατί;)

ΕΥΚΛΕΙΔΗΣ Β' 81 τ.l/70

Page 73: Ευκλειδης Β 81

------------ Μαθηματικά για τη Γ Λυκείου

των Γιώργου Γκαρή, Βασίλη Καρκάνη

Αν ο μιγαδικός αριθμός z ικανοποιεί τις συνθήκες lz - i l Sl και lz - 2i l =1 δείξτε ότι l:S iz l sJ3 .

Α ν z = χ + yι, τοτε εχουμε: � . , , �z-η �1 1χ+)i-η �1 �x+(y-1) il �11)i +(y-1)2 �1=1i+(y-1)2 �1 z-2ij =1 x+)i-2ij =1 lx+(y-2) � =1 )i +(y-2)2 =1 i +(y-2)2 =1

� {χ2 + y2 - 2y + 1 � 1 � {χ2 + y2 � 2y � {2χ2 + 2y2 � 4y ( 1 ) χ2 + y2 - 4y + 4 = 1 χ2 + y2 + 3 = 4y χ2 + y2 + 3 = 4y (2) (2) Αλλά: ( 1 ) �2χ2 + 2y2 � χ2 + y2 + 3 � χ2 + y2 � 3 � lzl2 � 3 � lzΙ � J3 (3)

Εξάλλου: (2)� y2 - 4y + 3 = -χ2 � y2 - 4y + 3 � o� 1 � Υ � 3 � y2 � 1 � χ2 + y2 � 1 � Ιzl 2 � 1 � Ιzl � 1 (4) Άρα: (3), (4)� 1 � lzl � J3

Οι εικόνες των μιγαδικών z που ικανοποιούν τη σχέση lz - i l S ( 1 ) είναι σημεία του κυκλικού δίσκου C1 με κέντρο Κ1 (0, 1 ) και ακτίνα ρ 1 = 1 . Οι εικόνες των

Υ

χ' ο χ

μιγαδικών z που ικανοποιούν τη σχέση lz - 2i l = 1 είναι σημεία του κύκλου Cz με κέντρο Kz(O, 2) και ακτίνα ρ2 = 1 . Εύκολα βρίσκουμε ' 'κλ C C ' ' A(J) 3 ) οτι οι κυ οι 1 , z τεμνονται στα σημεια 2, 2 και

Β ( J3 3 ) λ ' ' ξ ' ' Ε ' -2'2 υνοντας το συστημα των ε ισωσεων τους. ιναι προφανές ακόμα ότι οι εικόνες των μιγαδικών z που ικανοποιούν ταυτόχρονα τις σχέσεις ( 1 ) και (2) είναι τα σημεία του κλειστού τόξου ΑΚ,Β και άρα από τα σημεία αυτά, την ελάχιστη απόσταση από το 0(0, Ο) έχει το Κ1 (0, 1 ) . Για την μέγιστη απόσταση παρατηρούμε ότι για κάθε σημείο Μ

εσωτερικό του τόξου ΑΚι τα τρίγωνα Κ2ΜΟ, Κ2ΑΟ έχουν Κ2Α = Κ2Μ , Κ20 = Κ20 και Μ20 > ΜΚ20 , οπότε: ΟΑ>ΟΜ. Άρα τη μέγιστη απόσταση από το Ο έχουν (λfryω συμμετρίας ως προς τον y'y) τα muJda '�·�) = { � .%) Άιχ> ι� .. = lz.. I = <OΚ, ) = l ,

Να βρεθεί ο γεωμετρικός τόπος των εικόνων των μιγαδικών z που ικανοποιούν τη σχέση 1 + 1 = 2012 , z Φ ± lOOOi (1) lz + lOOOi l lz - lOOOi l lz2 + 106 1

Να βρεθεί η μέγιστη και η ελάχιστη τιμή του lz l , όπου z ένας μιγαδικός που ικανοποιεί την (1) .

Είναι z2 + 1 06 = z2 - ( 1 000i)2 = (z + 1 000i)(z - 1 000i) . Άρα lz2 + 1 06 1 = lz + 1 000i l l z - 1 000i l , οπότε: ( 1 ) � l z + 1 000i l + lz - 1 000i l = 20 1 2 (2) Η (2) παριστάνει έλλειψη C με εστίες τα σημεία Ε(Ο, 1 000) και Ε '(Ο, -1 000), δηλαδή : γ= Ι ΟΟΟ και : ,::. =

ΕΥΚΛΕΙΔΗΣ Β ' 81 τ.l/71

Page 74: Ευκλειδης Β 81

------------ Μαθηματικά για τη Γ Λυκείου -----------20 1 2, δηλαδή : α = 1 006, οπότε : β2=α2-γ2= 1 0062-1 0002=( 1 006-1 000)( 1 006+ 1 000) = 6·2006 = 22 · 3009, 2 2 2 2 δηλαδή : β = 2-../3009 < α . Άρα: C : ; + � = 1, δηλαδή C: 2

χ + � = 1 (Προφανώς τα σημεία Ε, β α 2 · 3009 1006 Ε ' που αντιστοιχούν στους αριθμούς z=± 1 000i δεν ανήκουν στην C) . 2 2 β) Αν Μ(χ, y) σημείο της έλλειψης .;. +� = 1 με α>β>Ο, δηλαδή με τις εστίες στον y 'y, τότε: β α

χ2 y2 x2 + y2 (ΟΜ)2 , χ2 y2 x2 + y2 (ΟΜ)2 1 = -2 + -2 :s; 2 = 2 � β :s; ( ΟΜ) εξαλλου 1 = -2 + -2 � 2 = 2 � α � ( ΟΜ) . β α β β β α α α Τελικά β :s; ( ΟΜ) :s; α � β :s; J z j :s; α και στην περίπτωσή μας 2-../3009 ::; J z j ::; 1 006, δηλαδή η μέγιστη τιμή του Jzj είναι 1 006 μόνον όταν και η ελάχιστη 2-../3009 μόνον όταν � .

(χ = Ο ) (y = O ) y = ± 1 006 χ = ±2ν3009

λ4. Να βρεθεί μιγαδικός αριθμός z ώστε: jz - 3 - i J = jz + 2 + 4i J = Jz + 3 - 3ij (1).

{ J z - 3 - i J = Jz + 2 + 4i J ( 2) Έστω z = χ + yi . Προφανώς: ( Ι ) <::::::> I · ι I · ι

και z - 3 - ι = z + 3 - 3ι ( 3) (2 ) <::::::> l x + yi - 3 - i j= J x + yi + 2 + 4i J <::::::> j (x - 3) + (y - l )i j = j (x + 2) + (y + 4)i j <::::::> �(χ - 3/ + (y - ιγ = �(χ+2)2 + (y+4)2 <::::::> (χ - 3)2 + (y - 1 )2 = (χ+2)2 + ( y+4)2 <::::::> χ2 - 6χ + 9 + y2 - 2y + 1 = χ2 + 4χ + 4 + y2 + 8y + 1 6 <::::::> - Ι Οχ - lOy - 10 = Ο <::::::> y = -χ - 1 (3) <::::::> J x + yi - 3 - i j = J x + yi + 3 - 3i j <::::::> j (x - 3) + (y - l)i j = j(x + 3) + (y - 3)i j <::::::> �(χ - 3)2 + (y - 1 )2 = �(χ+3)2 + (y - 3)2 <::::::> ( χ - 3)2 + (y - 1)2 = (χ + 3)2 + (y - 3)2 <::::::> χ 2 - 6χ + 9 + y2 - 2y + 1 = χ 2 + 6χ + 9 + y2 - 6y + 9 - 1 2χ + 48 = Ο <::::::> y = 3 χ + 2 Το σύστημα των (2),(3) δίνει χ = -� , y = _

_!_ , δηλαδή η ( 1 ) ισχύει μόνο για z = -� - _!_i . 4 4 4 4 2 "c Τ ιι6πος ( Γ: :ωμεηΗκι] προσέγγ ιση ) Έστω Α(3 , 1 ), Β(-2,-4), Γ(-3 ,3) και Μ οι εικόνες των μιγαδικών zA=3-i, z8=-2-4i, zr=-3+3i και z

αντιστοίχως. Τότε (l) <::::> j z - zA J = Jz - z6 J = Jz - zr J <::::> MA = MB = MΓ <::::> To Μ ανήκει στις μεσοκαθέτους ε 1 , ε2 των ΑΒ, ΑΓ Εύκολα βρίσκουμε ε1 : y = -χ - 1 και ε2 : y = 3χ + 2 των οποίων σημείο τομής είναι το Μ (-% , -�)

3": Τρ{ιπος Το Μ είναι κέντρο του περιγεγραμμένου κύκλου : χ 2 + y2 + αχ + βy + γ = Ο ( 1) του τριγώνου ΑΒΓ.

9 + 1 + 3α + β + γ = Ο } Η ( 1 ) επαληθεύεται από τις συντεταγμένες των Α, Β, Γ, δηλαδή : 4 + 1 6 - 2α - 4β + γ = Ο και τελικά

9 + 9 - 3α + 3β + γ = Ο 3 1 , α 3 β 1 α = 2 , β = z , γ = -1 5 . Άρα χΜ = -2 = -4 , ΥΜ = -z = -4.

λ 5 Βρείτε την ελάχιστη τιμή του A= jz + 2iJ , όταν ο μιγαδικός z μεταβάλλεται έτσι ώστε jz+3+iJ = Jz-3-7iJ (1)

ΕΥΚΛΕΙΔΗΣ Β ' 81 τ.l/72

Page 75: Ευκλειδης Β 81

------------ Μαθηματικά για τη Γ Λυκείου ------------Λύση Αν z = χ0 + y0i, και Μ η εικόνα του, τότε: ( 1 ) <:::::> l xo + y0i + 3 + i l = l xo + y0i - 3 - 7i l <:::::> l (x0 + 3) + (y0 + 1)i l = l (x0 - 3) + (y0 - 7)i l <:::::>

<:::::> ( χ0 + 3)2 + (Υ ο + 1 γ = ( χ0 - 3 )2 + ( Υ ο - 7γ <:::::> 3χ0 + 4y0 - 1 2 = Ο <:::::> Μ Ε (ε ) , Όπου (ε ) : 3χ + 4y - 1 2 = Ο . Το σημείο Γ(Ο,-2) είναι η εικόνα του zr=-2i. οπό:ε :

= - - > - 13 · 0 + 4(-2) - 1 21 _ ' . - . Α lz zr i - (ΓM) _ d (Γ , ( ε )) --)

- 4 . Άρα. Amin - 4 . 32 + 42 Α6 Έστω C ο γεωμετρικός τόπος των εικόνων των μιγαδικών αριθμών z για τους οποίους

ισχύει lz - 31 = 4 και zι ,z2 δύο μιγαδικοί που οι εικόνες τους ανήκουν στη γραμμή C ώστε lz , - z 2 1 = 8 . Να βρεθεί η τιμή της παράστασης Α= lz , + z 2 1

Λύση Λ · τρ6πος Η εξίσωση lz - 3 1 = 4 παριστάνει τον κύκλο C με κέντρο Κ(3 ,0) και ακτίνα ρ=4. Αν Α,Β οι εικόνες

των zι ,Zz αντίστοιχα, τότε lz1 - z2 1 = 8 � (ΑΒ)=8=2ρ � η ΑΒ είναι διάμετρος του κύκλου C � το Κ είναι μέσο του ΑΒ. Αν Μ η εικόνα του μιγαδικού αριθμού z1+z2 τότε ΟΜ =0Α+0Β = 20Κ = 2 · (3, 0) = (6, 0) . Άρα Μ(6,0), δηλαδή zι+z2=6+0i=6, οπότε: I z1 + z2 1 = 6 .

Β ' τρ6πος Έστω z1=κ+λi ,z2=μ+νi, και Α(κ,λ), Β(μ,ν) οι εικόνες των z1 ,z2 αντιστοίχως τότε z1+z2=(κ+μ)+(λ+ν)i. {χ - κ + μ {3 - κ + μ κ --2- --2- κ + μ = 6 Το Κ(3 ,0) μέσο του ΑΒ� � � { � z1 + z2 = 6 � Ιzι + z2 1 = 6 _ λ + ν 0 _ λ + ν λ + ν = Ο

Υκ --2- --2-Γ' τρ6πος Από το νόμο του παραλληλογράμμου για τους w 1 = z1 - 3, w 2 = z2 - 3 έχουμε:

lw ι + w2 1 2 + lw ι - w2 1 2 = 2 lw ι l 2 + 2 lw2 1 2 � Ιzι + z2 - 61 2 + lz ι - z2 1 2 = 2 · 42 + 2 · 42 � � lz1 + z2 - 612 + 82 = 64 � lz1 + z2 - 61 = Ο � z1 + z2 = 6 � lz1 + z2 1 = 6

Λ 7 Ν α βρεθούν οι μιγαδικοί z, w ώστε: lz - 2J31 + lz + 2J31 = 8 (1) και l lw --121 - lw + -1211 = 2 (2). Λύση Η ( 1 ) παριστάνει έλλειψη C1 με εστίες E1 (2.J3, 0) και E ; (-2.J3 , 0) , δηλαδή γ = 2.J3 και α = 4 ,

2 2 οπότε: β = �α2 -γ2 =vf1 6- 12 = 2 , αφού : ( 1) <=> (ΜΕ, ) + (ΜΕΊ ) = 8 > 4.J3 = (Ε1Ε1 ') . Άρα: C1 :� +L = 1 1 6 4 Η (2) παριστάνει υπερβολή C2 με εστίες Ε2 ( J2, Ο) και Ε� ( -J2, Ο) , δηλαδή γ = J2 και α = 1 , οπότε

β = �γ2 - α2 =� = 1 , αφού : (2) <=> 1(ME2 ) - (ME2') 1 = 2 < 2J2 = (E2E 'z ) Άρα: C2 : χ2 - y2 = 1 (Ισοσκελής υπερβολή) . Λύνουμε το σύστημα των εξισώσεων των C 1 , C2 και

, {�+L = 1 {χ2 + 4y2 = 1 6 {5y2 = 1 5 {y2 = 3 y ε {.J3,-.J3} εχουμε : 1 6 4 <=> <=> <=> <=> 2 2 1 x2 - y2 = 1 x2 - y2 = 1 χ2 = 4 χ ε {2 -2} χ - y = '

Άρα τα κοινά σημεία των C1 ,C2 είναι τα Κ(2, .J3 ), Λ(2,- .J3 ), Μ(-2, .J3 ) και Ν(-2, -.J3 ) και οι μιγαδικοί z,w για τους οποίους ισχύει z=w είναι οι z1=w1=2+ .J3 ί, z2=w2=2-2 .J3 ί, z3=w3=2+.J3 i, 24=w4=-2-.J3 i

ΕΥΚΛΕΙΔΗΣ Β ' 81 τ. l/73

Page 76: Ευκλειδης Β 81

------------- Μαθηματικά για τη Γ Λυκείου

α Η μc� Υ I (� την r 'λυκε ίου Δημήτρης Δρεκόλιας

Μαθητής της Γ Τάξης του Πειραματικού Λυκείου Αναβρύτων

λ 1J σ Ι1 Έστω η συνάρτηση f ( x ) = 2ex - x2 + 1 με πεδίο la) Έχουμε: f ( x ) = 2ex - 2x = 2 (e' - x ) > O , αφού ορισμού το σύνολο Dr = JR. ο

la) Να αποδειχθεί ότι η f αντιστρέφεται και να βρεθεί το σύνολο τιμών της ο

(\ · η : Θεωρείται γνωστή η ανισότητα e' :::: χ + Ι, για κάθε χ Ε JR. από την οποία προκύπτει ως γνωστόν ένας τρόπος εύρεσης της παραγώγου (e' ) ' = e' ο ] l μ> Να βρεθεί πραγματικός αριθμός α, τέτοιος ώ­στε να ισχύει Γ1 (3) = α 2a) Να λυθεί η εξίσωση 2( e"' -e' ) =χ4 -χ2 στο IR ο

2μ> Αν λ, μ Ε JR. και ισχύει: 2eλ-μ +2)ψ>λ7 +μ2 +2 , τότε να αποδειχθεί ότι: λ > μ 3a> Να βρεθεί το εμβαδόν Ε1 του χωρίου που πε­ρικλείεται μεταξύ των ευθειών χ = Ρ ι , χ = ρ2 , του άξονα χ'χ και της γραφικής παράστασης Cr της f, όπου ρ1 , ρ2 οι ρίζες της εξίσωσης του 2α ο

3μ) Να βρεθεί το εμβαδόν Ε2 του χωρίου που πε­ρικλείεται μεταξύ των ευθειών y = 3, x = O, x = l και της Cr της fo 4. Θεωρούμε το μιγαδικό αριθμό z = Ε 1 + κ + ( Ε2 - 2κ2 ) i όπου Ε1 , Ε2 τα εμβαδά του ερωτήματος 4 και κ Ε JR. ο

4a) Να βρεθεί ο γεωμετρικός τόπος των σημείων M(z)o 4μ)Να εξεταστεί αν υπάρχει μιγαδικός αριθμός z τέτοιος ώστε : Re (z) = In (z)

4y)Να βρεθεί μιγαδικός αριθμός z τέτοιος ώστε το άθροισμα του πραγματικού και του φανταστικού του μέρους να γίνεται μέγιστο ο

4δ)ί) Να αποδεδειχθεί ότι υπάρχουν ακριβώς δυο μιγαδικοί αριθμοί z1 , z2 , τέτοιοι ώστε να ισχύ-ει: Im (z1 ) = 1m(z2 ) = 0 ii) Έστω Μ1 ( χ1 , 0 ) και Μ2 (χ2 , 0) οι εικόνες των z1 , z2 και τρία σημεία Α,Β,Γ της παραβολής με τε­τμημένες χ" < χ Ρ < χ Υ μεταξύ των χ 1 , χ2 ο Να δείξε­τε ότι τα Α,Β,Γ είναι κορυφές τριγώνου και ότι υπάρχουν τρία σημεία Μ3,Μ4,Μ5 της παραβολής στα οποία οι εφαπτόμενες σχηματίζουν τρίγωνο όμοιο με το ΑΒΓΟ

e' :::: χ + Ι > χ ο Άρα f tJR. , οπότε η f είναι " 1-1 " Λ δηλαδή αντιστρέφεται ο

Η f είναι συνεχής στο Dr = JR. , αφού είναι παρα­γωγίσιμη ο Άρα f (JR.) = (Α , Β) όπου προφανώς: Α = Δ�./ ( χ ) = Δ�., ( 2e' - χ 2 + Ι ) = - οο και Β = lim f (x ) = lim (2e' - x2 + I } = +oo

Χ --+ +ο:.: Χ --++«ι

διότι : lim ( 2ex - χ2 ) = lim [χ2 (2 e: JJ X --+ +::JC Χ --++ω Χ

Καθόσον : lim(e:J(-100)= lim(ex J(-+ω)= lim(ex J=-+ωo χ__, χ -100 ,__, 2χ -100 χ__, 2 Οπότε:

χ�[ χ2 ( 2:: -IJ ] =-ιω ο Τελικά f (JR.) = IR

l μ) Έχουμε : Γ1 (3) = α <:::::> f ( α) = 3 <:::::> α = Ο, αφού f (Ο) = 3 και f! IR ο 2α) Επίσης έχουμε : 2 (e•' - ex ) = x4 - x2 <:::::> 2ex' - x4 + 1 = 2ex - x2 + I <=> f {x2 } = f (x ) <=> χ 2 = χ <:::::> χ = Ο ή χ = Ι αφού η f είναι " 1- 1" ο

2μ)Έχουμε: 2eλ-μ + 2λμ > λ2 + μ 2 + 2 ::::> 2eλ-μ > λ2 + μ2 + 2 - 2λμ ::::> 2eλ-μ > (λ-μ)2 +2 ::::> 2eλ-μ - (λ - μ )2 + ι > 3 => f (λ - μ) > f (O) ::::> λ - μ > Ο ::::> λ > μ , αφού f t!R

Λ

3a> Αφού f t!R και f (O) = 3 > Ο θα είναι f ( x) > Ο Λ • για κάθε χ Ε [ο, ι ] ,

I I Άρα: Ε1 = Jf ( x ) dx = f[2e' - x2 + l] dx = ο ο

= J2e'dx- Jx2dx+ Jldx = 2[ex ]� -[χ3 ]1 + 1 = 2e-� ο ο ο 3 0 3

3μ> Αφού f t!R θα έχουμε: Λ

f ( χ ) - 3 = f (χ ) - f (Ο) :::: Ο, για κάθε χ Ε [Ο, Ι ] Άρα:

I I I

Ε1 = Kf(x) -3]dx = Jσ(x) dx = K2ex -x2 -2]dx = ο ο ο

= 2 [ ex 1 - [ � Ι - 2 = 6e; Ι 3

ΕΥΚΛΕΙΔΗΣ Β' 81 τ.l/74

Page 77: Ευκλειδης Β 81

-------------- Μαθηματικά για τη Γ Λυκείου

4a) Έχουμε z = Ε1 + κ + {Ε2 - 2κ2 ) i = = --+ + --- κ ι κ ε � 6e - 4 k ( 6e - 1 3 2 2 ) • Ί!])

3 3 ' Αν z = χ0 + y0i τότε για να βρούμε γ.τ. των M(z) πρέπει και αρκεί να βρούμε την ικανή και ανα­γκαία συνθήκη μεταξύ των χ0 , Υ ο ώστε το σύστη-

μα Χο = 6e;4 + κ } (Σ)

να έχει λύση ως προς Υο = 6e; 1 3 _ 2k2

κ ε !R . (Σ) � Κ = Χο - 6e;4 } (Σ') , 2k2 _ 6e - 1 3 - -3-- Υο

οπότε για να

έχει λύση το (Σ ') ως προς κ πρέπει και αρκεί ( 6e - 4 )2 6e - 1 3 , 2 Χ0 --3- = -3- - y0 , δηλαδη τα x0,y0 να

επαληθεύουν την εξίσωση 2 ( 6e - 4 ) 6e - 1 3 ( 6e - 4 )2 y = -2x + 4χ -3- +-3-- 2 -3- , που

είναι εξίσωση παραβολής 4μ)Έχουμε : Re (z) = lm (z) �

6e - 4 6e - 1 3 � --+ κ = --- 2κ2 � 2κ2 + κ + 3 = 0 3 3 που είναι αδύνατη αφού έχει Δ=-23<0. Άρα δεν υπάρ­χουν τέτοιοι μιγαδικοί. 4.() Έστω S = Re ( z) + Im ( z) =

6e - 4 6e - 1 3 2 2 2 2 1 2e - 1 7 -S( ) = -- + κ +--- κ = - κ + κ + - κ 3 3 3 Το τριώνυμο S(κ) έχει α=-2 <Ο οπότε γίνεται μέ-γιστο μόνο για κ = _1._ = J.. , δηλαδή για 2α 4 z = 6e - 4 + .]_ + ( 6e - 1 3 _ .!.) i ο

3 4 3 8 4δ) i) Παρατηρούμε ότι:

) 6e - 1 3 )6e - 1 3 lm (z = 0 � ---2κ2 = Ο � κ = ± --, 3 6 α-• 2, 13 Ά ' δ ' ' δ ' φου e> 7>- . ρα υπαρχουν υο μονο μιγα ικοι 6

αριθμοί z1 , z2 τέτοιο ώστε Im ( z1 ) = Im ( z2 ) = Ο . ii) Τα σημεία Α, Β, Γ της παραβολής δεν είναι συνευθειακά αφού δεν υπάρχει ευθεία με περισσό­τερα από δύο κοινά σημεία με την παραβολή (για­τί;) . Άρα είναι κορυφές τριγώνου. Από το Θ.Μ.Τ. (γεωμετρική ερμηνεία) υπάρχουν χ3 ε (χα , χβ ) , χ4 ε (χβ , χγ ) και χ5 ε (χα , χ8 ) ώστε οι εφαπτόμενες της παραβολής στα σημεία Μ3,Μ4,Μ5 να είναι παράλληλες στις πλευρές ΑΒ, ΒΓ, ΑΓ του

τριγώνου ΑΒΓ δηλαδή να ορίζουν τρίγωνο όμοιο του ΑΒΓ.

η; ( ' ·" Η διάρθρωση της άσκησης είναι διμερής. Δύναται να χωριστεί σε δύο επιμέρους ενότητες. Τα ερωτή­ματα 1 , 2 συνδέουν τα "εργαλεία" που μας παρέ­χουν τα θεμελιώδη θεωρήματα του διαφορικού λο­γισμού με την επίλυση/απόδειξη εξισώσε­ων/ισοτήτων και ανισώσεων/ανισοτήτων .Τα ερω­τήματα 3, 4 (που αποτελούν και την βαθύτερη φι­λοσοφία της άσκησης) συνδέουν τις έννοιες και τα θεωρήματα του διαφορικού λογισμού και του ορι­σμένου ολοκληρώματος (ως εμβαδόν επιπέδου χω­ρίου) με την έννοια του μιγαδικού αριθμού . Ο μιγαδικός αριθμός μπορεί να δράσει στην άσκη­ση και γεωμετρικά. Εξάλλου η αρχική σύλληψη έ­χει τη βάση της στο γεγονός ότι ο μιγαδικός αριθ­μός δεν είναι τίποτα άλλο από ένα σημείο του επι­πέδου. Εντούτοις, στόχος της άσκησης είναι να α­ναδείξει πως ο μιγαδικός αριθμός μπορεί να μελε­τηθεί με τα εργαλεία της ανάλυσης. Επιλέξαμε ο γεωμετρικός τόπος να είναι παραβολή της μορφής y=ax2+ bx + c επειδή αυτή η κωνική το­μή μπορεί να θεωρηθεί άμεσα γραφική παράσταση συνάρτησης. Το πρόβλημα μπορεί να αναδιατυπωθεί και στην περίπτωση που ο γεωμετρικός τόπος είναι π.χ. κύκλος ή έλλειψη με τη διαφορά ότι θα απαιτού­νται παραπάνω από μια συνάρτηση για την μελέτη του μιγαδικού. (π.χ. συνάρτηση με κλάδους) Η άσκηση αναδεικνύει το γεγονός ότι το ορισμένο ολοκλήρωμα δεν είναι τίποτα άλλο παρά ένας πραγ­ματικός αριθμός, ο οποίος μπορεί να "εισχωρήσει" οπουδήποτε δρουν οι πραγματικοί αριθμοί. (π.χ. πραγματικό και φανταστικό μέρος του μιγαδικού) Αντί για ορισμένα ολοκληρώματα , η άσκηση θα μπορούσε να διατυπωθεί και με την εύρεση ορίων συναρτήσεων. Παρόλα αυτά, η μέθοδος εύρεσης εμβαδών επιπέδων χωρίων είναι μέσα στους στό­χους του προβλήματος. Η φιλοσοφία και η στόχευση της άσκησης είναι α­μιγώς διδακτική . Ένα μεγάλο τμήμα της εξεταστέ­ας ύλης της Γ Λυκείου εμπεριέχεται στο πρόβλη­μα. Ο λύτης/αναγνώστης είναι σε θέση να αποκο­μίσει και γενικές μεθόδους προσέγγισης των ασκή­σεων της ανάλυσης. τέλος, το γεγονός ότι το πρό­βλημα "απλώνεται" σε ένα ευρύ φάσμα της ανάλυ­σης και των μιγαδικών της Γ Λυκείου δύναται ως ένα βαθμό να εμπνεύσει όσους ασχολούνται με τα Μαθηματικά για την εξαγωγή ποικίλων πρωτότυ­πων προβλημάτων.

l:ι ιι:ίωσι : Το άρθρο αυτό ήταν να δημοσιευτεί στο τεύχος

80(τ.4), επειδή υπήρχε πολύ επίκαιρη ύλη δε δημο­σιεύθηκε. Έχει όμως αρκετό ενδιαφέρον και το δημο ­

σιεύουμε στο τεύχος 81 . Με χαρά μάθαμε ότι ο aρ­θρογράφος είναι φοιτητής πλέον της Ιατρικής Αθηνών (9ος _ στη σειρά) . Η συντακτική επιτροπή του περιοδι­κού του εύχεται καλή τύχη και καλή σταδιοδρομία.

ΕΥΚΛΕΙΔΗΣ Β' 81 τ. 1175

Page 78: Ευκλειδης Β 81

_#jJ Ο Εuκλείίδης «Η καρδιά των μαθηματικών είναι τα προβλήματα και οι λύσεις και ο κύριος λόγος ύπαρξης � προτεiνειι .. .. .. του μαθηματικού είναι να λύνει προβλήματα».

P. R. HALMOS Επιμέλεια: Γ. Κ. ΤΡΙΑΝΤΟΣ - Ν. Θ ΑΝΤΩΝΟΠΟΥ ΛΟΣ - Θ. Α. ΤΖΙ ΩΤΖΙ ΟΣ

γαπητοί φίλοι και συνάδελφοι, μία σειρά από aτυχείς συγκυρίες και καταστάσεις στο τεύχος 80, σας άφησαν ολόκληρο το Καλοκαίρι χωρίς προτεινόμενα θέματα. Για τον λόγο αυτό, στο παρόν τεύχος, η στήλη

καινοτομεί. Δεν θα δώσει τις λύσεις ασκήσεων προηγουμένων τευχών, παρά μόνον προτεινόμενα θέματα, που παρατίθενται αμέσως μετά την παρουσίαση μιας ενδιαφέρουσας εργασίας του φίλου της στήλης, Χημικού Δημήτρη Καρβελά, με θέμα την εύρεση των ακεραίων λύσεων εξισώσεων με πολύ μεγάλους συντελεστές καθώς και αντίστοιχων Διοφαντικών συστημάτων. Σας ευχόμαστε καλή σχολική χρονιά. Αναμένουμε πάντα τις εργασίες και τις προτάσεις σας. Οι εργασίες που αποστέλλονται ηλεκτρονικά ,είναι πολύ σημαντικό, να σώζονται με τέτοιο τρόπο, έτσι ώστε να αναγνωρίζονται από το Word office 2003 . Υπενθυμίζουμε τις ηλεκτρονικές διευθύνσεις των επιμελητών της στήλης: gktriantos@gmail .com -Nikan [email protected] - [email protected] .

ΕΠΙΛ ΥΣΗ ΤΗΣ ΓΡΑΜΜΙΚΗΣ ΔιοΦΑΝτΙΚΗΣ ΕΞΙΣΩΣΗΣ αχ + βy = γ.

(Από τον ΔΗΜΗΤΡΙΟ ΚΑΡΒΕΛΑ - Πεύκη ) Γνωρίζουμε ότι η γραμμική διοφαντική εξίσωση αχ + βy = γ, με α, β, γ Ε Ζ και α·β ::F Ο, ( 1 ) έχει ακέραιες λύσεις αν και μόνο αν ο μ.κ.δ (α,β) των α,β διαιρεί τον γ. Άρα η γενική περίπτωση, που επιλύεται και μας ενδιαφέρει, είναι όταν (α, β) = 1 . Αν λοιπόν έχουμε μια ακέραια λύση της (μερική) την (χ, y) = (χο , Υο) τότε, έχουμε άπειρες ακέραιες λύσεις, που δίνονται από τις σχέσεις: χ = χ0 + βt, y = Υο - αt, με t Ε Ζ. Στις εξισώσεις όμως που τα α, β, γ είναι ακέραιοι με πολύ μεγάλες απόλυτες τιμές ή παραμετρικά δοσμένοι, τότε η εύρεση της λύσης παρουσιάζει ιδιαίτερη δυσκολία. Μια εναλλακτική μέθοδος είναι η ακόλουθη . Χωρίς βλάβη της γενικότητας υποθέτουμε ότι Ιαl < l β l . Από την Ευκλείδεια διαίρεση έχουμε: β = απ1 + υ ι , με Ο � υι < Ι α l και γ = απ2 + υ2, με Ο � υ2 < Ι α Ι και Πι , π2, υ ι , υ2 Ε Ζ. Οπότε: ( 1 ) <:::> αχ + απιy + υ ιy = απ2 + υ2 <:::> α(χ + ΠιΥ - π2) + υ ιy = υ2. Θέτοντας χ + π1y - π2 = z Ε Ζ προκύπτει η ισοδύναμη της ( 1 ) : αz + υ 1y = υ2, (2) με

αγνώστους τους z, y και συντελεστές με μικρότερες απόλυτες τιμές από τους συντελεστές της αρχικής. Επαναλαμβάνοντας την ίδια διαδικασία και στην (2) μπορούμε να βρούμε μια ισοδύναμη της με ακόμη μικρότερες απόλυτες τιμές των συντελεστών των αγνώστων κ.ο.κ. Κατά αυτόν τον τρόπο καταλήγουμε σε μια ισοδύναμη διοφαντική της ( 1 ) με συντελεστή τουλάχιστον του ενός αγνώστου την μονάδα. Άρα έχουμε την δυνατότητα να βρούμε τον ένα βοηθητικό άγνωστο, χρησιμοποιώντας ως ακέραια παράμετρο τον άλλον. Κατόπιν με διαδοχικές αντικαταστάσεις των βοηθητικών αγνώστων στις εξισώσεις που έχουν προκύψει, υπολογίζουμε τις τιμές των αρχικών αγνώστων. Ως παράδειγμα των παραπάνω θα αναφερθούμε στην εύρεση των ακεραίων λύσεων της εξίσωσης 1 1 1 χ + 7777777777y = 999999999. ( 1 ) Λί>ση : Από τους αλγόριθμους των διαιρέσεων έχουμε: 7777777777 = 1 1 1π 1 + 7 και 999999999= 1 1 1 π2+0, με π, = 70070070, π2 = 9009009 (2)

(2) Επομένως, (1) <:::> 1 1 1χ + (1 1 1π 1 + 7)y = 1 1 1π 2 ή 1 1 1(χ + π 1 y - π 2 ) + 7y = Ο (3) . Θέτουμε

( 4) χ + π 1 y - π2 = z (4) ,οπότε (3)<:=> 1 1 1z + 7y = O ( 5) . Όμως, 1 1 1 = 7 · 1 5 + 6. έτσι, η ( 5) γράφεται: (7 · 1 5 + 6)z + 7y = Ο <:::> 7(1 5z + y) + 6z = Ο (6). Θέτουμε 1 5z + y = w (7) και η (6) γράφεται: 7w + 6z = Ο (8) . Επειδή είναι 7 = 6 · 1 + 1 , η (8) παίρνει τη μορφή : (6 · 1 + 1)w + 6z = Ο που ισοδυναμεί με την 6(w + z) + w = Ο (9). Θέτουμε w + z = k ( 1 0) , οπότε η (9) δίνει: 6k + w = Ο ή w = -6k ( 1 1 ) . Από την ( 1 0) ,λόγω της ( 1 1 ) , παίρνουμε : - 6k + z = k <:::> z = 7k ( 1 2). 'Ετσι,

( \ \ ) (7)<:::> 15 · 7k + y = -6k <:::> y = -1 1 1k ( 1 3) . ( 1 2 ) ( 1 2 ) (4) <:=> χ + π , (- 1 1 1k) - π2 = 7k (1 3 ) <:=> χ = 1 1 1π 1 k + 7k + π2 = (1 1 1π 1 + 7)k + π2 = = 7777777777 · k + 9009009 , k Ε Ζ

Παρατήρηση Ι : Κατά την διαδικασία επίλυσης της ( 1 ) , στο προηγούμενο παράδειγμα, είναι φανερό ότι η μετάβαση από μία εξίσωση σε μια ισοδύναμή της επιτυγχάνεται με την χρήση των υπολοίπων των διαιρέσεων που εκτελέσαμε και

ΕΥΚΛΕΙΔΗΣ Β' 81 τ.l/76

Page 79: Ευκλειδης Β 81

-------------- Ο Ευκλείδης προτείνει . . . -------------­

αλλαγή του αγνώστου που έχει συντελεστή τον λύση διοφαντικών συστημάτων. Για την εφαρμογή διαιρέτη . Έτσι, η διαδικασία απλουστεύεται, όπως της μεθόδου δίνουμε το παρακάτω παράδειγμα. φαίνεται στο επόμενο παράδειγμα: Να βρεθούν οι ακέραιες λύσεις του συστήματος. Ν α βρεθούν οι ακέραιες λύσεις της εξίσωσης: { 17χ + 2y + 5z + 7w = 263

2439χ + 2008y = 2179 (1) 19χ + 3y + 2z + 10w = 284 (1) Λύση Επειδή οι διαιρέσεις 2439 : 2008 , 2 1 79 : 2008 7χ + 5Υ + 4z + 13w = 275 δίνουν υπόλοιπα 43 1 , 1 7 1 αντιστοίχως, έχουμε (1) <::> 43 1χ + 2008a = 1 7 1 (2) . Επειδή οι διαιρέσεις 2008 : 43 1 , 1 7 1 : 43 1 δίνουν υπόλοιπα 284 και 1 7 1 αντιστοίχως, έχουμε και πάλι ότι: (2) <::> 43 1b + 284a = 1 7 1 (3) . Συνεχίζοντας με τον ίδιο ακριβώς τρόπο παίρνουμε τις ισοδυναμίες (3) <::> 147b + 284c = 1 7 1 <::> 147d + 1 37c = 24 <::> 1 0d + 1 37e = 24 <::> 1 0f + 7e = 4 <::> 3m + 7η = 4 <::> 3k + η = 1 <::> η = 1 - 3k , k Ε Ζ Αν τελευταία τιμή αντικατασταθεί στην προηγούμενη εξίσωση βρίσκουμε την τιμή του m. Με διαδοχικές προς τα πίσω αντικαταστάσεις εύκολα καταλήγουμε στις τιμές: x = -363 + 2008k και y = 442 - 2439k , k E Z . Με την ίδια μέθοδο μπορούν να λυθούν και ορισμένες εγγράμματες εξισώσεις, όπως για π.χ η εξίσωση : ( 1 ) (2η + 1)χ + (2η + 3)y = 2η + 5 , όπου η γνωστός ακέραιος αριθμός. Έχουμε, λοιπόν, ότι (1) <::> (2η + 1)a + 2y = 4 <::> a + 2k = Ο <::> a = -2k . Τότε, y = 2 + (2η + 1)k , χ = -1 - (2η + 3)k , k Ε Ζ . Παρατήρηση 2 : Η μέθοδος που περιγράψαμε επεκτείνεται και στην περίπτωση πολυωνυμικής εξίσωσης f(χι , χ2, • • • , xv) = Ο, όπου οι συντελεστές των αγνώστων είναι ακέραιοι και υπάρχουν δύο τουλάχιστον άγνωστοι που είναι μόνο στον πρώτο βαθμό και οι αντίστοιχοι συντελεστές τους πρώτοι μεταξύ τους. Αυτό επιτυγχάνεται επιλύοντας την ως προς τους αγνώστους αυτούς και θέτοντας το άλλο μέλος της ισότητας ως μεταβλητή. Παράδειγμα: Να βρεθούν οι ακέραιες λύσεις της εξίσωσης: 3χ + 7y + 5z2 + 2zw + 5w = 1379 (1) Λύση (1) <::> 3x + 7y = 1 379 - 5z 2 - 2zw - 5w (2) Αν τεθεί 1 379 - 5z 2 - 2zw - 5w = m E Z (3), η (2) δίνει 3χ + 7y = m (4). Σύμφωνα με τα παραπάνω οι λύσεις της ( 4) στο σύνολο των ακεραίων δίνονται από τις σχέσεις: χ = -2m + 7k και y = m - 3k με k, w, z E Ζ . 'Ετσι , για w = 1, z = -2 και k = 7 έχουμε: m = 1 3 88, χ = -2727, y = 1 367 . Δηλ. μία λύση της ( l ) είναι η (x, y, z, w) = = ( - 2727 ' 1 367 ' - 2 ' 1 ) . Π αραηΊρηση 3 : Φυσικά την μέθοδο αυτή μπορούμε να την χρησιμοποιήσουμε και για την

Λύση Η πρώτη των εξισώσεων του συστήματος γράφεται 1 7 χ + 2y = 263 - 5z - 7w και σύμφωνα με τα προηγούμενα έχει λύσεις : χ = 1 - 2a - b - c , y = 1 23 + 1 7a + 6b + 5c , z = b , w = c σι ) , όπου a, b, c Ε Ζ . Αντικαθιστούμε τις λύσεις αυτές στις άλλες δύο εξισώσεις του συστήματος και έχουμε: 1 3a + b + 6c = - 1 04 (2) , 7 1a + 27b + 3 1c = -34 7 (3) Οι λύσεις της (2) δίνονται από τις ισότητες: a = d , b = - 1 04 - 1 3d - 6e , C = e σ 2 ) με d, e Ε Ζ . Αντικαθιστούμε τις λύσεις αυτές στην εξίσωση (3) και έχουμε: 280d + 1 3 1e = -246 1 (4) . Οι λύσεις της ( 4) δίνονται από τις σχέσεις: d = - 1 3 - 1 3 1k , e = 9 + 280k , k E Z σ3 ) . Αντικαθιστώντας τις τιμές σ3 ) στις τιμές σ2 ) έχουμε: a = - 1 3 - 1 3 1k, b = 1 1 + 23k , c = 9 + 280k Τέλος, με αντικατάσταση των τιμών αυτών στις τιμές σι ) έχουμε τις ακέραιες λύσεις : χ = 7 - 4 1k, y = 1 3 - 689k, z = 1 1 + 23k, w = 9 + 280k ΗΡΟΤΕΙΝΟ ΜΕΝΑ Θ Ε Μ ΑΤΑ 1 94. Να βρεθούν οι ακέραιες λύσεις της εξίσωσης. (2z + 3w)(x + y - 1) + y = 2 ( 1 ) (Από τον κ . Δ ΗΜΗΤΡΙΟ ΚΑΡΒ ΕΛΑ - Πεύκη) 1 95. Να λυθεί στο σύνολο R η εξίσωση :

3χ 2 +6 6χ 2 +3 ι χ 2 +ι 2 Χ · 1 5 Χ + 2 Χ · 1 5 Χ + 1 36 Χ = 33856

(Από τον Γ. ΑΠΟΣΤΟΛΟΠΟΥ Λ Ο- Μεσολόγγι) 1 96. Α ν οι ετερόσημοι πραγματικοί αριθμοί χ , y

και ο α Ε R ικανοποιούν τη σχέση : χ 2 Υ2 2 5 χ Υ 2(- +-) · a - - · (- + -) · a = 3 y2 χ 2 2 Υ χ

Να βρεθεί η μέγιστη και η ελάχιστη τιμή του a . (Προτείνεται από τον μαθητή ΚΩΝΣΤ ΑΝΠΝΟ

ΤΣΟΥΒΑΛΑ - Μύρινα Λήμνου) 1 97. Δίνεται αμβλυγώνιο τρίγωνο ΑΒΓ. Να αποδειχθεί ότι ο περιγεγραμμένος κύκλος του, ακτίνας R, τέμνει τον κύκλο Euler του τριγώνου αυτού κατά χορδή μήκους d = 4R p(p + 1) , όπου p = συνΑσυνΒσυνΓ . 8p - 1 ( Προτείνεται από τον συνεργάτη τη; στήί.η ;

ΠΩΡΓΟ ΤΡΙΑΝΤΟ - Αθήνα ) ΕΥΚΛΕΙΔΗΣ Β ' 81 τ.l/77

Page 80: Ευκλειδης Β 81

Τα Μαθηματικά μας διασκεδά�ουν «Η ζωή έχει νόημα, για δύο τουλάχιστον πράγμα­τα : πρώτο να μελετά κα­νείς Μαθηματικά και δεύ­τερο να τα διδάσκει».

Simeon Denis Poisson

Όταν πριν από αρκετά χρόνια ξεκινήσαμε την στήλη αυτή «Τα Μαθηματικά μας διασκεδάζουν» γράφαμε: Τα Μαθηματικά αν και είναι επιστήμη που απαιτεί αυστηρή διατύπωση, έχουν τη μαγεία να αποσπούν το εν­διαφέρον όλων των ανθρώπων. Επινοήσεις σε προβλήματα ή ασκήσεις με κατάλληλο τρόπο διατυπωμένα ε­ξάπτουν το πνεύμα, διεγείρουν τη φαντασία και κεντρίζουν την περιέργεια. Πρώτοι οι Αρχαίοι Έλληνες όπως ο Διόφαντος, ο Ζήνωνας μας δίδαξαν αυτά τα Μαθηματικά.

Στη σrήλη αυτή θα παρουσιάζουμε θέματα τα οποία δεν απαιτούν ιδιαίτερες μαθηματικές γνώσεις αλλά μας διασκεδά­ζουν με την εκφώνησή τους ή τη λύση τους και είναι μια ευχάριστη και συναρπαστική ασχολία .

Σήμερα εφτά χρόνια μετά, αισθανόμαστε την ανάγκη να ευχαριστήσουμε όλους όσους έδειξαν ενδιαφέρον για τη στήλη και με τα καλά τους λόγια ή τις παρατηρήσεις τους βοήθησαν στη συνεχή βελτίωσή της. Για πάρα πολλά χρόνια ασχοληθήκαμε με τα θέματα αυτά, που είναι το καθαρό απόσταγμα από την επιστημονική έρευνα ή είναι παιχνίδια λογικής βγαλμένα από την σοφία της ανθρωπότητας. Είναι θέματα δοσμένα και διατυπωμένα με έξυπνο τρόπο που κεντρίζουν την περιέργειά και διεγείρουν την φαντασία.

Η πρώτη μας επαφή με τα μαθηματικά αυτά, όπως και σε όλους σας πιστεύω, ήταν στην παιδική ηλικία όταν οι μεγαλύτεροι δεν ρωτούσαν, αν ξέρεις τον Σωκράτη, τον Παπαδιαμάντη ή τον Σολωμό, αλλά ρωτούσαν αν ξέρουμε την προπαίδεια ή τη λύση κάποιου προβλήματος-γρίφου που η διατύπωσή του δεν ήθελε μαθηματική γλώσσα.

Αυτοί οι γρίφοι-προβλήματα οφείλονται στο Riemann, τον Eίiler, τον Gauss, τον Gδdel , τον Poincare, τον Pascal, τον Fibonacci, τον Νεύτωνα, τον Διόφαντο, τον Ζήνωνα και πάρα πολλούς άλλους επιστήμονες που αντιμε­τώπισαν τα προβλήματα αυτά στην έρευνά τους, για να τα δώσουν στη συνέχεια σε μας, ως παιχνίδια.

Στο προηγούμενο τεύχος είχαμε γράψει για τις παράξενες πράξεις τα αριθμητικά «τοπία» όπως τα ονομάσαμε. Με αφορμή την παρατήρηση του συναδέλφου Κώστα Τσομπανίδη από τη Δράμα, τον οποίο και ευχαριστούμε, δημοσιεύουμε ποιό κάτω και πάλι όμορφα αριθμητικά «τοπία» με τις δυνάμεις του λόγου iης χρυσιlς τομής και την ακολουθία Fibonacci .

Γνωρίζετε ότι: Ο Λεονάρντο Φιμπονάτσι [(Fibonacc·ί) = γιος του Μπονάτσι(=απλός)] ή Λεονάρντο της Πίζας ή Λεονάρντο Πιζάνο(1 170-1250) είναι Ιταλός μαθηματικός ο ο­ποίος άλλαξε την πορεία του δυτικού πολιτισμού, εισήγαγε στην Ευρώπη το αραβικό δεκαδι­κό σύστημα αρίθμησης, το σύμβολο για το μηδεv και την υποδιαστολή .

Σε μια σκοτεινή εποχή για τις επιστήμες στην Ευρώπη το 1 202 ο Fibonacci δημοσιεύει το LIBER ABACI ή βιβλίο των υπολογισμών, γεμάτο με τις μαθηματικές γνώσεις που είχε περισυλλέξει στα ταξίδια του, που έκανε μαζί με τον έμπορο πατέρα του, στη Βόρειο Α­φρική. Στο βιβλίο αυτό έδειχνε την πρακτικότητα του αραβικού αριθμητικού συστήματος στην τήρηση εμπορικών βιβλίων, στις χρηματικές συναλλαγές, τις μετατροπές των μέτρων και σταθμών, στον υπολογισμό των επιτοκίων και άλλες εφαρμογές. Το βιβλίο έτυχε θερ-

ΕΥΚΛΕΙΔΗΣ Β' 81 τ.l /78

Page 81: Ευκλειδης Β 81

-------------- Τα Μαθηματικά μας Διασκεδάζουν ------------­

μής υποδοχής ανάμεσα στους λόγιους της Ευρώπης και τους επηρέασε σημαντικά, όμως το αραβικό σύστημα αρίθ­μησης έγινε ευρέως γνωστό μετά την εφεύρεση της τυπογραφίας. Ο Ιωάννης του Παλέρμο, μέλος της αυλής του Φρειδερίκου Β', παρουσιάζει στον Fibonacci έναν αριθμό προβλημάτων-προκλήσεων, από τα οποία έλυσε τα τρία. Ένα από αυτά ήταν το εξής:

Κάποιος τοποθέτησε σε έναν αποκλεισμένο τόπο ένα ζευγάρι κουνελιών. Τα κουνέλια αυτά αναπαράγονται με ρυθμό ένα νέο ζευγάρι το μήνα και κάθε νέο ζευγάρι γίνεται γόνιμο δύο μήνες μετά κι αναπαράγεται με τον ί­διο ρυθμό. Πόσα ζευγάρια κουνελιών έχουν παραχθεί σε έναν χρόνο από το αρχικό ζεύγος; Το αποτέλεσμα είναι η ακολουθία των αριθμών Ο, 1 , 1, 2, 3, 5, 8, 13, 21 , 34, 55, 89, 144, 233, 377, 610, 987, 1 597, 2584, 4181 , 6765, 10946 . .. γνωστή ως ακολουθία Fibonacci, μια ακολουθία εξαιρετικά χρήσιμη στην επι­στήμη που κάθε νέος όρος της είναι το άθροισμα των δύο προηγούμενων όρων. Οι όροι της ακολουθίας ορίζονται από τον εξής αναδρομικό τύπο : α.=α._1+α,_2 με α0=0 και α1=1 .

Ο Fibonacci πίστευε ότι αυτοί οι αριθμοί μπορούν να ξεκλειδώσουν τα μυστικά της Φύσης. Μπορούμε να το αντιληφθούμε αυτό αν λάβουμε υπόψη πως η ακολουθία, καθώς και η λογαριθμική σπείρα που δημιουργείται σε σχέση με τον Φ απαντώνται σχεδόν παντού : Στη Φυσική, τη Βιολογία, τη Βοτανολογία, την Αστρονομία, την Πυρηνική Φυσική, την Οικονομία, την Εκπαίδευση, την Ποίση, την Μουσική , την Αρχαιολογία, την Αρχιτεκτονική, την Τέχνη και είναι μια βάση για ιίίταiίΦΙj�ιi

Η λογαριθμική σπείρα Fibonacci Εξετάζουμε το λόγο δύο διαδοχικών αριθμών της ακολουθίας 2/ 1 =2 , 3/2= 1 . 5 , 5/3= 1 .666 . . . , 8/5= 1 .6 ,

1 3/8= 1 .625 , 2 l / l 3= l .6 1 5 . . . , . . . . . . , 1 0946/6765= 1 ,6 1 803 . . . , . . . . . . παρατηρούμε ότι τείνει στον Φ, όσο δηλαδή διαι­ρούμε μεγαλύτερους όρους της ακολουθίας ο λόγος προσεγγίζει όλο και περισσότερο τον γνωστό "χρυσό λόγο"

που είναι ίσος με τον άρρητο αριθμό Φ = 1 + .J5 = 1 , 6 1 8033 . . . (Φ προς τιμήν του γλύπτη Φειδία που εφάρ-

2 μοζε το χρυσό 'λόγο στα έργα του). Ας θαυμάσουμε την αρμονία του «τοπίου» των σχέσεων με το Φ.

I Φ1= 1Φ+Ο

I Φ�=8Φ+5

I Φ11=89Φ+55

I Φ�= Ι Φ+ Ι

I Φ'= 1 3Φ+8

I Φ12= 1 44Φ+89

I Φ�=2Φ+ Ι

I Φ5=2 ΙΦ+ Ι 3 I Φ13=233Φ+ 1 -1-1

I Φ4=3Φ+2 I Φ5=5Φ+3 I Φ9=34Φ+2 1 I Φ10=55Φ+.1-Ι I φ14=377Φ+233 I ψ•=v"'όρος.Φ+(ν"c- 1 )

Παρατηρούμε ότι στο δεύτερο μέρος της ισότητας οι συντελεστές του Φ είναι όροι της ακολουθίας Fibonacci, ενώ οι στα­θεροί όροι που προστίθενται είναι επίσης όροι της ακολουθίας Fibonacci κατά μια θέση μικρότεροι.

Η Ανθοδέσμη Μια ανθοδέσμη έχει γαρδένιες, γλαδιόλες και γαρίφαλα. Όλα τα λουλούδια της aνθοδέσμης είναι γαρδένιες ε­

κτός από τέσσερα, όλα είναι γλαδιόλες εκτός από τέσσερα και όλα είναι γαρίφαλα εκτός από τέσσερα. Πόσα λου­λούδια από κάθε είδος έχει η ανθοδέσμη;

ΑΨΟΥ- Στην υγειά σαι:: Όταν φτερνίζεστε οι φίλοι σας συνήθως λένε ένα τριψήφιο αριθμό, εσείς αθροίζετε τα ψηφία του μέχρις ότου

το άθροισμα να είναι <ή=του 24 για να βρείτε το αντίστοιχο γράμμα της ΑΒητα. Έχουν όμως όλα τα γράμματα την ίδια συχνότητα εμφάνισης; (από τον Π.Θ. Χριστόπουλο)

Παράξενος αριθμός

Στις 1 Ο θέσεις του παραπάνω σχήματος γράψετε έναν αριθμό με δέκα ψηφία έτσι ώστε, το ψηφίο στην πρώτη θέση να δείχνει τον συνολικό αριθμό των μηδενικών του αριθμού, το ψηφίο στη θέση με την ένδειξη 1 να δείχνει τον συνολικό αριθμό των Ι , το ψηφίο στη θέση με την ένδειξη 2 να δείχνει τον συνολικό αριθμό των 2 και ούτω

ΕΥΚΛΕΙΔΗΣ Β' 81 τ. 1179

Page 82: Ευκλειδης Β 81

------------- Τα Μαθηματικά μας Διασκεδάζουν ------------­

καθεξής, μέχρι που στην τελευταία θέση, να δείχνει τον συνολικό αριθμό των 9 στον αριθμό. (Έχει ξαναδημοσιευθεί

στο 76° τεύχος)

Ο μάντης με τα φρούτα Ο μάντης Frouazi έχει τρία κλειστά πανέρια που το καθένα έχει και μία επιγραφή . Η πρώτη γράφει "Μήλα" , η

δεύτερη γράφει "Πορτοκάλια" και η τρίτη γράφει "Μήλα και Πορτοκάλια" . Ξέρουμε ότι και οι τρεις επιγραφές είναι τοποθετημένες λάθος. Ο Frouazi μας ζητά να βγάλουμε ένα μόνο φρούτο από ένα μόνο πανέρι και χωρίς να κοιτάξουμε μέσα ή να ψαχουλέψουμε, να βάλουμε καθεμιά από τις επιγραφές στο σωστό πανέρι. Μπορείτε;

Ο Γρίφος του Einstein Η Καρσανίδου Κατερίνα μας έστειλε την πιο κάτω λύση για τον γρίφο του Einstein που είχαμε δημοσιεύσει στο 70 τεύχος το 2008. Η Κατερίνα έκανε αρίθμηση των σπιτιών από δεξιά προς τα αρι­στερά και η λύση της είναι διαφορετική από τη δική μας που η αρίθμηση των σπιτιών είχε γίνει από αριστερά προς τα δ εξιά. Μπράβο της.

Σπίτι 1 0 20 30 40 50 Χρώμα κίτρινο μπλε άσπρο πράσινο κόκκινο Εθνότητα Νορβηγός Δανός Γερμανός Σουηδός Άγγλος Ποτό νερό τσάι γάλα καφέ μπίρα Τσιγάρα ΡΜ Bl Pr Du ΒΕ Κατοικίδιο πουλιά ; γάτες σκύλος άλογο

Άρα το Ψ ΑΡΙ είναι στο δεύτερο σπίτι που μένει ο Δανός. Ευχαριστούμε την Κατερίνα Καρσανίδου, μαθήτρια Β 'Λυκείου από τη Βέροια για τη λύση στον γρίφο του Ein­stein, τον συνάδελφο Κώστα Τσομπανίδη για την εύστοχη αρατήρηση, του συνάδελφου Γιώργο Πλατάκη και τον Μηχανολόγο της Ολυμπιακής Αεροπορίας Πέτρο Θ. Χριστόπουλο για τους ωραίους γρίφους που μας έστειλε.

Η Ανθοδέσμη Δύο από κάθε είδος διότι αν τα λουλούδια της aν­

θοδέσμης είναι όλα Χ τότε έχουμε: (Χ-4)+(Χ-4)+(Χ-4)=Χ ή 2Χ= 12 ή Χ=6

Α ΨΟΥ- Στην υγειά σας Προφανώς όχι, δείτε το αποτέλεσμα.

Τελικό Εμφανίσεις Αντίστοιχο άθροισμα αριθμού Γράμμα

1 3 Α 2 6 Β 3 1 0 Γ 4 1 5 Δ 5 2 1 Ε 6 28 z 7 42 Η 8 48 Θ 9 56 Ι 1 0 63 κ 1 1 69 Λ 1 2 73 Μ 1 3 75 Ν 1 4 7 5 -

1 5 73 ο 1 6 69 π 1 7 63 Ρ 1 8 55 Σ 1 9 45 τ 20 36 Υ 2 1 28 φ 22 2 1 χ 23 1 5 ψ 24 1 0 Ω

Σύνολο 999

0/ο

0,3 0,6 1 ,0 1 ,5 2, 1 2,8 4,2 4,8 5,6 6,3 6,9 7,3 7,5 7,5 7,3 6,9 6,'3 5,5 4,5 3 ,6 2,8 2, 1 1 ,5 1 ,0

1 00

. . . •· •• ΙιιιΙΙι • Σtιρά1

Α θ Γ Δ Ε Ζ Η Θ I Κ Λ Μ Ν Ξ Ο Π Ρ Ι Τ Υ Φ Χ Ψ Ω

Τυχερά τα Ν και Ξ. Παράξενος αριθμός: Αν π.χ. τα μηδενικά ήταν 9

στην 1 η θέση θα γράφαμε 9 και στην τελευταία 1 άτοπο γιατί μένουν 8 θέσεις, αν τα μηδενικά ήταν 8 στην 1 η θέση θα γράφανε 8 στην 9η θέση γράφουμε 1 και στην δεύτερη θέση επίσης 1 άτοπο γιατί μένουν 7 θέσεις, κ.ο.κ. οπότε αν τα μηδενικά είναι 6 γράφουμε στην πρώτη θέση 6 στην 7η θέση γράφουμε 1 αλλά αν στη 2η θέση γράψουμε 1 θα εί­ναι λάθος γιατί θα γίνουν 2 οι αριθμοί 1 , γι' αυτό στη 2η θέση θα βάλουμε 2 και στην 3η θέση 1 και ο αριθμός είναι 62 1 000 1 000 ο οποίος είναι μοναδικός.

Ο μάγος με τα φρούτα Θα πάρουμε ένα φρούτο από το πανέρι που έχει την επι­

γραφή "Μήλα και Πορτοκάλια" αν αυτό είναι π.χ. μήλο, τότε σε αυτό το πανέρι θα βάλουμε την επιγραφή "Μήλα". Στο πανέρι τώρα που είχε την επιγραφή "Μήλα" θα βάλουμε την επιγραφή "Πορτοκάλια", και τέλος στο πανέρι που είχε την επιγραφή "Πορτοκάλια" θα βάλουμε την επιγραφή "Μήλα και Πορτοκάλια".

ΕΥΚΛΕΙΔΗΣ Β' 81 τ.l/80

Page 83: Ευκλειδης Β 81

Α9. Αν μεταξύ των πλευρών α,β,γ τριγώνου ΑΒΓ ισχύ­ει η σχέση α2 + β2 > 5γ2 , τότε η πλευρά γ είναι η μι­κρότερη πλευρά του τριγώνου

Κώστας Γ. Σάλαρης, μαθηματικός Αθήνα Λίιση : Έχουμε: β + γ > α =::> β2 + 2βγ + γ2 > α2 =::> =::> 2βz + 2βγ + γz > αz + βz > Sγ2 =::> 2β2 + 2βγ - 4γ2 > ο =::> β2 + βγ - 2γ2 > ο =::> =::> (β-γ) (β+2γ) > Ο=::>β > γ . Ομοίως: α + γ > β =::> α > γ .

(Με απαγωγή σε άτοπο) Θέλουμε να δείξουμε τη συνεπαγωγή : α2 + β2 > 5γ2 =::> {α > γ και β > γ ) . Έστω ότι δεν ισχύει.

Τότε θα έχουμε: <i +β2 >V { Ι) και (α ::; γ ή β ::; γ ) . ( Ι )

i) Αν α ::; γ, τότε: α2 ::; γ2 =::> -α2 � -γ2 =::> βz � Sγz _ αz � Sγ2 _ γz = = 4γz =::> β � 2γ . Αλλά γ � α, οπότε β + γ � 2γ + α =::> β � α + γ , ενώ β < α + γ, πράγμα άτοπο. i i ) Αν β ::; γ, εργαζόμαστε ομοίως. Λύσεις (AS, Α9) Κ. Καββαδίας (2° Λύκειο Κέρκυρας)

Αν οι πλευρές ενός τριγώνου ΑΒΓ έχουν μήκη τους ακεραίους αριθμούς α,β ,γ και το ένα ύψος του ισούται με το άθροισμα των άλλων δύο τοτε να δείξετε ότι ο α­ριθμός α2 + β2 + γ2 είναι τέλειο τετράγωνο ακεραίου.

Κώστας Γ. Σάλαρης, μαθηματικός Αθήνα Λύση Έστω υ" = υβ + υγ ( 1 ) . Τότε:

2Ε 2Ε 2Ε ( Ι ) =::> - = - + - =::> α β + α γ = β γ =::>

α β γ

α2 + β2 + γ2 = α2 + β2 + γz + 2 (βγ - αβ - αγ) = (β + γ - α)z

Τ ' l Ω� ,!_-η -':' .. ·\_υ �c,l : io� ) - / Να δείξετε ότι η συνάρτηση f ( χ ) = ln χ , δεν είναι

πολυωνυμική . Μήπως είναι ρητή συνάρτηση; Άννυ Φανέλη, μαθηματικός Αθήνα

Λύση Έστω : Ιη χ = αv χν + αν- ι χ v- ι + . . . + α1 χ + α0 , για κάθε χ > ο , όπου αν ::�= ο . Τότε :

ιη.! = α_, + α_,_ι + . . . +3_ +au =::> -lnx= α_, + α_,_ι + . . . +3_+au =::> χ χν xv-1 χ xv χ''-1 χ

Επιμέλεια: Αντώνης Κυριακόπουλος Γιώργος Τασσόπουλος

2 v β 2ν- 1 β Ο αν χ + 2 ν- 1 χ + . . . + ι χ + αν = ' για κάθε χ > Ο , δηλαδή για περισσότερες από 2ν τιμές του χ . Άρα : αν = Ο , ενώ αν ::1= Ο (άτοπο) Έστω :

Q(χ) = βμ χμ + . . . + β 1 χ + β0 , όπου ανβμ ::Ι= Ο και

ln x = ��:� ( Ι ) για κάθε χ > Ο , με Q ( x ) ::�= O .

, Ι P ' ( x ) Q ( x ) - Q' ( x ) P ( x ) Τοτε: ( I ) =:> - = 2 ( )

=::> χ Q χ

Q2 ( χ ) = Α ( χ ) (2), 'Οπου βαθ Q2 ( χ ) = 2μ ,

Α ( χ ) = χ[(αv · νχ ν- ι + . . . + αι ) · (βμ χ μ + . . . + β ι χ + βο ) ­- (βμ · μχμ- ι + . . . + β ι ) · (ανχ ν + . . . + αι χ + αο )J = ανβμ (ν - μ) χ ν+μ + γν+μ- Ι Χ

ν+μ- 1 + . . . + γ ι Χ και η σχέση (2) ισχύει για περισσότερες από το max {2μ, μ + ν} , τιμές του χ.

Α ν μ = ν , τότε: βαθ Α ( χ ) ::; ν + μ - I , οπότε : ( I ) =::> 2μ ::; ν + μ - I =::> μ + I ::; ν =::> μ < ν , ενώ μ = ν (άτοπο)

Α ν μ ::1= ν , τότε: βαθ Α ( χ ) = ν + μ , οπότε: ( I ) :::::> 2 μ = ν + μ =::> μ = ν , ενώ μ ::1= ν (άτοπο)

Η μέθοδος της παραγώγισης μπορεί προφανώς να εφαρ­μοστεί και στην πρώτη περίπτωση . Προτιμήσαμε όμως στην περίπτωση αυτή τη στοιχειώδη απόδειξη , η οποία μπορεί να γίνει και στη Β Λυκείου .

Να δείξετε ότι μόνο οι τριάδες ( Ι ,λ,-λ), (λ, Ι ,-λ), x+y+ω= l }

(λ,-λ, Ι ), λε R*, επαληθεύουν το σύστημα: _!_+_!_ +_!_ = I {Σ) χ Υ ω

Θεωρούμε ημικύκλιο (Κ, ρ) διαμέτρου ΑΒ. Η κά­θετη στην ΑΒ στο μέσο Γ του ΚΒ, τέμνει το ημικύκλιο στο Δ και η ΔΑ τέμνει τα ημικύκλια διαμέτρων ΑΚ, ΑΓ, στα Ε,Ζ αντιστοίχως. Να βρεθεί το τμήμα ΕΖ ως συνάρτηση του ρ.

Θεωρούμε έναν αριθμό α ::1= Ο και μια συνάρτη-ση f : JR -t JR , για την οποία ισχύει: [ f (χ) - 3] f (χ + a) = f (χ) - 5 , για κάθε χ ε JR . ( I ) . Να αποδείξετε ότι, για κάθε χ ε JR , ισχύουν: Ι ) f(x) ::1= 3, f(x) ::1= 2 και f(x) ::1= I . 2) f(x + 4α) = f(x) .

Καλοκαιρινή Κατασκήνωση : Σοφι κό Κορινθίας : Δημοτικό (Δ , Ε , Στ ' )

(Ιούλιος - Αύγουστος 20 1 2) Πληροφορ ίες Γραφε ία Ε .Μ.Ε . τηλ . : 2 103616532 -2 1036 17784, www.hms .gr - info@ hms.gr

Page 84: Ευκλειδης Β 81

ΔΗΜΗΤΡΗΣ ΝΤΑΒΟΣ

ΜΑθ Η ΜΑΤΙ ΚΑ

Μαθηματικά Γ -\υκεiοιι

θι;τικής & Τι:χ,·ολογικiις Κατι:ίι01ΙνΟ1)ς

Α ' Τόμος

ΜαθηματικΔ Γ Λv-

Β 'nΙμος

Παράγω-γος 1 Υ

• Ιiιφιιγι•ΙJΟ.::

• Εψυ:rrιψi-••ι

διάθεση_

> > . ΠΑΤΑΚΗΣ